Download as doc, pdf, or txt
Download as doc, pdf, or txt
You are on page 1of 181

FANPAGE TÀI LIỆU TIẾNG ANH NÂNG CAO

TEST 11
Part 1. For questions 1-13, read the following passage and do the tasks that follow.

Recent stock-market crashes


For as long as there have been financial markets, there have been financial crises. Most
economists agree, however, that from 1994 to 2013 crashes were deeper and the resultant
troughs longer-lasting than in the 20-year period leading up to 1994. Two notable
crashes, the Nifty Fifty in the mid-1970s and Black Monday in 1987, had an average loss
of about 40% of the value of global stocks, and recovery took 240 days each, whereas the
Dot-com and credit crises, post-1994, had an average loss of about 52%, and endured for
430 days. What economists do not agree upon is why recent crises have been so severe or
how to prevent their recurrence.

John Coates, from the University of Cambridge in the UK and a former trader for
Goldman Sachs and Deutsche Bank, believes three separate but related phenomena
explain the severity. The first is dangerous but predictable risk-taking on the part of
traders. The second is a lack of any risk-taking when markets become too volatile.
(Coates does not advocate risk-aversion since risk-taking may jumpstart a depressed
market.) The last is a new policy of transparency by the US Federal Reserve – known as
the Fed – that may have encouraged stock-exchange complacency, compounding the
dangerous risk-taking.

Many people imagine a trader to have a great head for maths and a stomach for the
rollercoaster ride of the market, but Coates downplays arithmetic skills, and doubts
traders are made of such stern stuff. Instead, he draws attention to the physiological
nature of their decisions. Admittedly, there are women in the industry, but traders are
overwhelmingly male, and testosterone appears to affect their choices.

Another common view is that traders are greedy as well as thrill-seeking. Coates has not
researched financial incentive, but blood samples taken from London traders who
engaged in simulated risk-taking exercises for him in 2013 confirmed the prevalence of
testosterone, cortisol, and dopamine – a neurotransmitter precursor to adrenalin
associated with raised blood pressure and sudden pleasure.

Certainly anyone faced with danger has a stress response involving the body’s
preparation for impending movement – for what is sometimes called ‘Fight or flight’, but,
as Coates notes, any physical act at all produces a stress response: even a reader’s eye
movement along words in this line requires cortisol and adrenalin. Neuroscientists now
see the brain not as a computer that acts neutrally, involved in a process of pure thought,
but as a mechanism to plan and carry out a movement, since every single piece of
information humans absorb has an attendant pattern of physical arousal.
FANPAGE TÀI LIỆU TIẾNG ANH NÂNG CAO

For muscles to work, fuel is needed, so cortisol and adrenalin employ glucose from other
muscles and the liver. To burn the fuel, oxygen is required, so slightly deeper or faster
breathing occurs. To deliver fuel and oxygen to the body, the heart pumps a little harder
and blood pressure rises. Thus, the stress response is a normal part of life, as well as a
resource in fighting or fleeing. Indeed, it is a highly pleasurable experience in watching
an action movie, making love or pulling off a multi-million-dollar stock-market deal.

Cortisol production also increases during exposure to uncertainty. For example, people
who live next to a train line adjust to the noise of passing trains, but visitors to their home
are disturbed. The phenomenon is equally well-known of anticipation being worse than
an event itself: sitting in the waiting room thinking about a procedure may be more
distressing than occupying the dentist’s chair and having one. Interestingly, if a patient
does not know approximately when he or she will be called for that procedure, cortisol
levels are the most elevated of all. This appeared to happen with the London traders
participating in some of Coates’ gambling scenarios.

When there is too much volatility in the stock market, Coates suspects adrenaline levels
decrease while cortisol levels increase, explaining why traders take fewer risks at that
time. In fact, typically traders freeze, becoming almost incapable of buying or selling
anything but the safest bonds. In Coates’ opinion, the market needs investment as it falls
and at rock bottom – at such times, greed is good.

The third matter – the behaviour of the Fed – Coates thinks could be controlled, albeit
counterintuitively. Since 1994, the US Federal Reserve has adopted a policy called
Forward Guidance. Under this, the public is informed at regular intervals of the Fed’s
plans for short-term interest rates. Recently, rates have been raised by small but
predictable increments. By contrast, in the past, the machinations of the Fed were largely
secret, and its interest rates fluctuated apparently randomly. Coates hypothesises these
meant traders were on guard and less likely to indulge in wild speculation. In introducing
Forward Guidance, the Fed hoped to lower stock and housing prices; instead, before the
crash of 2008, the market surged from further risk-taking, like an unleashed pit bull
terrier.

There are many economists who disagree with Coates, but he has provided some
physiological evidence for both traders’ recklessness and immobilisation and made the
radical proposal of greater opacity at the Fed. Although, as others have noted, we could
just let more women onto the floor.

Questions 1-7
In boxes 1-7, write:
True if the statement agrees with the information
False if the statement contradicts the information
Not Given if there is no information on this
FANPAGE TÀI LIỆU TIẾNG ANH NÂNG CAO

1. Economic experts generally agree that financial crashes around the 2000s were
less severe than those between 1974 and 1993.
2. According to John Coates, risk-taking can sometimes assist a weak market.
3. Contrary to popular belief, traders tend to show strong determination to deal with
market volatility.
4. The analysis of blood samples collected from London traders in 2013 revealed that
many of them suffered from high blood pressure and instant gratification.
5. Neuroscientists now view the brain as an organ to control movement.
6. A person waiting to see a dentist may have extremely high cortisol levels if he or
she does not have a specific appointment.
7. An increase in female traders may be a potential solution to current market
problems.
Your answers.

1. 2. 3. 4. 5. 6. 7.

For questions 8-13, complete the following paragraph with words taken from the
passage (NO MORE THAN THREE WORDS for each blank).
The global financial status has hit a dead-cat bounce. Such severity of recent crashes can
be unraveled and analyzed through some following explanations by John Coates, an
experienced trader for Goldman Sachs and Deutsche Bank. Firstly, traders are now more
willing to take risks, especially when the market is rising. With a proven increase in their
8) ________________, they tend to challenge themselves with the market. Secondly,
traders may become more averse to risks. This occurs when the stock market witnesses a
plethora of 9) ________________, which may result in a rise in traders’ 10)
________________. Consequently, we can see their immobilization. The third reason
may derive from US Federal Reserve policy, also known as 11) ________________,
which was supposed to calm the market. In the past, the uncertainty of interest rates
might discourage traders from making 12) _______________. With Fed’s plans for short-
term interest rates being informed more regularly, traders became less cautious and 13)
________________ was encouraged.
Part 2. For questions 14-23, read an extract from an article and choose the answer
A, B, C or D that fits best according to the text. Write your answers in the
corresponding numbered boxes provided.
The machine that is celebrity culture has given us the meteoric rise and fall of the child
actor, with plenty of cautionary tales to point to and ask if something should have been
done to prevent them. Recently, the Chinese government took the extreme and
unprecedented measure of banning the children of celebrities from appearing in any type
FANPAGE TÀI LIỆU TIẾNG ANH NÂNG CAO

of reality TV programming, in an effort to prevent the manufacturing of child stars. It


would appear that perhaps limiting the exposure a child has to fame serves to protect and
ensure a solid, stable upbringing.

The pressure of fame is undoubtedly onerous, even for adults, who, despite growing up
out of the spotlight, sometimes buckle under the stress of stardom they achieved later in
life and exhibit all manner of behavioral disorders after their stardom has waned. The
same can be said of child actors, but the effect is seemingly multiplied by the fact that, if
achieving stardom as children, their view of reality is possibly warped and they may
never even have the chance to acquire the necessary coping skills. But given that some
child actors – in fact, most – can make a go of their careers into adulthood, are children
really so incapable of handling such pressure or is there actually no problem at all?

■ A) Banning children from acting has an element of common sense to it, but imagine, if
you will, television programmes, films and so forth absent of children. ■ B) While the
Chinese government’s move to limit the exposure of children may seem well intentioned,
at least on the surface, it is not entirely realistic to say that children are not allowed to
appear on the small or big screen. ■ C) Of course, they are applying it to one particular
media – that of reality TV; nonetheless, is such a ban sensible for any type of media? ■
D)

Upon closer examination of the phenomenon of the child star, we see examples both of
success and failure. How many of each do we have? Is there a disproportionately high
amount of failure in the lives of child actors if we look at the statistics and compare their
problems with those of ordinary people? We see a child star fail and we immediately
blame fame, but what about the success stories of other child actors such as Jodie Foster,
Daniel Radcliffe and Leonardo Dicaprio, all of whom got their start as very young
children? Are we to credit fame for their success in the same way we blame it for others’
failures?

In the case of the latter, there are the stars we know about, as they went on to achieve
long-lasting fame, even top acting awards. Child stars are not always destined to eternally
seek the limelight, however, so there are many cases of success stories that people often
don’t know about. Peter Ostum, who played Charlie Bucket in Willy Wonka & the
Chocolate Factory, went on to pursue a doctorate in veterinary medicine. Shirley Temple,
leading box-office star in the 1930s from the age of seven, became a politician and the
first female US ambassador. Polish child stars and identical twin brothers Lech and
Jaroslaw Kaczynski gave up acting and were respectively elected as president and prime
minister of Poland, positions they held at the same time.

Invariably, though, it’s the catastrophic demise that we hear about, not just of child
actors, of course, but when it does happen to them, we feel a mixture of sorrow and
disbelief. To data, there is little statistical evidence to support the claim that fame and
FANPAGE TÀI LIỆU TIẾNG ANH NÂNG CAO

celebrity culture ruin the lives of child actors; the only proof we have is what we perceive
to be true. Protections are in place, to an extent, to help ensure that children have as
normal an upbringing as possible. California, for example, has enacted laws which
mandate that children must continue with their educational studies exactly as they would
if they weren’t in films, even going so far as to require teachers on set if need be. In this
vein, ensuring support for child actors may need to go further than the broad restrictions
exercised by China.

14. Which best serves as the title for the passage?


A. Too young to be famous
B. The lucrative career for children
C. How to become successful from an early age
D. Laws and regulations restricting child participation in the showbiz

15. The word " cautionary tales” in paragraph 1 mostly means


A. stories that are memorable
B. jokes and tricks to warn children
C. stories that give warning to people
D. hilarious conversations

16. What can be inferred about the solution taken by Chinese government regarding child
stars?
A. The author wholeheartedly subscribes to it.
B. It will hamper the boom of child stars.
C. It will impose a ban on children participation in films.
D. It is unprecedented in history.

17. In the second paragraph, the author implies that children


A. are better prepared to deal with fame than adults.
B. never learn coping skills when they are famous young.
C. may or may not be perniciously influenced by fame.
D. are destined to become abnormal adults if they are in the limelight from an early
age.

18. What is the author’s opinion of the government ban in the third paragraph?
A. He is not sure whether or not it originates from good will.
B. He is in support of it being put into practice.
C. He supposes it is futile in the context of reality TV.
D. He believes the drive behind is to manipulate the media.

19. In the fourth paragraph, the author suggests that


A. fame is the culprit behind the phenomenon of child stars.
FANPAGE TÀI LIỆU TIẾNG ANH NÂNG CAO

B. fame may not contribute to the flop of a star.


C. the fame of some stars shows it has no detrimental impact.
D. the failure of some stars means that fame is debilitating.

20. The author uses the instances in the fifth paragraph to convey the idea that
A. child actors can attain whatever they desire.
B. fame can repel some from a sustainable acting career.
C. a more thorough study is needed to give out evaluation.
D. success can be a springboard to other positions in life.

21. The author concludes by saying that


A. it is futile to protect children from the perils of fame
B. it is worth taking into account the steps to help children handle fame
C. our conception of fame is greater than the reality
D. children should receive insightful education about how to deal with fame

22. Look at the four squares [■] that indicate where the following sentence could be
added to the passage.
As this sort of media is supposed to reflect real life, it would seem surreal if there
were no children in these stories, as if children had ceased to exist altogether
Where would the sentence best fit?
A. First square B. Second square C. Third square D. Fourth square

23. The word " mandate” in the final paragraph mostly means
A. giving official permission for something to happen
B. ordering something to happen
C. putting something into action
D. allowing for the existence of something

Your answers
14. 15. 16. 17. 18.

19. 20. 21. 22. 23.

Part 3. In the passage below, seven paragraphs have been removed. For questions
24-30, read the passage and choose from the paragraphs A-H the one which fits each
gap. There is ONE extra paragraph which you do not need to use. Write your
answers in the corresponding numbered box provided.

Non-Verbal communication
FANPAGE TÀI LIỆU TIẾNG ANH NÂNG CAO

Sociological research points to the theory that certain ways of positioning or moving the
body have a direct correlation with how one is perceived. People emit an aura of strength
or power dependent on posture, gestures and eye movement. Quick, enthusiastic,
meaningful movements and gestures suggest a dynamic, alert person. People who look at, and
maintain eye contact with their audience while conversing with them exude confidence and
fearlessness.

24.

Being conscious of one's posture and gestures when sitting is also conducive to creating
the right impression on the beholder. When one wishes to appear self-assured and
knowledgeable in an important interpersonal situation where sitting is required, a high,
straight-backed chair should be chosen when possible. Placing and clasping the ha nds
behind the head, with elbows stretched to the sides, adds to the impression of comfortable
assertiveness. It also keeps the hands under control and out of danger of unwanted fidgeting.

25.

The appropriate placing of the hands depends on the situation. When one is in an upright
position, standing face to face with another person, the palms of the hands should not be evident.
Hands open in a palm-revealing gesture can connote a begging or submissive attitude,

26.

Research shows that people in business have adopted a standard routine. Many
professionals assume a hand-to-chin position when seated and when engaged in
discussion, they will sit erectly in their chair with one leg crossed casually over the other,
seemingly calm and cool. The moment the discussion begins to take on a more serious
note, the professional will uncross the legs, place both feet squarely on the floor and lean
forward to indicate assertiveness and strength. It does not do, though, to be too defensive.

27.

Staring or glaring eyes flash "threat or warning" to both humans and animals, How often have
we observed a dog with hackles raised, staring down an opponent? Just as beasts send signals of
FANPAGE TÀI LIỆU TIẾNG ANH NÂNG CAO

dominance in this fashion, so do humans. The eyes can communicate other emotions as well.
Eyes that blink rapidly and shift from side to side give the illusion of nervousness or
untrustworthiness.

28.

In some situations, staring, as opposed to maintaining eye contact, can have a negative
effect. In addition to being regarded as rude, star ing et someone can indeed make them back
down or surrender. Adversely, it could antagonise them into accepting the suggested challenge.

29.

This phenomenon proves to be even more evident when a person takes on the role of
listener. In almost all social scenarios, people demonstrate a heightened sense of
attentiveness when a perceived person of power speaks. Whether one is discussing a
teacher in a classroom or a company president in the boardroom, all eyes seem to be
riveted on the sage, advice-giving, more dominant speaker.

30.

It is said that we form impressions of people within the first thirty seconds of meeting
them. Within this time period, the beholder is strongly influenced by a person's stance,
gestures and physical actions. When someone is aware of this, they are far more prepared
to consciously use the tools of impression management to control the overall impression
they make on those around them.

The missing paragraphs

A. In the instances where a speaker wishes to present and reinforce an impression of


honesty and powerfulness on his audience, body language may speak more loudly and
more effectively than the actual spoken words.

B. Just as staring can be offensive at times, looking at someone too frequently or for too
long a duration can also suggest something negative; namely submissiveness. With
regard to the higher primates and humans, both occupy themselves for longer periods of
time observing their superiors as opposed to their inferiors.
FANPAGE TÀI LIỆU TIẾNG ANH NÂNG CAO

C. Just as posture plays a role in impression management, so too do the eyes. The eyes
can communicate a million words without the owner consciously knowing they are doing
so. The eyes are the 'mirror of the soul" and by moving them, or using them in certain
ways, diverse messages can be relayed to an observer.

D. There are times when one's hands seem to have a 'mind of their own'. Unconsciously,
one might tap their fingers on the arms of a chair, fidget with jewellery, or even crack
one's knuckles. All of these actions are very clear signs of boredom, nervousness or
inattentiveness.

E. A flatterer is a person who controls his impression management in such a way that the
message being sent is one of a likeable, friendly person. The flatterer's aim is to be seen
only in a positive light and will tell an audience exactly what they want to hear to create
this aura.

F. Somebody who is relaxed enough to stand before his audience without any visual signs
of stress exudes self-assuredness and honesty. Even though these people are comfortable
in their stance, they hold themselves erect and avoid looking round-shouldered or
hunched over. Being lazy with one's posture could be indicative of defeat, while standing
tall and proud paints a picture of one who is in charge.

G. It is said that many politicians are very aware of this impression that the eyes can
make on an audience. Many insist that there be no movement in a television studio where
they are being taped, for fear that their eyes may automatically flit to the sudden
movement. Obviously, no politician wants to appear to be lying or feeling uncomfortable
about what he is saying during a live taping of a political debate or speech.

H. When sitting, many people make the mistake of folding their arms across their chests.
This gives the impression of being closed off or inaccessible to the audience. By rights,
the hands should be clasped loosely, with fingers interlaced, and placed in the lap. This
gesture indicates an open, friendly attitude to the audience.

Your answers
24. 25. 26. 27. 28. 29. 30.

Part 4. The passage below consists of four sections marked A, B, C and D. For
questions 31-40, read the passage and do the task that follows. Write your answers
(A, B, C or D) in the corresponding numbered boxes provided.
FANPAGE TÀI LIỆU TIẾNG ANH NÂNG CAO

A. Oak
Oak wood has a density of about 0.75 g/cm3, great strength and hardness, and is very
resistant to insect and fungal attack because of its high tannin content. It also has very
appealing grain markings, particularly when quartersawn. Oak planking was common on
high status Viking longships in the 9th and 10th centuries. The wood was hewn from
green logs, by axe and wedge, to produce radial planks, similar to quarter-sawn timber.
Wide, quarter-sawn boards of oak have been prized since the Middle Ages for use in
interior paneling of prestigious buildings such as the debating chamber of the House of
Commons in London, and in the construction of fine furniture. Oak wood, from Quercus
robur and Quercus petraea, was used in Europe for the construction of ships, especially
naval men of war, until the 19th century, and was the principal timber used in the
construction of European timber-framed buildings.
Today oak wood is still commonly used for furniture making and flooring, timber frame
buildings, and for veneer production. Barrels in which wines, sherry, and spirits such as
brandy, Scotch whisky and Bourbon whiskey are aged are made from European and
American oak. The use of oak in wine can add many different dimensions to wine based
on the type and style of the oak. Oak barrels, which may be charred before use, contribute
to the colour, taste, and aroma of the contents, imparting a desirable oaky vanillin flavour
to these drinks. The great dilemma for wine producers is to choose between French and
American oakwoods. French oaks give the wine greater refinement and are chosen for the
best wines since they increase the price compared to those aged in American oak wood.
American oak contributes greater texture and resistance to ageing, but produces more
violent wine bouquets. Oak wood chips are used for smoking fish, meat, cheeses and
other foods.
B. Elm
Elm wood was valued for its interlocking grain, and consequent resistance to splitting,
with significant uses in wagon wheel hubs, chair seats and coffins. The elm's wood bends
well and distorts easily making it quite pliant. The often long, straight, trunks were
favoured as a source of timber for keels in ship construction. Elm is also prized by
bowyers; of the ancient bows found in Europe, a large portion of them are elm. During
the Middle Ages elm was also used to make longbows if yew was unavailable.
The first written references to elm occur in the Linear B lists of military equipment at
Knossos in the Mycenaean Period. Several of the chariots are of elm, and the lists twice
mention wheels of elmwood. Hesiod says that ploughs in Ancient Greece were also made
partly of elm.
The density of elm wood varies between species, but averages around 560 kg per cubic
metre.
Elm wood is also resistant to decay when permanently wet, and hollowed trunks were
widely used as water pipes during the medieval period in Europe. Elm was also used as
piers in the construction of the original London Bridge. However this resistance to decay
in water does not extend to ground contact.
The Romans, and more recently the Italians, used to plant elms in vineyards as supports
for vines. Lopped at three metres, the elms' quick growth, twiggy lateral branches, light
FANPAGE TÀI LIỆU TIẾNG ANH NÂNG CAO

shade and root-suckering made them ideal trees for this purpose. The lopped branches
were used for fodder and firewood. Ovid in his Amores characterizes the elm as "loving
the vine", and the ancients spoke of the "marriage" between elm and vine.
C. Mahogany
Mahogany has a straight, fine and even grain, and is relatively free of voids and pockets.
Its reddish-brown color darkens over time, and displays a reddish sheen when polished. It
has excellent workability, and is very durable. Historically, the tree's girth allowed for
wide boards from traditional mahogany species. These properties make it a favorable
wood for crafting cabinets and furniture.
Much of the first-quality furniture made in the American colonies from the mid-18th
century was made of mahogany, when the wood first became available to American
craftsmen. Mahogany is still widely used for fine furniture; however, the rarity of Cuban
mahogany and over harvesting of Honduras and Brazilian mahogany has diminished their
use. Mahogany also resists wood rot, making it attractive in boat construction. It is also
often used for musical instruments, particularly the backs, sides and necks of acoustic
guitars and drum shells because of its ability to produce a very deep, warm tone
compared to other commonly used woods such as maple or birch. Guitars often feature
mahogany in their construction. Mahogany is now being used for the bodies of high-end
stereo phonographic record cartridges and for stereo headphones, where it is noted for
“warm” or “musical” sound.
D. Beech
Beech wood is an excellent firewood, easily split and burning for many hours with bright
but calm flames. Chips of beech wood are used in the brewing of Budweiser beer as a
fining agent. Beech logs are burned to dry the malts used in some German smoked beers,
giving the beers their typical flavour. Beech is also used to smoke Westphalian ham,
various sausages, and some cheeses.
The European species Fagus sylvatica yields a utility timber that is tough but
dimensionally unstable. It weighs about 720 kg per cubic metre and is widely used for
furniture framing and carcass construction, flooring and engineering purposes, in
plywood and in household items like plates, but rarely as a decorative wood. The timber
can be used to build chalets, houses and log cabins.
Beech wood is used for the stocks of military rifles when traditionally preferred woods
such as walnut are scarce or unavailable or as a lower-cost alternative.
The fruit of the beech tree is known as beechnuts or mast and is found in small burrs that
drop from the tree in autumn. It is small, roughly triangular and edible, with a bitter,
astringent taste. They have a high enough fat content that they can be pressed for edible
oil. Fresh from the tree, beech leaves are a fine salad vegetable, as sweet as a mild
cabbage though much softer in texture. The young leaves can be steeped in gin for
several weeks, the liquor strained off and sweetened to give a light green/yellow liqueur
called beech leaf noyau.

Choose from the list of woods (A-D) for each question. Some of the choices may be
required more than once.
FANPAGE TÀI LIỆU TIẾNG ANH NÂNG CAO

31. Which wood contains something that prevents it being attacked?


32. Which wood doesn't have the reputation of being aesthetically pleasing?
33. Which wood can be buried in water indefinitely with little ill effect?
34. Which wood can enhance the value a food or drink?
35. Which wood is most likely to be found on stage at a rock concert?
36. Which wood became associated with luxurious buildings?
37. Which wood is the most flexible and is therefore used where this is required?
38. Which wood is most inflammable?
39. Which wood was used as an agricultural aid?
40. Which wood can alter its colour?

Your answers
31. 32. 33. 34. 35.

36. 37. 38. 39. 40.

Key and explanations:


1. F
Most economists agree, however, that from 1994 to 2013 crashes were deeper and the
resultant troughs longer-lasting than in the 20-year period leading up to 1994.
FANPAGE TÀI LIỆU TIẾNG ANH NÂNG CAO

2. T
Coates does not advocate risk-aversion since risk-taking may jumpstart a depressed
market.
3. F
Many people imagine a trader to have a great head for maths and a stomach for the
rollercoaster ride of the market.
4. NG
5. T
Neuroscientists now see the brain not as a computer that acts neutrally, involved in a
process of pure thought, but as a mechanism to plan and carry out a movement, since
every single piece of information humans absorb has an attendant pattern of physical
arousal.
6. T
Interestingly, if a patient does not know approximately when he or she will be called
for that procedure, cortisol levels are the most elevated of all.
7. T
Although, as others have noted, we could just let more women onto the floor.
8. adrenaline levels
9. volatility
10. cortisol levels
11. Forward Guidance
12. wild speculation
13. further risk-taking
Part 2.
14. A (the most appropriate and wholesome title about the phenomenon of child stars
elaborated in the passage)

15. C (cautionary tale: câu chuyện để cảnh báo)

16. D (Recently, the Chinese government took the extreme and unprecedented measure of
banning the children of celebrities from appearing in any type of reality TV
programming, in an effort to prevent the manufacturing of child stars)
FANPAGE TÀI LIỆU TIẾNG ANH NÂNG CAO

17. C (But given that some child actors – in fact, most – can make a go of their careers
into adulthood, are children really so incapable of handling such pressure or is there
actually no problem at all?)

18. A (While the Chinese government’s move to limit the exposure of children may seem
well intentioned, at least on the surface, it is not entirely realistic to say that children
are not allowed to appear on the small or big screen)

19. B (We see a child star fail and we immediately blame fame, but what about the
success stories of other child actors such as Jodie Foster, Daniel Radcliffe and Leonardo
Dicaprio, all of whom got their start as very young children? Are we to credit fame for
their success in the same way we blame it for others’ failures?)

20. C (In the case of the latter, there are the stars we know about, as they went on to
achieve long-lasting fame, even top acting awards. Child stars are not always destined to
eternally seek the limelight, however, so there are many cases of success stories that
people often don’t know about)

21. B (Protections are in place, to an extent, to help ensure that children have as normal
an upbringing as possible)

22. B (“this sort of media” refers to television programmes, “absent of children” refers to
the phrase “there were no stories” )

23. A (mandate = give official permission for sth to happen)

Part 3.

24. F

Somebody who is relaxed enough to stand before his audience without any visual signs
of stress exudes self-assuredness and honesty. Being lazy with one's posture could be
indicative of defeat, while standing tall and proud paints a picture of one who is in
charge… Being conscious of one's posture and gestures when sitting is also conducive
to creating the right impression on the beholder.

25. D

There are times when one's hands seem to have a 'mind of their own'… The appropriate
placing of the hands depends on the situation.

26. H
FANPAGE TÀI LIỆU TIẾNG ANH NÂNG CAO

When one is in an upright position, standing face to face with another person, the
palms of the hands should not be evident… When sitting, many people make the
mistake of folding their arms across their chests.

27. C

Just as posture plays a role in impression management, so too do the eyes… Staring or
glaring eyes flash "threat or warning" to both humans and animals, How often have we
observed a dog with hackles raised, staring down an opponent? Just as beasts send signals
of dominance in this fashion, so do humans. The eyes can communicate other emotions
as well.

28. G

Eyes that blink rapidly and shift from side to side give the illusion of nervousness or
untrustworthiness… Many insist that there be no movement in a television studio where
they are being taped, for fear that their eyes may automatically flit to the sudden
movement.

29. B

With regard to the higher primates and humans, both occupy themselves for longer
periods of time observing their superiors as opposed to their inferiors… In almost all
social scenarios, people demonstrate a heightened sense of attentiveness when a
perceived person of power speaks

30. A

In the instances where a speaker wishes to present and reinforce an impression of honesty
and powerfulness on his audience, body language may speak more loudly and more
effectively than the actual spoken words…It is said that we form impressions of people
within the first thirty seconds of meeting them. Within this time period, the beholder is
strongly influenced by a person's stance, gestures and physical actions.

Part 4.

31. A

Oak wood has a density of about 0.75 g/cm3, great strength and hardness, and is very
resistant to insect and fungal attack because of its high tannin content.
FANPAGE TÀI LIỆU TIẾNG ANH NÂNG CAO

32. D - It weighs about 720 kg per cubic metre and is widely used for furniture framing
and carcass construction, flooring and engineering purposes, in plywood and in
household items like plates, but rarely as a decorative wood.

33. B - Elm wood is also resistant to decay when permanently wet, and hollowed
trunks were widely used as water pipes during the medieval period in Europe.

34. A - The use of oak in wine can add many different dimensions to wine based on the
type and style of the oak … Oak barrels, which may be charred before use, contribute to
the colour, taste, and aroma of the contents, imparting a desirable oaky vanillin
flavour to these drinks.

35. C - It is also often used for musical instruments, particularly the backs, sides and
necks of acoustic guitars and drum shells because of its ability to produce a very deep,
warm tone compared to other commonly used woods such as maple or birch. Guitars
often feature mahogany in their construction.

36. A - Wide, quarter-sawn boards of oak have been prized since the Middle Ages for use
in interior paneling of prestigious buildings…

37. B - The elm's wood bends well and distorts easily making it quite pliant.

38. D - Beech wood is an excellent firewood, easily split and burning for many hours
with bright but calm flames. Chips of beech wood are used in the brewing of Budweiser
beer as a fining agent.

39. B - The Romans, and more recently the Italians, used to plant elms in vineyards as
supports for vines. Lopped at three metres, the elms' quick growth, twiggy lateral
branches, light shade and root-suckering made them ideal trees for this purpose.

40. C - Its reddish-brown color darkens over time, and displays a reddish sheen when
polished.

TEST 12
Part 1. For questions 1-13, read the following passage and do the tasks that follow.

Bismarck: A master of political and diplomatic juggling?


A. Otto Von Bismarck’s rise up the political ladder was swift and relentless. Having
entered parliament in 1847, he always harboured lofty ambitions, chief among them
perhaps being the reunification of Germany into one strong, centrally controlled state,
though his own personal thirst for power was arguably even stronger. On becoming
FANPAGE TÀI LIỆU TIẾNG ANH NÂNG CAO

Prussian Chancellor, he set about fulfilling his ambitions and in doing so proved himself
to be a diplomat of some considerable skill. Victory in the Austro-Prussian war
effectively ended Austria as a factor in German affairs. His political and military juggling
was taken a step further when he orchestrated a situation where France declared war on
Germany in 1870, making the French seem responsible for a conflict he had always
intended to create. And following another swift military triumph, this time over the
French, the German empire was proclaimed in January 1871.
B. In little more than nine years, Bismarck realised his lifelong ambition, steering
Germany to reunification. And by defeating Austria and France in quick succession, he
also created a power vacuum on mainland Europe, which he was determined to fulfill
himself. This was another opportunity for Bismarck to demonstrate his political and
diplomatic cunning. He set about creating a dictatorial Germany in which he, as head of
the Prussian parliament, would automatically become chancellor of the German empire.
He drafted a new German constitution to suit his own purposes and, despite maintaining a
veneer of democracy, the German parliament was effectively powerless to oppose him.
Provinces that were slow to support him were enticed with bribes and before long the
German empire was his to command.
C. It is a testament to his political skill that Bismarck achieved so much so quickly. At
this point in his colourful political career, he did appear, for all intents and purposes, a
master of political and diplomatic juggling. But challenges lay ahead and Bismarck’s
next target was the Catholic church, which he deemed too powerful and a threat to his
political dominance. He proceeded to enact a series of laws that seriously eroded the
power of the church. However, his plans backfired and Bismarck was forced to make a
political U-turn. Though here again, he somehow managed to save face. The damage to
his reputation was limited and indeed by the late 1870s, he had even managed to win over
the church whose support he now needed.
D. Bismarck viewed the growing popularity of the Socialist Democratic Party as a serious
threat. He bided his time and used the attempted assassination of the Kaiser as an excuse
to attack the socialists in 1878, blaming them for the attempt on the Kaiser’s life. He
immediately arrested the leaders, banned party meetings, and suppressed socialist
newspapers. But despite his efforts to destroy the socialist movement, its popularity had
trebled by 1890. Just as his interventions with the church had not gone as planned,
Bismarck once again failed to achieve his objective; though, to his credit, he held on to
power.
E. His domestic position was relatively secure after 1871, Bismarck devoted a lot of his
time to foreign policy. Having used war to unite Germany and make her great, Bismarck
now believed that his ambitions were best served by peace. His plan to isolate a hostile
France would require all his considerable diplomatic skills. The Dreikaiserbund
FANPAGE TÀI LIỆU TIẾNG ANH NÂNG CAO

agreement of 1873 between Germany, Austria-Hungary, and Russia was a first step
towards doing just that. The Balkan crisis, a conflict involving Russia and Austria-
Hungary, severely tested his diplomatic credentials, but his answer was to offer himself
as an ‘honest broker’ to help resolve the dispute. The subsequent Congress of Berlin
which he hosted was an outstanding success and only served to reinforce Bismarck’s
reputation as a shrewd diplomat. Bismarck’s foreign policy would continue in this vein
throughout his reign as Chancellor. He built up strategic alliances with the big powers,
Russia, Italy, and Austria-Hungary, in the hope that he could keep his main threats,
France and Britain, isolated.
F. In truth, Bismarck’s reign as chancellor of the German empire does seem to confirm
him as a shrewd and wily diplomat and politician, one whose objectives were broadly
achieved. Does this mean his so-called juggling was a success? Perhaps, but Bismarck
left a less than perfect legacy. He created a Germany in which the Kaiser had the ultimate
say in domestic affairs and enjoyed far too much power should he choose to wield it. This
meant that the future of the empire largely depended on the strength and character of just
one man, the Kaiser. A weak Kaiser would be disastrous for the country’s welfare, and so
it would soon prove. In the final analysts, Bismarck put Germany back on the map again
as a great power during his reign, but we should not forget that he created the political
situation that would be the downfall of his country in the end. His political and
diplomatic juggling, therefore, simply cannot be considered a total success.
Questions 1-6
Reading passage has six paragraphs A-F.
Choose the correct heading, i - ix, from the list of headings below.
Write the correct number i-ix in the boxes 1-6.
i The pace at which Bismarck advanced his ambitions
ii Separating church and state
iii A change of mind
iv Unexpected ramifications of socialist threat
v Unremitting efforts to create a virtual dictatorship
vi External objectives and activities
vii Abortive attempts to curb a movement
viii Political power on the wane
ix A critical analysis
FANPAGE TÀI LIỆU TIẾNG ANH NÂNG CAO

1. Paragraph A
2. Paragraph B
3. Paragraph C
4. Paragraph D
5. Paragraph E
6. Paragraph F
Your answers

1. 2. 3. 4. 5. 6.

Questions 7-13
Do the following statements agree with the information given in the Reading
Passage?
In boxes 7-13, write
TRUE if the statement agrees with the information
FALSE if the statement contradicts the information
NOT GIVEN if there is no information on this
7. Germany defeated France and Austria successively to claim the dominant position in
Europe.
8. The Congress of Berlin was a great personal success for Bismarck.
9. After Bismarck enacted laws to weaken the Catholic Church, he was forced to change
his policy and made a church an ally.
10. The Socialist Democratic Party took power from Bismarck in 1890.
11. Bismarck’s foreign policy was to wage war with all countries that posed a military
threat.
12. Bismarck considered his reign as German chancellor a failure.
13. It can be concluded that Bismarck was not actually a master of political juggling.
Your answers
FANPAGE TÀI LIỆU TIẾNG ANH NÂNG CAO

7. 8. 9. 10. 11. 12. 13.

Part 2. For questions 14-23, read an extract from an article and choose the answer
A, B, C or D that fits best according to the text. Write your answers in the
corresponding numbered boxes provided

Super memorisers

There are people in this world who are innately possessed of an ability to remember
things with quite an extraordinary degree of detail and exactness. These super
memorisers, as they are known, typically possess a brain naturally and distinctively wired
to maximize its memorizing potential. They are gifted such that they require no particular
training or effort to sharpen their memories; they remember things just as effortlessly as
most people forget them. Few of us are born with such gifts, sadly, but there is much
cause for optimism yet for those looking to improve their brain’s performance tangibly in
this area; a fact which Boris Konrad is testament to.

Konrad is a champion memoriser who, in winning gold in the German Memory


Championships one year recognized and recalled the names of 195 people in just fifteen
minutes. [■] His powers of recollection, then, are as admirable as they are undisputed, but
Konrad is self-trained and started out with a recall capacity that was unremarkable. [■]
Instead, he spent years developing memory strategies and employing those strategies to
improve in the area through practice and dedication. [■] He, then, is a living proof that
the average Joe with a gift for forgetfulness can reinvent himself in the area. [■]However,
his example is as much a reminder of the extent of devotion that is required to reach this
level as it is of the possibilities if one is prepared to put in the effort, and there are not
many people prepared to expend a similar level of effort to this end, which is what really
makes Konrad unique.

That said, whilst you may not become a super memoriser overnight, new research
suggests that it is possible to tangibly improve your memory in a relatively short space of
FANPAGE TÀI LIỆU TIẾNG ANH NÂNG CAO

time by devoting roughly half an hour of your every day to the process. It is necessary to
learn and employ memorizing strategies such as the Memory Palace technique Konrad
uses, though, to yield such results; otherwise, you might be as well be doing something
else. In one recent study, for example, participants spent one month training their
memories in the aforementioned technique for 30 minutes every day, which more than
doubled their ability to remember list of information after just 40 days. More
impressively, recall performance remained high whether or not training continued at the
end of one month, which suggests after rewiring of the brain can be permanent.

Even innately gifted memorisers use such mnemonic techniques to enhance their recall
ability, and, of the recall methods which exist, the methods of loci, which has already
been referred to here as the Memory Palace method, is the most prevalent one adopted as
revealed by a recent study of 35 memory champions. Indeed, at a more rudimentary level,
this method has been employed by orators and others required to remember long strings
of interconnected information for some time, and it actually dates back to Ancient
Greece, where it was first conceived of, remaining prevalent right through to the Middle
Ages and the Renaissance. It is only a new method then, today, in the sense that the skill
had been lost as such to most people for a very long time. Indeed, many of today’s super
memorisers employed it intuitively rather than in a conscious effort to enhance their
recall powers.

Participants in the study was separated into three distinct groups, with one group
receiving no memory training whatsoever and making no notable recall gains during the
process. The second group dedicated time and effort to upping their recall capacity, but
employed an everyday technique. However, those using the Memory Palate technique
more than doubled their initial recall capacity by the process’s end. What’s more, their
brain functions actually changed and their brain patterns began to bear more of a likeness
to those of innate super memorisers, leading researchers to speculate that a total rewire
might well be possible over time, such that a normal individual’s recall capacity could
match that of any memory champion. The conclusion, therefore, was that memory is not
FANPAGE TÀI LIỆU TIẾNG ANH NÂNG CAO

necessarily an innately bestowed gift you either are possessed of or not. Most of us have
the potential to hone and expand our memories very meaningfully indeed.

14. According to the first paragraph, people with marvelous power of retention usually
A. undergo rigorous practice to become that way.
B. demonstrate brain functions resembling anyone else.
C. throw in little effort to commit things to memory.
D. be tangibly more intelligent than other people.
15. Why are Konrad’s achievements mentioned in paragraph two?
A. To underline the excellence of inborn super memorisers
B. To demonstrate the disparity between laymen and super memorisers
C. To prove that almost every individual can make a significant enhancement in their
retention powers
D. To show how even innate super memorisers have to exert themselves
16. What does the writer mean in the phrase ‘you might as well be doing something else’
in paragraph three?
A. It is futile to invest time in improving recall capacity.
B. People should concentrate on their strengths.
C. The Memory Palace technique is not universally suitable.
D. Progress is only visible if you train in the right way.
17. What did the study mentioned in the third paragraph conclude?
A. The implication of moderate brain exercises can be enduring.
B. The maintenance of a decent recall capacity requires continuous training.
C. Your recall ability can more than double within one month.
D. Progress is only noticeable with at least 30 minutes spent on brain training.
18. What do we learn about the method of loci?
A. It is superior to the Memory Palace method.
B. Most skillful memorisers purposefully adopt it.
C. It has evolved from methods first employed in Ancient Greece.
FANPAGE TÀI LIỆU TIẾNG ANH NÂNG CAO

D. It has been widely used among super memorisers.


19. What did the results of the study mentioned in the final paragraph reveal?
A. How the brains of super memorisers function is inimitable.
B. The application of appropriate technique exerts a tremendous influence on the
brain.
C. Memory training is futile unless employing a confirmed technique.
D. Most super memorisers are not talented at memorizing at birth.
20. The word "mnemonic techniques” in paragraph 4 mostly means
A. gimmicks used to improve brain functions.
B. endowed talents of super memorisers.
C. tips specifically designed to aid the process of retaining information.
D. skills adopted by super memorisers to avoid absent-mindedness.
21. Look at the four squares [■] that indicate where the following sentence could be
added to the passage.
Indeed, after just 30 seconds, examination, he is able to retain the order of an
entire deck of cards.
Where would the sentence best fit?
A. First square
B. Second square
C. Third square
D. Fourth square
22. The word "orators” in paragraph 4 mostly means
A. language experts
B. body language experts
C. oral students
D. proficient speakers
23. The paragraph following the passage would most probably discuss
A. further research into the workings of brains of super memorisers.
FANPAGE TÀI LIỆU TIẾNG ANH NÂNG CAO

B. the defeat of a normal person over a super memoriser in terms of recalling


capacity.
C. potential ways to expand our power of retention.
D. the popularity of memory palace technique.

Your answers
14. 15. 16. 17. 18.

19. 20. 21. 22. 23.

Part 3. In the passage below, seven paragraphs have been removed. For questions
24-30, read the passage and choose from the paragraphs A-H the one which fits each
gap. There is ONE extra paragraph which you do not need to use. Write your
answers in the corresponding numbered box provided.
Blind to Change

How much of the world around you do you really see? You only take in tiny pieces of
information at a time and that can have unnerving consequences, says Laura Spinney.

Imagine you’re walking across a college campus when an unknown man asks you for
directions. While you’re talking, two men pass between you carrying a door. After an
irritating minute of interruption you carry on describing the route. When you’ve finished
you are informed that you’ve just taken part in a psychology experiment, and asked if you
noticed any changes after the two men passed with the door. “No”, you reply uneasily.
The unknown man then explains that the man who approached you initially walked off
behind the door, leaving this man in his place. You are stunned; the two men are dressed
differently and have different voices and haircuts.

24

Rather than logging every detail of the visual scene, we are actually highly selective
about what we take in. Our impression of seeing everything is just that – an impression.
In fact we extract a few details and rely on memory, or perhaps even our imagination, for
the rest.

25
FANPAGE TÀI LIỆU TIẾNG ANH NÂNG CAO

Yet in 1991, the controversial claim was made that our brains hold only a few salient
details about the world - and that this is the reason we are able to function at all. We
don’t store elaborate pictures in short-term memory, because it isn’t necessary and would
take up valuable computing power.

\ 26

Just a year later, at a conference on perception in Vancouver, it was reported that people
shown computer-generated pictures of natural scenes were blind to changes that were
made during an eye movement. In a typical laboratory demonstration of this you might be
shown a picture on a computer screen of, say, a couple dining on a terrace.

27

It’s an unnerving experience. But to some extent, such “change blindness” is artificial
because the change is masked in some way. In real life, there tends to be a visible
movement that signals the change. But not always. For instance, we have all had the
experience of not noticing a traffic signal change because we had briefly looked away.

28

For instance, an experiment was done at Harvard in which people were shown a
videotape of a basketball game and asked to count the passes made by one or other team.
After about 45 seconds a man dressed in a gorilla suit walked slowly across the scene,
passing between the players. Although he was visible for five seconds, an amazing 40 per
cent of the viewers failed to notice him.

29

Such lapses raise important questions about vision. For instance, how can we reconcile
these gross lapses with our subjective experience of having continuous access to a rich
visual scene? One researcher has actually shown that imagining a scene activates parts of
the visual cortex in the same way as seeing it. He says that this supports the idea that we
take in just what information we consider important at the time, and fill in the gaps where
FANPAGE TÀI LIỆU TIẾNG ANH NÂNG CAO

the details are less important. The illusion that we see “everything” is partly a result of
filling in the gaps using memory. Such memories can be created based on beliefs and
expectations.

30

This particular idea has not been generally accepted. Yet most researchers in the field do
agree that of all the myriad visual details of any scene that we could record, we take only
what is relevant to us at the time. This leads us to the uncomfortable realization that, for
all our subjective experience of a rich visual world, it may, in fact, be impossible to tell
what is real and what is imagined.

A. Now imagine that the task absorbing their attention had been driving a car, and the
distraction had been a pedestrian crossing their path. According to some estimates, nearly
half of all motor-vehicle accidents in the US can be attributed to driver error, including
momentary loss of attention. It is more than just academic interest that has made both
forms of cognitive error hot research topics.

B. The image would disappear, to be replaced for a fraction of a second by a blank


screen, before reappearing significantly altered - by the raising of a railing in the
background, perhaps. Many people search the screen for up to a minute before they see
the change. A few never spot it.

C. In contrast, other researchers argue that we can get the impression of visual richness
without holding any of that richness in our heads. For instance, the “grand illusion”
theory argues that we held no picture of the visual world in our brains at all. Instead, we
refer back to the external visual world as different aspects become important. The illusion
arises from the fact that as soon as you ask yourself “Am I seeing this or that?” you turn
your attention to it and see it.

D. It sounds impossible, but when this test was carried out, a full 50 per cent of those
who took part failed to notice the substitution. The subjects had succumbed to what is
called change blindness. Taken with a glut of recent experimental results, this
phenomenon suggests we see far less than we think we do.

E. The relationships between attention, awareness and vision have yet to be clarified.
Because we have a less than complete picture of the world at any one time, there is the
potential for distortion and error. How that complete picture could be objectively
established is controversial, but there is one obvious way forward.
FANPAGE TÀI LIỆU TIẾNG ANH NÂNG CAO

F. This flies in the face of what vision researchers have long believed: that seeing really
means making pictures in the brain. According to this theory, by building detailed
internal representations of the world, and comparing them over time, we would be able to
pick out anything that changed.

G. And there’s a related phenomenon called inattentional blindness, that doesn’t need any
experimental visual trick at all: if you are not paying attention to some feature of a scene,
you won’t see it.

H. Rather, we log what has changed and assume the rest has stayed the same. Of course :
this is bound to mean that we miss a few details. Experimenters had already shown that
we may ignore items in the visual field if they appear not to be significant - a repeated
word or line on a page of text for instance. But nobody realized quite how little we really
do “see”.

Your answers
24. 25. 26. 27. 28. 29. 30.

Part 4. The passage below consists of five sections marked A-E. For questions 31-40,
read the passage and do the task that follows. Write your answers (A-E) in the
corresponding numbered boxes provided.

Notorious Art 'Collector' Goes Free


Last week, Brian Barnes was acquitted of stealing a valuable ancient artefact. We
asked some of the key players about the trial.

A. Amelia Ashford (journalist)

I've been covering this case since the night of the robbery. A source from the museum
contacted me and told me that a priceless and irreplaceable ancient Egyptian gold
statuette had been stolen. This was not an opportunistic crime. The perpetrator knew in
advance exactly where the guards would be and had a meticulously crafted scheme for
bypassing the museum's security system.

The police investigation soon focused on Mr. Barnes, an art dealer with a shady past.
They had him under surveillance and an undercover officer, posing as a rich art collector,
was negotiating to buy the statuette. Mr. Barnes must have been tipped off, or realised
what was going on, and tried to flee the country. He was arrested at the airport, but this
FANPAGE TÀI LIỆU TIẾNG ANH NÂNG CAO

abrupt end to the police operation meant that, when the case came to court, they didn't
have enough evidence to secure a conviction.

B. Brian Barnes (the defendant)

Justice was done. It's as simple as that. The jury delivered the only possible verdict. To be
honest, I take my hat off to whoever stole that statuette. It was an audacious crime, and
the thieves clearly baffled the police, but I had nothing to do with it. I'm just an honest
businessman, and when the robbery took place, I was at the theatre. Surely, if the police
were unable to find anyone who could corroborate that, that's their fault, not mine. Now, I
just want to put this whole thing behind me. I'm going to take a well-earned holiday, and
then I'm looking forward to getting back to work.

C. Cristina Calviano (prosecution lawyer)

Obviously I'm very disappointed. I felt that we had a solid case against Mr. Barnes, and
enough persuasive arguments to get a conviction. The defendant clearly had the financial
means to mount an extravagant defence, and he took full advantage of that. What really
hurt us, though, was the judge's ruling that some of our key evidence could not be
introduced in court. The jury never heard that Mr. Barnes practically admitted stealing
the statuette to an undercover police officer or that we found a partial fingerprint at the
scene of the crime that we believe is Mr. Barnes'. We had to rely on a witness who saw
the defendant with an item resembling the statuette on the night in question, and sadly
that wasn't enough to sway the jury in our favour. Unfortunately, this means that the
statuette is now unlikely to ever be recovered.

D. Daniel Dawson (defence barrister)

To begin with, I don't think it befits someone in Ms. Calviano's position to criticise in
public the way the court handled the case. There were good legal reasons why she wasn't
allowed to present the evidence that she mentioned. It's also disingenuous of herto claim
that, had this evidence been included, it would have altered the outcome of the trial.
Take, for example, the partial fingerprint that the police found (which, by the way, was
the only tangible evidence that the prosecution had). The experts disagreed about it, and
even the police's forensic scientist admitted that she wasn't able to state with 100%
certainty that it was my client's print. Quite frankly, this is a case that should never have
come to court.

E. Ed Ellis (police detective)


FANPAGE TÀI LIỆU TIẾNG ANH NÂNG CAO

Of course it's a shame, but that's the way it goes. You win some and you lose some. We'll
have to reopen our file on the case now but, after so much time and with no other
suspects, I doubt that my superiors will commit too many resources to it. We think that
Barnes managed to pass the statuette off to an accomplice before he was arrested, but
after that the trail goes cold.

The only consolation is that Mr. Barnes doesn't seem like the sort of person who will be
able to keep a low profile. He's on our radar now, and I have a hunch that this won't be
his last brush with the law.

Which person (A—E) does the following?

31. states that the physical evidence in the case was inconclusive

32. believes that reinvestigating the case won’t be the police’s priorities

33. suggests Mr. Barnes may have been warned about the police investigation

34. suspects Mr. Barnes will be in trouble with the police again

35. mentions Mr. Barnes' affluence as his assisting factor

36. expresses admiration for the thieves

37. says that the robbery had been carefully planned

38. complains about a decision made by a court official

39. mentions that Mr. Barnes' alibi could not be confirmed

40. accuses somebody of acting in an unprofessional manner

Your answers
31. 32. 33. 34. 35.

36. 37. 38. 39. 40.

Key and explanations:


Part 1.
1. i
2. v
3. iii
FANPAGE TÀI LIỆU TIẾNG ANH NÂNG CAO

4. vii
5. vi
6. ix
7. F
And by defeating Austria and France in quick succession, he also created a power
vacuum on mainland Europe, which he was determined to fulfill himself.
8. T
He drafted a new German constitution to suit his own purposes and, despite maintaining a
veneer of democracy, the German parliament was effectively powerless to oppose him.
9. T
However, his plans backfired and Bismarck was forced to make a political U-turn.
Though here again, he somehow managed to save face. The damage to his reputation was
limited and indeed by the late 1870s, he had even managed to win over the church
whose support he now needed.
10. F
Bismarck once again failed to achieve his objective; though, to his credit, he held on to
power.
11. F
Having used war to unite Germany and make her great, Bismarck now believed that his
ambitions were best served by peace.
12. NG
13. T
His political and diplomatic juggling, therefore, simply cannot be considered a total
success.
Part 2.
14. C
They remember things just as effortlessly as most people forget them
15. C
He, then, is a living proof that the average Joe with a gift for forgetfulness can
reinvent himself in the area
FANPAGE TÀI LIỆU TIẾNG ANH NÂNG CAO

16. D
New research suggests that it is possible to tangibly improve your memory in a
relatively short space of time by devoting roughly half an hour of your every day to the
process. It is necessary to learn and employ memorizing strategies such as the Memory
Palace technique Konrad uses, though, to yield such results; otherwise, you might be as
well be doing something else.
17. A
In one recent study, for example, participants spent one month training their memories in
the aforementioned technique for 30 minutes every day, which more than doubled their
ability to remember list of information after just 40 days. More impressively, recall
performance remained high whether or not training continued at the end of one month,
which suggests after rewiring of the brain can be permanent.
18. D
The methods of loci, which has already been referred to here as the Memory Palace
method, is the most prevalent one adopted as revealed by a recent study of 35
memory champions
19. B
The conclusion, therefore, was that memory is not necessarily an innately bestowed gift
you either are possessed of or not. Most of us have the potential to hone and expand
our memories very meaningfully indeed.
20. C
Mnemonic = helping you to remember something
21. A
22. D
23. C
Most of us have the potential to hone and expand our memories very meaningfully
indeed.
Part 3.
24. D

When you’ve finished you are informed that you’ve just taken part in a psychology
experiment, and asked if you noticed any changes after the two men passed with the
FANPAGE TÀI LIỆU TIẾNG ANH NÂNG CAO

door… It sounds impossible, but when this test was carried out, a full 50 per cent of those
who took part failed to notice the substitution.

25. F

Rather than logging every detail of the visual scene, we are actually highly selective
about what we take in… This flies in the face of what vision researchers have long
believed: that seeing really means making pictures in the brain.

26. H

We don’t store elaborate pictures in short-term memory, because it isn’t necessary


and would take up valuable computing power… Rather, we log what has changed and
assume the rest has stayed the same.

27. B

In a typical laboratory demonstration of this you might be shown a picture on a


computer screen of, say, a couple dining on a terrace… The image would disappear, to
be replaced for a fraction of a second by a blank screen, before reappearing significantly
altered - by the raising of a railing in the background, perhaps.

28. G

And there’s a related phenomenon called inattentional blindness, that doesn’t need any
experimental visual trick at all: if you are not paying attention to some feature of a
scene, you won’t see it…After about 45 seconds a man dressed in a gorilla suit walked
slowly across the scene, passing between the players. Although he was visible for five
seconds, an amazing 40 per cent of the viewers failed to notice him

29. A

For instance, an experiment was done at Harvard in which people were shown a
videotape of a basketball game and asked to count the passes made by one or other
team… Now imagine that the task absorbing their attention had been driving a car,
and the distraction had been a pedestrian crossing their path.

30. C

Such memories can be created based on beliefs and expectations… In contrast, other
researchers argue that we can get the impression of visual richness without holding
any of that richness in our heads.
FANPAGE TÀI LIỆU TIẾNG ANH NÂNG CAO

Part 4.

31. D - Take, for example, the partial fingerprint that the police found (which, by the
way, was the only tangible evidence that the prosecution had). The experts disagreed
about it, and even the police's forensic scientist admitted that she wasn't able to state
with 100% certainty that it was my client's print.
22. E - We'll have to reopen our file on the case now but, after so much time and with
no other suspects, I doubt that my superiors will commit too many resources to it.
33. A - Mr. Barnes must have been tipped off, or realised what was going on, and tried
to flee the country.
34. E - I have a hunch that this won't be his last brush with the law.
35. C - The defendant clearly had the financial means to mount an extravagant defence,
and he took full advantage of that.
36. B - To be honest, I take my hat off to whoever stole that statuette. It was an
audacious crime, and the thieves clearly baffled the police, but I had nothing to do with it
37. A - This was not an opportunistic crime. The perpetrator knew in advance
exactly where the guards would be and had a meticulously crafted scheme for bypassing
the museum's security system.
38. C - What really hurt us, though, was the judge's ruling that some of our key
evidence could not be introduced in court. The jury never heard that Mr. Barnes
practically admitted stealing the statuette to an undercover police officer or that we found
a partial fingerprint at the scene of the crime that we believe is Mr. Barnes'.
39. B - I'm just an honest businessman, and when the robbery took place, I was at the
theatre. Surely, if the police were unable to find anyone who could corroborate that,
that's their fault, not mine.
40. D - To begin with, I don't think it befits someone in Ms. Calviano's position to
criticise in public the way the court handled the case.

TEST 13
Part 1. For questions 1-13, read the following passage and do the tasks that follow.

Eats, Shoots and Leaves


a book review
The title of Eats, Shoots and Leaves refers to a famously misplaced comma in a wildlife
manual that ended up suggesting a panda rather violently “eats, shoots and leaves”
FANPAGE TÀI LIỆU TIẾNG ANH NÂNG CAO

instead of eating shoots and leaves. The author of this book, journalist Lynne Truss, is
something akin to a militant linguist, dedicating this “zero tolerance” manifesto on
grammar to the striking Bolshevik printers of St. Petersburg who, in demanding the same
remuneration for punctuation as they received for letters, ended up setting in motion the
first Russian Revolution.
Some of the book involves humorous attacks on erroneous punctuation. There is the
confused Shakespearian thespian who inadvertently turns a frantic plea: “Go, get him
surgeons!” into the cheerful encouragement of “Go get him, surgeons!” Street and shop
signs have a ubiquitous presence. A bakery declares “FRESH DONUT’S SOLD HERE”
and a florist curiously announces that “Pansy’s here!” (Is she?). The shameless title of a
Hollywood film Two Weeks Notice is reeled in for criticism—“Would they similarly call
it One Weeks Notice?” Truss enquire—and sometimes, as in the case of signs promoting
“ANTIQUE’S” and “Potatoe’s”—one questions whether we are bearing witness to new
depths of grammar ignorance, or a postmodern caricature of atrocious punctuation.
Eats, Shoots and Leaves is not just a piece of comedy and ridicule, however, and Truss
has plenty to offer on the question of grammar usage. If you have ever wondered whether
it is acceptable to simply use an “em dash” in place of a comma—the verdict from Truss
is that you can. “The dash is less formal than the semicolon, which makes it more
attractive”, she suggests. “It enhances conversational tone; and it is capable of quite
subtle effects.” The author concludes, with characteristic wry condescension, that the em
dash’s popularity largely rests on people knowing it is almost impossible to use
incorrectly. Truss is a personal champion of the semicolon, a historically contentious
punctuation mark elsewhere maligned by novelist Kurt Vonnegut Jr., as a “transvestite
hermaphrodite representing absolutely nothing”. Coming to the semicolon’s defense,
Truss suggests that, while it can certainly be over-used—she refers to the dying words of
one 20th century writer: “I should have used fewer semicolons” —the semicolon can
perform the role of “a kind of Special Policeman in the event of comma fights.”
Truss has come under criticism on two broad points. The first argument criticises the
legitimacy of her authority as a punctuation autocrat. Louis Menand, writing in the New
Yorker, details Eats, Shoots and Leaves' numerous grammatical and punctuation sins: a
comma-free non-restrictive clause; a superfluous ellipsis; a misplaced apostrophe; a
misused parenthesis; two misused semicolons; an erroneous hyphen in the word “abuzz",
and so on. In fact, as Menand notes, half the semicolons in the Truss book are spuriously
deployed because they stem from the author’s open flouting of the rule that semicolons
must only connect two independent clauses. “Why would a person not just vague about
the rules but disinclined to follow them bother to produce a guide to punctuation?"
Menand inquires. Ultimately, he holds Truss accused of producing a book that pleases
those who “just need to vent'' and concludes that Eats, Shoots and Leaves is actually a
FANPAGE TÀI LIỆU TIẾNG ANH NÂNG CAO

tirade against the decline of language and print that disguises itself, thinly and poorly, as
some kind of a style manual.
Linguist David Chrystal has criticised what he describes as a “linguistic purism" coursing
through Truss’ book. Linguistic purism is the notion that one variety of language is
somehow more pure than others, with this sense of purity often based on an idealised
historical point in the language’s development, but sometimes simply in reference to an
abstract ideal. In The Fight for English. How Language Pundits Ate, Shot and Left,
Chrystal —a former colleague of Truss—condemns the no-holds-barred approach to
punctuation and grammar. “Zero tolerance does not allow for flexibility," he argues . “It
is prescriptivism taken to extremes. It suggests that language is in a state where all the
rules are established with 100 per cent certainty. The suggestion is false. We do not know
what all the rules of punctuation are. And no rule of punctuation is followed by all of the
people all of the time."
Other detractors of Truss’ “prescriptivism" are careful to disassociate needless purism
from robust and sensible criticism, an oppositional stance they call descriptivism. “Don’t
ever imagine," Geoffrey K. Pullum on the Language Log emphasises, “that I think all
honest attempts at using English are just as good as any others. [Bad] writing needs to be
fixed. But let’s make sure we fix the right things." In other words, we do not require a
dogmatic approach to clean up misused language. Charles Gaulke concurs, noting that his
opposition to “prescriptivism" does not require contending with the existence of
standards themselves, but questioning whether our standards should determine what
works, or whether what works should determine our standards.
Ultimately, it is unlikely the purists and pedagogues will ever make absolute peace with
those who see language as a fluid, creative process within which everyone has a role to
play. Both sides can learn to live in a sort of contentious harmony, however. Creativity
typically involves extending, adapting and critiquing the status quo, and revising and
reviving old traditions while constructing new ones. Rules must exist in order for this
process to take place, if only for them to be broken. On the flip side, rules have an
important role to play in guiding our language into forms that can be accessed by people
across all manner of differences, so it is vital to acknowledge the extent to which they can
be democratic, rather than merely autocratic in function. Nevertheless, all the regulations
in the world cannot stem the natural spring of language, which bursts through rivets and
snakes around the dams that linguistic authorities may try to put in place. We should
celebrate rather than curse these inevitable tensions.
Questions 1-6
In boxes 1-6, write:
True if the statement agrees with the information
False if the statement contradicts the information
FANPAGE TÀI LIỆU TIẾNG ANH NÂNG CAO

Not Given if there is no information on this

1. A well-known error lays the groundwork for Truss’s book.


2. Humour is utilised to aid the delivery of subliminal messages in the book.
3. Truss’s view of punctuation marks diverges from that of another writer.
4. According to an adversary, the book fails to serve its stated role.
5. Mutability in the rule of punctuation is accepted when zero tolerance is applied.
6. The development of language is fundamentally unstoppable.
Your answers

1. 2. 3. 4. 5. 6.

For questions 7-13, complete the following paragraph with words taken from the
passage.
The review gives readers an insight into a book on grammar and punctuation by Lynne
Truss which was dedicated to people who ignited the first Russian Revolution with their
request of (7) _________________________. Included in the book is criticism heaped
upon incorrect punctuation, with the repeated appearance of (8)
_________________________. Some cases involving such anomalous use can even be
considered a(n) (9) _________________________. Beside certain merits, the book has
also come under fire. One of the arguments against it describes Truss as (10)
_________________________ and highlights grammatical mistakes in her book. It has
also been castigated due to a sense of (11) ________________________ conveyed.
Furthermore, Truss’s perspective, described as prescriptivism, has met with disapproval
from people who oppose the use of (12) _________________________ to rectify
linguistic errors. This oppositional stance and prescriptivism are stated to be constantly at
loggerheads, but they can still co-exist in such a state of (13)
_________________________.
Part 2. For questions 14-23, read an extract from an article and choose the answer
A, B, C or D that fits best according to the text. Write your answers in the
corresponding numbered boxes provided.

Why do we keep pets?


FANPAGE TÀI LIỆU TIẾNG ANH NÂNG CAO

Why do we keep pets? After all, we are surely the only species on the planet inclined to
do so. Even our closet cousins, the great apes, do not similarly indulge. Indeed, when
footage of four chimps ‘playing’ with an antelope recently emerged, it caused much
debate on the subject momentarily until the exact circumstances of the ‘play date’ were
understood. The chimps at first played innocently with the calf, then gradually their play
became rougher and rougher and sadly, the calf did not come out the other end of the
game alive. Despite this, the chimps continued to amuse themselves with the corpse with
no less enthusiasm for a further half hour. Now, do you see?

Humans, contrarily invest heavily on an emotional and financial level in their domestic
animals, and this again begs the question: why exactly? After all, there is no conclusive
evidence to suggest that we benefit health-wise from, for instance, increased longevity by
keeping them and the effort of caring for pets definitely outweighs any payback there is.
As mentioned, pets are high-maintenance and require meals, healthcare and housing, and,
though they provide compainionship for some and practical services like guarding for
others, caring for them is extremely time-consuming. They often require walking,
grooming, worming and much else besides. Yet in spite of all this, for many, they are an
intrinsic part of the family, almost like having a child around

However, the comparisons end before they even begin in that respect. After all, there are
practical reasons for having children. From an evolutionary perspective, they keep the
genes alive. Not to mention the fact that they can return the favor of care years down the
line when you have grown old and they are in the prime of their existence. And whilst
they also demand an enormous amount of care and attention (admittedly, there are
similarities in some respects, then!), they can and are typically expected to contribute
actively to family affairs themselves as they get older. There is, in short, a clear return on
investment where kids are concerned.

[■]. On the other hand, in the past, perhaps a practical agenda for keeping pets could not
be ruled out so summarily. [■] For instance, pet-tending arguably may have demonstrated
desirable female qualities in the care-giver, making the carer more attractive to potential
FANPAGE TÀI LIỆU TIẾNG ANH NÂNG CAO

suitors. [■] Indeed, the urge to care also highlighted other admirable traits of a more
empathetic nature. [■]

Indeed, even today, some researchers argue the merits of pet ownership from a practical
point of view, suggesting that they might be helpful to humans lacking emotion and
social support. The science, at least in one respect, is clear and such people are definitely
more vulnerable to disease and infection. However, when it comes to the question of
whether pets can adequately fulfil the vacant role, substituting for human interaction, the
situation is a lot more cloudy and contradictory research exists, with opinions often toing
and froing.

Besides, many of us are more like the chimps than we’d perhaps care to admit. For
instance, the notion of pet keeping is a cultural one which does not exist across all
cultures. Some Kenyan indigenous tribes, for instance, do not have a word for pet. They
keep dogs, yet, but solely for their own protection. Indeed, in some cultures young-dog
meat itself is a staple of the traditional diet. Many Westerners claim this shocks them and
damn such behavior, but they will happily eat young sheep, with lamb being a favorite
dish for many. What’s the difference? You see, it’s all about perspective.

Some researchers suggest that pet keeping is a purely cultural phenomenon akin to
fashion, indeed. In other words, it is socially contagious and we keep pets because our
friends and neighbors do – and there is much evidence to support this assertion, in fact.
However, the problem with this theory is it’s a bit like that of the Big Bang, just as we
can’t account for where the particles which produced same to create the universe came
from, nor can we explain how or why the perpetually self-reinforcing trend of pet-
keeping began.

14. Why does the writer use the example of the chimps in paragraph one?
A. to give an instance about the contrast between toys and pets
B. to demonstrate similar behavioral patterns between chimps and humans
C. to exemplify how evolved chimps have become
FANPAGE TÀI LIỆU TIẾNG ANH NÂNG CAO

D. to prove that the subject remains a hotly debated manner


15. The writer implies that
A. keeping pets is not as altruistic as having children.
B. having children and pets are basically the same.
C. it would be more reasonable to have children than keep pets.
D. child bearing ought to take precedence over pet keeping.
16. According to the writer, the rationale behind pet ownership in the past may have been
A. people’s aspiration to appear sympathetic.
B. a yearning by members of one gender to show their compatibility as parents.
C. a craving for the generation of more weath and improvement of social status.
D. a need for an additonal care-giver in the family.
17. In paragraph five, the writer suggests that
A. it has yet to be confirmed whether or not pets have beneficial impacts on the health
conditions of socially isolated people.
B. most researchers concur in relation to the practical benefits of having pets today.
C. research indicates pet owners are more susceptible to disease and infection.
D. pets can substitute the roles of humans in a social context for lonely individuals.
18. What does the writer mean when he mentions ‘perspective’ in the sixth paragraph?
A. Dogs are blatantly abused in certain cultures.
B. Dogs can fulfil the dual role of protector and pet.
C. We cannot judge others by our own conventions.
D. Several countries exploit dogs solely for hunting purposes.
19. Why does the author refer to Big Bang Theory?
A. To suggest that theories fall in and out of popularity within the scientific
community.
B. To disprove the notion that pet keeping is fashion led.
C. To call for the implementation of a scientific approach to examine pet keeping
habits.
D. To insinuate that a certain theory about pet keeping habits is flawed.
FANPAGE TÀI LIỆU TIẾNG ANH NÂNG CAO

20. The word “antelope” in paragraph 1 mostly means


A. a favorite toy of chimps.
B. chimps’ companion.
C. a corpse that chimps play with.
D. a large mammal like a deer.
21. Look at the four squares [■] that indicate where the following sentence could be
added to the passage.
Alternatively, pet-keeping may have served as a symbol of status and wealth; after
all, before you can own one, you must sound finances.

Where would the sentence best fit?

A. First square.
B. Second square.
C. Third square.
D. Fourth square.
22. Why does the author mention Big Bang in the final paragraph?
A. To highlight the role of the Big Bang theory.
B. To identify a stark contrast between the origin of the universe and that of the pet
keeping habit.
C. To account for the origin of the pet keeping trend.
D. To illustrate the ambiguity in finding out the root of the pet keeping trend.
23. The phrase “perpetually self-reinforcing trend” in the final paragraph mostly means
A. a tendency that has reinforced pet keeping habits so far.
B. a trend that has existed for a long time.
C. a trend that serves to strengthen itself for a long time.
D. a tendency that aims to perpeptually keep pets.
Your answers

14. 15. 16. 17. 18.


FANPAGE TÀI LIỆU TIẾNG ANH NÂNG CAO

19. 20. 21. 22. 23.

Part 3. In the passage below, seven paragraphs have been removed. For questions
24-30, read the passage and choose from the paragraphs A-H the one which fits each
gap. There is ONE extra paragraph which you do not need to use. Write your
answers in the corresponding numbered box provided.

VALUES FOR A GODLESS AGE


When the Berlin Wall came tumbling down in 1989 so did the plaster cast which had kept
the idea of human rights in limbo. It was now free to evolve in response to the changing
conditions of the late twentieth century.

24.

Of course, in one sense, the quest for universal human rights standards after the Second
World War was an early attempt to communicate across national boundaries, albeit a
rather faltering endeavour, with its claims to universality challenged both in terms of
authorship and content. More recently, a loosening of the reins of the human rights
dialogue has ushered in wider debate.

25.

Perhaps the best known of these is Amnesty International, established in 1961. Before
Amnesty, there were very few organizations like it, yet now there are thousands operating
all over the world. Whether campaigning for the protection of the environment or third-
world debt relief, any such organization is engaged in the debate about fundamental
human rights. And it is no longer just a soft sideshow.

26.

The fact that strangers from different countries can communicate with each other through
the worldwide web is having a similar effect in dealing a blow to misinformation. During
one recent major human rights trial over sixty websites sprang up to cover the
proceedings, while sales of the government-controlled newspaper in that country
plummeted.
FANPAGE TÀI LIỆU TIẾNG ANH NÂNG CAO

27.

The effect of increased responsibility at this highest level has been to continually extend
the consideration of who is legally liable, directly or indirectly, under international
human rights law. In part, this is an acknowledgement that even individuals need to be
held responsible for flagrant breaches of others ‘rights, whether these are preventing
protesters from peacefully demonstrating or abusing the rights of children.

28..

It has been noted that paradoxically, in such circumstances, it may be in the interests of
human rights organizations to seek to reinforce the legitimacy and authority of the state,
within a regulated global framework.

29.

Part of the new trend in human rights thinking is therefore to include powerful private
bodies within its remit. The International Commission of Jurists has recently explored
ways in which international human rights standards could be directly applied to
transnational corporations.

30.

Whatever the way ahead, the lessons of the past must be learnt. Any world view or set of
values which is presented as self-evident is ultimately doomed to failure. The case for
human rights always needs to be made and remade. In a world where globalization too
often seems like a modernized version of old-fashioned cultural imperialism, it is
important to query the claim that human rights are universally accepted.

The missing paragraphs:


A. The problem is that the growth of globalization makes the protection of nation states a
pointless goal in certain circumstances. Transnational corporations with multiple
subsidiaries operating in a number of countries simultaneously wield significant
economic and political power and it is often extremely difficult for the state - both home
and host governments - to exercise effective legal control over them.

B. If the proliferation of pressure groups has raised the profile of the human rights debate,
satellite television has reinforced much of the content of their campaigns. The fact that
FANPAGE TÀI LIỆU TIẾNG ANH NÂNG CAO

from our armchairs we can all see live what is happening to others around the world has
had an enormous impact on the way the struggle for human rights is viewed. It would not
be remotely believable to plead ignorance nowadays, for 24-hour news coverage from the
world's hotspots reaches us all.

C. This is, after all, a uniquely propitious time, as the values and language of human
rights are becoming familiar to more and more people, who judge the merits or otherwise
of political and economic decisions increasingly in human rights terms. Arguments seem
fresh and appealing in many quarters where once they sounded weak and stale.

D. On a global scale, it is not strong states that are the problem here but weak ones, as
they fail to protect their citizens from private power - whether it is paramilitaries
committing murder and torture or transnational corporations spreading contamination and
pollution.

E. One of the most significant of these is what has come to be called 'globalization', the
collapsing of national boundaries in economic, political and cultural life. From the
expanding role of the world's financial markets and the spread of transnational
corporations to the revolution in communications and information technology, more and
more areas of people's lives are affected by regional, international or transnational
developments, whether they are aware of this or not.

F. Not only must states not infringe rights, and enforce those rights which fall within their
direct sphere (like providing a criminal justice system or holding fair elections), but they
also have 'positive obligations' to uphold rights enshrined in human rights treaties, even
when it is private parties which have violated them.

G. The results of its investigations were published in 1999 in a unique pamphlet on


Globalization, Human Rights and the Rule of Law. The issue to be faced is whether to
treat these and other corporations as 'large para-state entities to be held accountable under
the same sort of regime as states', or whether to look for different approaches to
accountability 'that are promulgated by consumer groups and the corporations
themselves.'

H. No longer the preserve of representatives of nation states meeting under the auspices
of the United Nations, a developing conversation is taking place on a global scale and
involving a growing cast of people - for an increasing range of pressure groups now
frame their aspirations in human rights terms.

Your answers
24. 25. 26. 27. 28. 29. 30.
FANPAGE TÀI LIỆU TIẾNG ANH NÂNG CAO

Part 4. The passage below consists of four sections marked A, B, C and D. For
questions 31-40, read the passage and do the task that follows. Write your answers
(A, B, C or D) in the corresponding numbered boxes provided.

A NIGHT TO REMEMBER
Our reporter spent a night camping out inside a zoo in Australia
A. Scanning the breakfast menu, I found myself swiftly losing my appetite: on offer
were beetle larvae, maggots and frozen mice. Fortunately, spending the night at Sydney’s
Taronga Zoo needn’t involve sampling the animals’ meals, though it does entail getting
extremely close to all manner of creatures. The Roar and Snore programme, curiously
little known in Australia let alone overseas, allows you to spend a night camping in the
heart of the zoo, long after the crowds of day trippers have dispersed. And if your
experience of zoos is creatures slumped motionless in their enclosures, or skulking in
their dens, then let me assure you that at dusk, the place comes alive and echoes with the
din of hundreds of creatures making their presence known. A ferryboat dropped me off at
the foot of the zoo, which sprawls across a steeply sloping swathe of bushland across the
bay from the city. Guides, Steve and Nikki, awaited with bright, enthusiastic smiles. Ten
of us had signed up and, in no time at all, we were busily erecting tents on a rectangle of
grass outside the zoo’s education centre. An adjacent area of gum trees was inhabited by
kangaroos, wallabies and an alarmingly inquisitive emu – our first animal encounter.
B. After a quick briefing, we set off into the gathering gloom. Steve unlocked a huge
metal gate that then clanged ominously behind us. We were in, though not without some
trepidation – what had we let ourselves in for? It was just us, a handful of security staff
and 2,000 furred and feathered inmates. A gibbon let out a haunting, liquid whoop and
wild possums frolicked noisily through the eucalyptus trees above us, their dark
silhouettes framed against a full moon. The skyscrapers of the city centre twinkled in the
distance as a pair of zebras scuffled in a cloud of dust; farther along, a giant Kodiak bear
lumbered menacingly around a large, rock-strewn enclosure. An awesome spectacle in
the gloom, but the buzz that evoked was soon surpassed when a pair of snow leopards
came within a few feet of us as they paced around the rocky ledges of their cage.
C. Not all the animals at Taronga are behind bars. Semi-tame African guinea fowl nest
in the gum trees and a water buffalo let out a surprisingly cow-like moo, whilst a group of
macaws screeched as we climbed back up to the education centre. It gets chilly after
dusk, but Steve had left nothing to chance and a warming barbecue was already sizzling
with sausages and chicken kebabs. Meanwhile, Nikki brought out some animals that are
used to being handled: first a diamond python called Little Spots and then a koala. Even
the Australians in the group were enthralled as Nikki showed us the pad of hardened skin
FANPAGE TÀI LIỆU TIẾNG ANH NÂNG CAO

that enables the species to sit for hours at a time in the crook of a tree. As she explained,
the gum trees they eat contain only four percent protein, so they have little energy to do
much else.
D. After a bitterly cold night under canvas, it was a relief to be woken at dawn for
breakfast. As the sun rose and bathed the zoo in a soft, orange light, we fed carrots to the
zoo’s small herd of giraffes. One of them, Hope, is blind, and sought out the carrots, with
her long, slobbering blue tongue. ‘If you touch it, you’ll notice it feels rough, like a
cat’s,’ said her keeper. As Hope wrapped hers around my outstretched finger, it felt more
like sandpaper. As the sun’s rays warmed our chilled bodies, we were taken on a behind-
the-scenes tour, down a corridor with doors leading to each enclosure, and notices
reminding keepers of the animals’ diet and welfare requirements. In a scrubbed kitchen,
the inmates’ food is prepared. Here another cheery keeper, Kristy, showed me the grey-
coloured gloop that is fed to small marsupials. ‘Have a taste,’ she said. I hesitated, then
dipped in my finger. It was delicious, like honey yoghurt. We were at the end of our visit
and the zoo was again admitting the public. Within an hour it seemed as though every
four-year-old in Sydney was careering around the place, letting out squeals of excitement
– a sharp contrast to the peaceful, privileged glimpse we’d been given.
In which section does the writer
31.......... mention a noise that made him feel slightly uneasy?
32.......... acknowledge the forethought of one of his hosts?
33.......... allude to the cleanliness of a section of the zoo?
34.......... suggest that most visitors don’t see the animals at their best?
35.......... imply that the activity he attended would benefit from wider publicity?
36.......... admit to an initial reluctance to take up a suggestion?
37.......... give us an impression of the scale and extent of the whole site?
38.......... describe a physical sensation that more than lived up to expectations?
39.......... report on one way in which the well-being of the animals is ensured?
40.......... mention a thrilling encounter with one particular species?
Your answers

31. 32. 33. 34. 35.

36. 37. 38. 39. 40.


FANPAGE TÀI LIỆU TIẾNG ANH NÂNG CAO

Key and explanations:


Part 1.
1. Not Given (NG). Việc một lỗi sai nổi tiếng đã đặt nền móng cho cuốn sách của Truss
là thông tin không được đề cập đến trong bài. Trong bài đọc có dữ kiện sau: “The title of
Eats, Shoots and Leaves refers to a famously misplaced comma in a wildlife manual that
ended up suggesting a panda rather violently “eats, shoots and leaves” instead of eating
shoots and leaves.” có thể gây nhầm lẫn rằng nội dung câu 1 là đúng. Tuy nhiên, tác giả
bài viết chỉ đề cập đến việc tiêu đề của cuốn sách ám chỉ đến lỗi nhầm dấu phẩy này,
không nói gì đến việc lỗi đó có liên quan như thế nào đến việc Truss viết sách và nội
dung cuốn sách.
FANPAGE TÀI LIỆU TIẾNG ANH NÂNG CAO

2. Not Given (NG). Mối liên quan, quan hệ hỗ trợ giữa nét hài hước (humour) và sự
truyền tải những thông điệp sâu xa (the delivery of subliminal messages) không được tác
giả bài viết đề cập. Dữ kiện sau đây có thể gây nhầm lẫn: “Eats, Shoots and Leaves is not
just a piece of comedy and ridicule, however, and Truss has plenty to offer on the
question of grammar usage.” Tuy nhiên, nội dung đoạn thông tin này chỉ nhắc đến hai
khía cạnh biệt lập: “comedy and ridicule” và “grammar usage” trong cuốn sách của Truss
mà thôi.
3. True (T). Dẫn chứng nằm ở dữ kiện sau: “Truss is a personal champion of the
semicolon, a historically contentious punctuation mark elsewhere maligned by novelist
Kurt Vonnegut Jr., as a “transvestite hermaphrodite representing absolutely nothing”. Có
thể thấy, Truss ủng hộ dấu chấm phẩy, trong khi tiểu thuyết gia Kurt Vonnegut Jr. lại
miêu tả dấu chấm phẩy chẳng biểu hiện được điều gì cả.
4. True (T). Dẫn chứng nằm ở dữ kiện sau: “Ultimately, he holds Truss accused of
producing a book that pleases those who “just need to vent'' and concludes that Eats,
Shoots and Leaves is actually a tirade against the decline of language and print that
disguises itself, thinly and poorly, as some kind of a style manual.” Có thể thấy, theo như
người phản đối thì cuốn sách của Truss đã không thực hiện đúng vai trò của nó.
5. False (F). Dẫn chứng nằm ở dữ kiện sau: “Zero tolerance does not allow for
flexibility," he argues. “It is prescriptivism taken to extremes. It suggests that language is
in a state where all the rules are established with 100 per cent certainty. The suggestion is
false. We do not know what all the rules of punctuation are. And no rule of punctuation is
followed by all of the people all of the time."
6. True (T). Dẫn chứng nằm ở dữ kiện sau: “Nevertheless, all the regulations in the world
cannot stem the natural spring of language, which bursts through rivets and snakes
around the dams that linguistic authorities may try to put in place.” Có thể thấy, sự phát
triển của ngôn ngữ vẫn diễn ra (cannot stem the natural spring of language), dù có bao
nhiêu cản trở (dams).
7. remuneration for punctuation. Có thể thấy thông tin cho câu 7 ở câu cuối đoạn 1: “The
author of this book, journalist Lynne Truss, is something akin to a militant linguist,
dedicating this “zero tolerance” manifesto on grammar to the striking Bolshevik printers
of St. Petersburg who, in demanding the same remuneration for punctuation as they
received for letters, ended up setting in motion the first Russian Revolution.”
8. street and shop signs. Có thể thấy thông tin cho câu 8 ở đầu đoạn 2: “Street and shop
signs have a ubiquitous presence.”
9. postmodern caricature of atrocious punctuation. Có thể thấy thông tin cho câu 9 ở cuối
đoạn 2: “and sometimes, as in the case of signs promoting “ANTIQUE’S” and
FANPAGE TÀI LIỆU TIẾNG ANH NÂNG CAO

“Potatoe’s”—one questions whether we are bearing witness to new depths of grammar


ignorance, or a postmodern caricature of atrocious punctuation.”
10. a punctuation autocrat. Có thể thấy thông tin cho câu 10 ở đầu đoạn 4: “Truss has
come under criticism on two broad points. The first argument criticises the legitimacy of
her authority as a punctuation autocrat.”
11. linguistic purism. Có thể thấy thông tin cho câu 11 ở đầu đoạn 5: “Linguist David
Chrystal has criticised what he describes as a “linguistic purism" coursing through Truss’
book.”
12. a dogmatic approach. Thông tin cho câu 12 ở câu thứ hai và ba từ cuối lên đoạn 6:
“[Bad] writing needs to be fixed. But let’s make sure we fix the right things." In other
words, we do not require a dogmatic approach to clean up misused language.”
13. contentious harmony. Thông tin cho câu 13 ở đầu đoạn cuối cùng: “Ultimately, it is
unlikely the purists and pedagogues will ever make absolute peace with those who see
language as a fluid, creative process within which everyone has a role to play. Both sides
can learn to live in a sort of contentious harmony, however.”
Part 2.
14. A
Despite this, the chimps continued to amuse themselves with the corpse with no less
enthusiasm for a further half hour. Now, do you see?
15. C
However, the comparisons end before they even begin in that respect. After all, there
are practical reasons for having children. There is, in short, a clear return on
investment where kids are concerned.
16. A
Indeed, the urge to care also highlighted other admirable traits of a more
empathetic nature
17. A
the situation is a lot more cloudy and contradictory research exists, with opinions
often toing and froing.
18. C
FANPAGE TÀI LIỆU TIẾNG ANH NÂNG CAO

Many Westerners claim this shocks them and damn such behavior, but they will
happily eat young sheep, with lamb being a favorite dish for many. What’s the
difference? You see, it’s all about perspective.
19. D
The problem with this theory is it’s a bit like that of the Big Bang, just as we can’t
account for where the particles which produced same to create the universe came
from, nor can we explain how or why the perpetually self-reinforcing trend of pet-
keeping began.
20. D
21. D
22. D

However, the problem with this theory is it’s a bit like that of the Big Bang, just as we
can’t account for where the particles which produced same to create the universe
came from, nor can we explain how or why the perpetually self-reinforcing trend of
pet-keeping began.
23. C
Part 3.
24. E
It was now free to evolve in response to the changing conditions of the late twentieth
century…One of the most significant of these is what has come to be called
'globalization', the collapsing of national boundaries in economic, political and cultural
life.
25. H
No longer the preserve of representatives of nation states meeting under the auspices of
the United Nations, a developing conversation is taking place on a global scale and
involving a growing cast of people - for an increasing range of pressure groups now
frame their aspirations in human rights terms… Perhaps the best known of these is
Amnesty International, established in 1961.
26. B
FANPAGE TÀI LIỆU TIẾNG ANH NÂNG CAO

If the proliferation of pressure groups has raised the profile of the human rights debate,
satellite television has reinforced much of the content of their campaigns… The fact
that strangers from different countries can communicate with each other through the
worldwide web is having a similar effect in dealing a blow to misinformation.
27. F
Not only must states not infringe rights, and enforce those rights which fall within
their direct sphere (like providing a criminal justice system or holding fair elections),
but they also have 'positive obligations' to uphold rights enshrined in human rights
treaties, even when it is private parties which have violated them…The effect of
increased responsibility at this highest level has been to continually extend the
consideration of who is legally liable, directly or indirectly, under international human
rights law.
28. D
On a global scale, it is not strong states that are the problem here but weak ones, as they
fail to protect their citizens from private power - whether it is paramilitaries committing
murder and torture or transnational corporations spreading contamination and
pollution… It has been noted that paradoxically, in such circumstances, it may be in the
interests of human rights organizations to seek to reinforce the legitimacy and authority
of the state, within a regulated global framework.
29. A
It has been noted that paradoxically, in such circumstances, it may be in the interests of
human rights organizations to seek to reinforce the legitimacy and authority of the
state, within a regulated global framework… The problem is that the growth of
globalization makes the protection of nation states a pointless goal in certain
circumstances.
30. G
The International Commission of Jurists has recently explored ways in which
international human rights standards could be directly applied to transnational
corporations… The results of its investigations were published in 1999 in a unique
pamphlet on Globalization, Human Rights and the Rule of Law.
Part 4.
31. B
A gibbon let out a haunting, liquid whoop and wild possums frolicked noisily through
the eucalyptus trees above us, their dark silhouettes framed against a full moon.
FANPAGE TÀI LIỆU TIẾNG ANH NÂNG CAO

32. C
It gets chilly after dusk, but Steve had left nothing to chance and a warming barbecue
was already sizzling with sausages and chicken kebabs.
33. D
In a scrubbed kitchen, the inmates’ food is prepared.
34. A
And if your experience of zoos is creatures slumped motionless in their enclosures,
or skulking in their dens, then let me assure you that at dusk, the place comes alive and
echoes with the din of hundreds of creatures making their presence known.
35. A
The Roar and Snore programme, curiously little known in Australia let alone overseas,
allows you to spend a night camping in the heart of the zoo, long after the crowds of day
trippers have dispersed.
36. D
Here another cheery keeper, Kristy, showed me the grey-coloured gloop that is fed to
small marsupials. ‘Have a taste,’ she said. I hesitated, then dipped in my finger.
37. A
A ferryboat dropped me off at the foot of the zoo, which sprawls across a steeply
sloping swathe of bushland across the bay from the city.
38. D
‘If you touch it, you’ll notice it feels rough, like a cat’s,’ said her keeper. As Hope
wrapped hers around my outstretched finger, it felt more like sandpaper.
39. D
As the sun’s rays warmed our chilled bodies, we were taken on a behind-the-scenes tour,
down a corridor with doors leading to each enclosure, and notices reminding keepers of
the animals’ diet and welfare requirements.
40. B
An awesome spectacle in the gloom, but the buzz that evoked was soon surpassed when
a pair of snow leopards came within a few feet of us as they paced around the rocky
ledges of their cage.
FANPAGE TÀI LIỆU TIẾNG ANH NÂNG CAO

TEST 14
Part 1. For questions 1-13, read the following passage and do the tasks that follow.

Keep taking the tablets


The history of aspirin is a product of a rollercoaster ride through time, of accidental
discoveries, intuitive reasoning and intense corporate rivalry.
A. In the opening pages of Aspirin: The Remarkable Story of a Wonder
Drug, Diarmuid Jeffreys describes this little white pill as ‘one of the most amazing
creations in medical history, a drug so astonishingly versatile that it can relieve
headache, ease your aching limbs, lower your temperature and treat some of the
deadliest human diseases’.
B. Its properties have been known for thousands of years. Ancient Egyptian
physicians used extracts from the willow tree as an analgesic, or pain killer.
FANPAGE TÀI LIỆU TIẾNG ANH NÂNG CAO

Centuries later the Greek physician Hippocrates recommended the bark of the
willow tree as a remedy for the pains of childbirth and as a fever reducer. But it
wasn't until the eighteenth and nineteenth centuries that salicylates the chemical
found in the willow tree became the subject of serious scientific investigation. The
race was on to identify the active ingredient and to replicate it synthetically. At the
end of the nineteenth century a German company, Friedrich Bayer & Co.
succeeded in creating a relatively safe and very effective chemical
compound, acetylsalicylic acid, which was renamed aspirin.
C. The late nineteenth century was a fertile period for experimentation, partly
because of the hunger among scientists to answer some of the great scientific
questions, but also because those questions were within their means to answer.
One scientist in a laboratory with some chemicals and a test tube could make
significant breakthroughs whereas today, in order to map the human genome for
instance, one needs ‘an army of researchers, a bank of computers and millions and
millions of dollars’.
D. But an understanding of the nature of science and scientific inquiry is not enough
on its own to explain how society innovates. In the nineteenth century, scientific
advance was closely linked to the industrial revolution. This was a period when
people frequently had the means, motive and determination to take an idea and
turn it into reality. In the case of aspirin that happened piecemeal - a series of
minor, often unrelated advances, fertilised by the century’s broader economic,
medical and scientific developments, that led to one big final breakthrough.
E. The link between big money and pharmaceutical innovation is also a significant
one. Aspirin’s continued shelf life was ensured because for the first 70 years of its
life, huge amounts of money were put into promoting it as an ordinary everyday
analgesic. In the 1970s other analgesics, such as ibuprofen and paracetamol, were
entering the market, and the pharmaceutical companies then focused on
publicising these new drugs. But just at the same time, discoveries were made
regarding the beneficial role of aspirin in preventing heart attacks, strokes and
other afflictions. Had it not been for these findings, this pharmaceutical marvel
may well have disappeared.
F. So the relationship between big money and drugs is an odd one. Commercial
markets are necessary for developing new products and ensuring that they remain
around long enough for scientists to carry out research on them. But the
commercial markets are just as likely to kill off' certain products when something
more attractive comes along. In the case of aspirin, a potential ‘wonder drug* was
around for over 70 years without anybody investigating the way in which it
achieved its effects, because they were making more than enough money out of it
FANPAGE TÀI LIỆU TIẾNG ANH NÂNG CAO

as it was. If ibuprofen or paracetamol had entered the market just a decade earlier,
aspirin might then not be here today. It would be just another forgotten drug that
people hadn't bothered to explore.
G. None of the recent discoveries of aspirin's benefits were made by the big
pharmaceutical companies; they were made by scientists working in the public
sector. 'The reason for that is very simple and straightforward,' Jeffreys says in his
book. 'Drug companies will only pursue research that is going to deliver financial
benefits. There's no profit in aspirin any more. It is incredibly inexpensive with
tiny profit margins and it has no patent any more, so anyone can produce it.' In
fact, there's almost a disincentive for drug companies to further boost the drug, he
argues, as it could possibly put them out of business by stopping them from selling
their more expensive brands.
H. So what is the solution to a lack of commercial interest in further exploring the
therapeutic benefits of aspirin? More public money going into clinical trials,
says Jeffreys. ‘If I were the Department of Health. I would say “this is a very
inexpensive drug. There may be a lot of other things we could do with it." We
should put a lot more money into trying to find out.'
I. Jeffreys' book which not only tells the tale of a 'wonder drug' but also explores the
nature of innovation and the role of big business, public money and regulation
reminds us why such research is so important.

Questions 1-6
The Reading Passage has nine paragraphs A-I.
Choose the correct heading for each paragraph (A-F) from the list of headings
below.
Write the correct number (i-x) in boxes 1-6.
FANPAGE TÀI LIỆU TIẾNG ANH NÂNG CAO

List of headings:
i. The most powerful analgesic
ii. The profit potential of aspirin
iii. Saved from oblivion by drug companies
iv. Recognition of an important medicinal property
v. A double-edged sword
vi. An unstructured pattern of development
vii. Major pharmaceutical companies
viii. A wonder drug
ix. Roots of the scientific advancements in the 19th century
x. The discovery of new medical applications

1. Paragraph A
2. Paragraph B
3. Paragraph C
4. Paragraph D
5. Paragraph E
6. Paragraph F
Your answers

1. 2. 3. 4. 5. 6.

Questions 7-13
Do the following statements agree with the views of the writer in Reading Passage 3?
In boxes 7-13 write
YES if the statement agrees with the views of the writer

NO if the statement contradicts the views of the writer


FANPAGE TÀI LIỆU TIẾNG ANH NÂNG CAO

NOT GIVEN if it is impossible to say what the writer thinks about this
7. The discovery of aspirin’s full medicinal properties was a very unusual one.
8. The 19th century saw significant changes in the way in which scientific experiments
were carried out.
9. For nineteenth-century scientists, small-scale research was far from enough to make
important discoveries.
10. The creation of a market for aspirin as a painkiller was achieved through commercial
advertising campaigns.
11. In the 1970s sales of new analgesic drugs overtook sales of aspirin.
12. Between 1900 and 1970, there was little research into aspirin because commercial
companies thought it had been adequately exploited.
13. Jeffrey suggests that there should be state support for further research into the
possible applications of aspirin.
Your answers

7. 8. 9. 10. 11. 12. 13.

Part 2. For questions 14-23, read an extract from an article and choose the answer
A, B, C or D that fits best according to the text. Write your answers in the
corresponding numbered boxes provided.

Using video gaming in education


It has become conventional wisdom that spending too much time playing video games
has a detrimental effect on children’s studies and their social development. However,
some educationalists are now questioning this theory and are using video games as
effective educational tools thus bridging the gap between recreational and educational
activities.

Due to the sophisticated nature of today’s games, teachers are able to justify the inclusion
of video and online games for many pedagogical reasons. There may, for example, be
sociological, psychological, and ethical implications built into the gameplay. Harvey
Edwards, who teaches IT classes in London, was one such educator who decided to use
video games in his lessons. To do this, he chose Minecraft, an online game in which
players create and develop imaginary worlds. He was somewhat uneasy about attempting
FANPAGE TÀI LIỆU TIẾNG ANH NÂNG CAO

such an unconventional approach, not because of some students’ unfamiliarity with the
game but rather due to them not being able to make sense of what he was trying to do
with it. He worried that it might interfere with his learners’ focus, but he couldn’t have
been more surprised by the results.

Minecraft is an example of a ‘sandbox game’, in which gamers roam around and change
a virtual world at will. Instead of having to pass through numbered levels to reach certain
places, there’s full access from start to finish. The original version can be adapted to
control which characters and content are left in. Each student can then be allocated tasks
– such as house-building, locating items or problem-solving – which they must complete
within the game. Elements of more general skills can be subtly incorporated into the
lessons, such as online politeness and safety, teamwork and resolving differences.
Edwards feels that presenting such lessons in the context of a game students probably
already know and enjoy enables him to connect with them at greater depth, and in more
motivational ways.

Bolstered by his success, Edwards introduced his approach to another school nearby. He
recalls that the first couple of sessions didn’t live up to his expectations. Those who had
played Minecraft before were keen for others to adopt their own style of play.
Unsurprisingly, this assortment of styles and opinions as to how the game should proceed
were far from harmonious. However, the sessions rapidly transformed into something
more cohesive, with the learners driving the change. With minimal teacher input, they set
about choosing leaders and established several teams, each with its own clearly-defined
role. These teams, now party to clear common goals, willingly cooperated to ensure that
their newborn world flourished, even when faced with the toughest of challenges.

‘Human’ inhabitants in a Minecraft ‘society’ are very primitive and wander around the
imaginary world, waiting for guidance from players. [A] This dynamic bears a
resemblance to traditional education, an observation highlighted by Martina Williams,
one of the leaders of the group. [B] ‘Through the game, we were no longer passive
learners in the classroom, being told what and how to learn, but active participants in our
own society. [C] The leaders, meanwhile, had a vision for their virtual world as a whole,
encouraging everyone to play their part in achieving the group’s goals. [D] Through
creating their own characters and using these to build their own ‘world’, students will
have gained some experiential understanding of societal structure and how communities
work.

But not everyone is convinced by video games’ potential academic value. While many
progressive commentators cite extensive evidence to maintain that video games
FANPAGE TÀI LIỆU TIẾNG ANH NÂNG CAO

encourage collaboration and build problem-solving skills, more traditional factions


continue to insist they are a distraction that do not merit inclusion in any curriculum.
Even less evangelical cynics, who may grudgingly acknowledge games have some
educational benefit, assert that this is only the case in the hands of creative educators.
However, the accusation most often levelled at video games is that they detract from the
social aspect of the classroom, particularly taking part in discussions. Dr Helen Conway,
an educational researcher, argues that video games can be used to promote social
activities. ‘Students become animated talking about the game and how to improve their
gameplaying and problem-solving skills,’ she says. ‘I find it strange, this image that
many people have,’ Conway says. ‘Children are often totally detached from their peers
when undertaking more traditional activities, like reading books, but we never suggest
that books are harmful because they’re a solitary experience.

14. The first time Edwards used a game in his classes, he was

A. convinced that learners would realise why he wanted them to play it.

B. convinced that learners would see the reasons for playing it.

C. anxious that he had chosen the wrong one for learners to play.

D. sure that his reasons for getting learners to play it were valid.

15. The writer suggests that Minecraft is a good choice of educational game because

A. any number of learners can use it simultaneously.

B. teachers can remove any inappropriate material.

C. gamers can create educative tasks whilst playing it.

D. players can develop their skills in a step-by-step way.

16. Which of the following words in the fourth paragraph is used to convey a feeling of
approval?

A. keen B. harmonious C. driving D. newborn

17. In the fifth paragraph, the writer draws a comparison between a Minecraft ‘society’
and

A. relationships within the group as they played.

B. the way in which countries organise themselves.


FANPAGE TÀI LIỆU TIẾNG ANH NÂNG CAO

C. typical students in a school environment.

D. how leadership operates in different situations.

18. In the sixth paragraph, the writer feels that critics of video games in education

A. are unwilling to admit that using them in class has benefits.

B. make accurate observations about teachers who use them.

C. use flawed research to support their objections to using them.

D. acknowledge the drawbacks of more traditional teaching methods.

19. The words ‘this image’ in the sixth paragraph refer to

A. people who criticise gaming in education.

B. students discussing a game in a group.

C. a group of students reading individually.

D. a solitary player absorbed in a game.

20. Where does this sentence belong to in the fifth paragraph?

Each group member had ideas as to how their function should develop.

A. [A] B. [B] C. [C] D. [D]

21. The word ‘subtly’ in the third paragraph is closest in meaning to __________.

A. intricately B. ingeniously C. ingenuously D. haphazardly

22. The word ‘grudgingly’ in the sixth paragraph is closest in meaning to __________.

A. gleefully B. vivaciously C. genially D. reluctantly

23. Which of the following best describes the author's attitude towards the application of
gaming into education?

A. supportive B. neutral C. cynical D. satirical

Your answers

14. 15. 16. 17. 18.


FANPAGE TÀI LIỆU TIẾNG ANH NÂNG CAO

19. 20. 21. 22. 23.

Part 3. You are going to read a magazine article. Seven paragraphs have been
removed from the article. Choose from the paragraphs A - H the one which fits each
gap (24 - 30). There is one extra paragraph which you do not need to use.

Mobile Misgivings
It's getting hard to be anonymous. To do anything, you have to prove who you are. Want
to buy something or draw some cash? That's a wodge of credit cards to lug around, and a
plethora of four-digit PINs to remember. Even before stepping out of the front door,
you've got to find your driving licence or rail pass, perhaps even your passport.

24.

Inside every digital mobile phone is a SIM card. SIM stands for Subscriber Information
Module, and the chip embedded in the SIM card is what makes the mobile yours. For
now, the SIM just identifies you to the phone system, and maybe holds details of your
favourite phone numbers. In future it could identify you to everyone who needs to know
who you are and would enable you to carry out transactions which require a form of
identification.

25.

The Finnish government is looking at using SIM's in place of a national identity card -
and eventually a passport. Under this plan, the SIM wilt become a person's legal proof of
identity. And there's no reason why it couldn't unlock your health records, social security
details and other personal information. One click and a hospital would know exactly who
it's dealing with.

26.

People can loge or mislay their phones, and they are a tempting target for thieves, who
can easily dispose of them on the black market. That's bad enough when there's only a
FANPAGE TÀI LIỆU TIẾNG ANH NÂNG CAO

large phone bill at stake. When your phone becomes the key to your identity, secrets and
cash, you'll want so make sure it stays safely locked up, even if only the gadget itself falls
into the wrong Lands. "Having something that contains all this information would be
extremely rash," says Roger Needham, managing director of Microsoft's British research
laboratory in Cambridge. "People will simply find it unacceptable."

27.

The beauty of this system is that the identifier would act as one half of what's called a
public key encryption system. The identifier, kept safe inside the phone, acts as a key,
known to no one else. To read a message locked with this private key requires a second,
public key, which can be freely distributed.

28.

An increasing number of countries are passing laws to give private keys the same legal
force as signatures. This has unleashed a flood of encryption systems, and the problem
now is to get governments and companies to agree on a standard. "It needs to be simple,
secure and transparent," says Mica Nierninen, head of mobile commerce company More
Magic Software, in Helsinki. "We have the maths to show that it is secure. The only
problem now is making it global."

29.

The private key is protected by a PIN, and the card will shut itself off if wrong numbers
are keyed in three times. To switch it back on, the owner must take it to a police station
with another form of ID. If a card is stolen, the police will cancel it permanently. Either
way, information on the card stays safe.

30.

Pearson thinks consumers, too, will learn to trust a chip with their identity, not least
because it will make life so much easier. A private key will do away with hard-to-
remember log-in codes and passwords for websites, as well as all those credit cards and
FANPAGE TÀI LIỆU TIẾNG ANH NÂNG CAO

PINS. "People already give up their privacy quite happily just to get access to a website,"
he says. "As long as they get something out of it."

A. But for these dreams to become reality, there'll have to be a revolution in public
attitudes. People will have to let go of their apprehensions about e-commerce and learn to
trust their mobiles; "Cultivating that trust is a very difficult thing to do and takes it lot of
time," says Ian Pearson, resident futurologist at British Telecom.
B. Your credit history could be accessed by your bank manager with your agreement,
which would negate the need for you to visit the bank. The manager could communicate
with you through your mobile phone and either give you advice over the phone or invite
you to the bank for a face-to-face consultation. This has already been piloted in Britain
and has received a positive reception.
C. The solution, according to experts in the field, is to share precious information on
secure servers accessible via a WAP connection on the web. The SIM would only store a
personal identifier - a long string of digits that would unlock the servers and give access
to the information they hold. To use the identifier, the phone's owner would have to
punch in a PIN.
D. "Even in its embryonic form, FINEID gives people a secure way to access sensitive
information," says Vatka. "And when you get it in a mobile phone you're not even tied to
a terminal," he says. Many believe that identity theft will be inevitable no matter how
careful safeguards are. But since it is already taking place and this system is more secure,
businesses will probably be keen to adopt it.
E. In a few years, this plastic and paper baggage could be history. A single chip hidden in
your cellphone will be all you need - a little treasure that holds your complete identity.
But beware! Lose your phone, and your identity and money go with it. The big question
is whether people will be willing to trust so much to a sliver of silicon.
F. You might use this set-up to send a request to a bank using its public key to see the
details of your account, which it would decrypt using its private key. The bank would
then send you the requested information encrypted with your public key, which only your
private key could decrypt. Thus both messages would be secure.
G. To pay for a meal, say, you will use the phone to transfer money through the phone
network to a restaurant's computer. There will be no payment slip to sign because your
SIM will do it for you. Likewise, when you board a plane you won't have to wait in line
for a boarding pass and seat number.
H. "The Finnish government has taken the initiative with a national standard that
companies can use free of charge," says Vesa Vatka of the Finnish Population Register
FANPAGE TÀI LIỆU TIẾNG ANH NÂNG CAO

Centre in Helsinki. "At the moment this system - called FINEID - uses a smart card and a
card reader attached to a computer, but the plan is to integrate to a SIM," says Vatka.
Your answers

24. 25. 26. 27. 28. 29. 30.

Part 4. The passage below consists of four paragraphs marked A-D. Read the
passage and do the task that follows. Write your answers (A-D) in the corresponding
numbered boxes provided.

Studying the law


A. Ordinary people regularly encounter law in a variety of circumstances. Freely-
negotiated commercial contracts may bind them to act in particular ways. By becoming
members of a sports club or a trade union they agree to comply with a set of rules.
Sometimes these forms of law will use the courts to enforce their arrangements. In other
cases privately-instituted adjudication bodies are established, a third party being
appointed to decide whether an agreement or rule has been broken or not. These kinds of
arrangements may seem very different from the normal idea of law, especially if law is
thought of mainly in terms of the criminal law. However, it is possible to see law simply
as a way of regulating our actions, of deciding what can be done and what cannot be
done. Most laws are not about something spectacular but, rather, about the details of
ordinary life. Every time a purchase is made, a contract is made. Both parties make
promises about what they will do; one to hand over the goods, one to pay the price. In
this and other ways, everybody is involved in law every day of their lives.

B. Legal rules can be divided up in many different ways. The rules show differences in
purpose, in origin and form, in the consequences when they are breached, and in matters
of procedure, remedies and enforcement. One of the most fundamental divisions in law is
the division between criminal and civil law. Newcomers to the study of law tend to
assume that criminal law occupies the bulk of a lawyer's caseload and of a law student's
studies. This is an interesting by-product of the portrayal of the legal system by the
media. Criminal law weighs very lightly in terms of volume when measured against non-
criminal (that is, civil) law. There are more rules of civil law than there are of criminal
law; more court cases involve breach of the civil law than that of the criminal law.

C. The term 'national law' is used to mean the internal legal rules of a particular country,
in contrast to international law which deals with the external relationships of a state with
FANPAGE TÀI LIỆU TIẾNG ANH NÂNG CAO

other states. There is no world government or legislature issuing and enforcing laws to
which all nations are subject. The international legal order has no single governing body
and operates by agreement between states. This means that the creation, interpretation
and enforcement of international law lie primarily in the hands of states themselves. Its
scope and effectiveness depend on the sense of mutual benefit and obligation involved in
adhering to the rules. Disputes about the scope and interpretation of international law are
rarely resolved by the use of international courts or binding arbitration procedures of an
international organisation. This is because submission to an international court or similar
process is entirely voluntary and few states are likely to agree to this if there is a serious
risk of losing their case or where important political or national interests are at stake.

D. One source of detailed information about the legal system is statistical analyses.
Information about the number of cases handled by a court shows in specific terms what a
court's workload is. Changes in these from year to year may indicate some effects of
changes in the law and practice. Statistical tests can establish that there is a relationship, a
correlation, between different things. For example, the length of a sentence for theft may
correlate with the value of the items stolen or the experience of the judge who heard the
case. This means that the sentence will be longer if, for example, more items are stolen or
the judge is more experienced. A correlation can provide evidence for a theory. Such
confirmation is important; without it we have little to establish the impact the law has,
being forced to rely on individual instances of its application and having to assume that
these have general truth. Empirical study of the operation of law may reveal areas of
improvement. It can also confirm that, measured by particular standards, the courts are
working well.

In which extract are the following mentioned?

31. the relative frequency of certain types of legal cases

32. input by those who are not directly involved in a dispute

33. how common transactions assume certain guarantees

34. ascertaining the effectiveness of a legal system

35. determining the consequences of altering the legal system

36. the influence of popular depictions of the law

37. a reluctance to submit to formal legal processes

38. how a decentralised legal system depends on a feeling of reciprocity


FANPAGE TÀI LIỆU TIẾNG ANH NÂNG CAO

39. the lack of drama in the way the law operates

40. the absence of a certain type of legal institution

Your answers

31. 32. 33. 34. 35.

36. 37. 38. 39. 40.

Key and explanations:


Part 1.
1. viii
In the opening pages of Aspirin: The Remarkable Story of a Wonder Drug, Diarmuid
Jeffreys describes this little white pill as ‘one of the most amazing creations in
FANPAGE TÀI LIỆU TIẾNG ANH NÂNG CAO

medical history, a drug so astonishingly versatile that it can relieve headache, ease your
aching limbs, lower your temperature and treat some of the deadliest human diseases’.
In Paragraph A, the writer describes Aspirin as an “amazing creation” and list its various
benefits. Therefore, the answer is viii.
2. iv
Its properties have been known for thousands of years. Ancient Egyptian physicians used
extracts from the willow tree as an analgesic, or pain killer. Centuries later the Greek
physician Hippocrates recommended the bark of the willow tree as a remedy for the pains
of childbirth and as a fever reducer. But it wasn't until the eighteenth and nineteenth
centuries that salicylates, the chemical found in the willow tree, became the subject
of serious scientific investigation. The race was on to identify the active ingredient and
to replicate it synthetically. At the end of the nineteenth century a German company,
Friedrich Bayer & Co. succeeded in creating a relatively safe and very effective chemical
compound, acetylsalicylic acid, which was renamed aspirin.
In paragraph B, the writer mentions the discovery of salicylates – a chemical in willow
trees with remarkable healing property. Therefore, the answer is iv.
3. ix
The late nineteenth century was a fertile period for experimentation, partly because of
the hunger among scientists to answer some of the great scientific questions, but also
because those questions were within their means to answer. One scientist in a
laboratory with some chemicals and a test tube could make significant breakthroughs
whereas today, in order to map the human genome for instance, one needs ‘an army of
researchers, a bank of computers and millions and millions of dollars’.
The author attributes scientific advancements partly to the thirst for knowledge among
scientists. Another reason given is that in the 19th century, small-scale research was
enough to make important discoveries. Therefore, the answer is ix.
4. vi
But an understanding of the nature of science and scientific inquiry is not enough on its
own to explain how society innovates. In the nineteenth century, scientific advance was
closely linked to the industrial revolution. This was a period when people frequently had
the means, motive and determination to take an idea and turn it into reality. In the case of
aspirin that happened piecemeal - a series of minor, often unrelated advances,
fertilised by the century’s broader economic, medical and scientific developments,
that led to one big final breakthrough.
FANPAGE TÀI LIỆU TIẾNG ANH NÂNG CAO

The author mentions that the development of aspirin happened piecemeal-a series of
minor, often unrelated advances, which is a synonym of “an unstructured pattern of
development”. Therefore, the answer is vi.
5. x
The link between big money and pharmaceutical innovation is also a significant one.
Aspirin’s continued shelf life was ensured because for the first 70 years of its life, huge
amounts of money were put into promoting it as an ordinary everyday analgesic. In the
1970s other analgesics, such as ibuprofen and paracetamol, were entering the market, and
the pharmaceutical companies then focused on publicising these new drugs. But just at
the same time, discoveries were made regarding the beneficial role of aspirin in
preventing heart attacks, strokes and other afflictions. Had it not been for these
findings, this pharmaceutical marvel may well have disappeared.
The author mentions new applications of aspirin such as preventing heart attacks, strokes
and other afflictions. Therefore, the answer is x. The heading iii is tempting but it is
wrong because this paragraph doesn’t mention that drug companies made the new
discoveries.
6. v
So the relationship between big money and drugs is an odd one. Commercial markets
are necessary for developing new products and ensuring that they remain around
long enough for scientists to carry out research on them. But the commercial
markets are just as likely to kill off' certain products when something more
attractive comes along. In the case of aspirin, a potential ‘wonder drug' was around for
over 70 years without anybody investigating the way in which it achieved its effects,
because they were making more than enough money out of it as it was. If ibuprofen or
paracetamol had entered the market just a decade earlier, aspirin might then not be here
today. It would be just another forgotten drug that people hadn't bothered to explore.
The author explains that commercial companies have both good and bad effects on drugs:
necessary for developing new products, and killing off certain products when something
more attractive comes along. Therefore, the answer is v.
7. NOT GIVEN

The author never comments on the nature of aspirin discovery.

8. NOT GIVEN

The author compares the difference between how researches were carried out in the late
19th century and today but he/she doesn’t mention any change that happened during the
19th century.
FANPAGE TÀI LIỆU TIẾNG ANH NÂNG CAO

9. NO

One scientist in a laboratory with some chemicals and a test tube could make significant
breakthroughs whereas today, in order to map the human genome for instance, one needs
‘an army of researchers, a bank of computers and millions and millions of dollars’.

10. YES

Aspirin’s continued shelf life was ensured because for the first 70 years of its life, huge
amounts of money were put into promoting it as an ordinary everyday analgesic.

Aspirin financial success was mostly attributed to extensive promotions from drug
companies.

11. NOT GIVEN

In the 1970s sales of new analgesic drugs overtook sales of aspirin.

The author mentions that aspirin’s position was swayed by the advent of new drugs but
doesn’t mention sales.

12. YES

In the case aspirin, a potential ‘wonder drug' was around for over 70 years without
anybody investigating the way in which it achieved its effects, because they were making
more than enough money out of it as it was.

The author mentions the reason the way aspirin achieved its effects was not investigated.
He/she indicates that pharmaceutical companies didn’t investigate the way aspirin
worked because they were making more than enough money out of it, which means they
think that the drug has been already exploited to its full potential.

13. YES

More public money going into clinical trials, says Jeffreys. ‘If I were the Department of
Health. I would say “this is a very inexpensive drug. There may be a lot of other things
we could do with it." We should put a lot more money into trying to find out

Part 2.
14. D

However, some educationalists are now questioning this theory and are using video
games as effective educational tools thus bridging the gap between recreational and
educational activities. There may, for example, be sociological, psychological, and
FANPAGE TÀI LIỆU TIẾNG ANH NÂNG CAO

ethical implications built into the gameplay. Harvey Edwards, who teaches IT classes in
London, was one such educator who decided to use video games in his lessons.

15. B

The original version can be adapted to control which characters and content are left in.

16. C

17. C

This dynamic bears a resemblance to traditional education, an observation highlighted by


Martina Williams, one of the leaders of the group.

18. A

While many progressive commentators cite extensive evidence to maintain that video
games encourage collaboration and build problem-solving skills, more traditional factions
continue to insist they are a distraction that do not merit inclusion in any curriculum.
Even less evangelical cynics, who may grudgingly acknowledge games have some
educational benefit, assert that this is only the case in the hands of creative educator.

19. D

I find it strange, this image that many people have.

Conway ngạc nhiên vì mọi người nghĩ là game là một trò chơi đơn độc, trong khi người
chơi game có thể thảo luận với nhau.

20. C

21. B

22. D

23. A

Part 3.
24. E
That's a wodge of credit cards to lug around, and a plethora of four-digit PINs to
remember… In a few years, this plastic and paper baggage could be history.
25. G
FANPAGE TÀI LIỆU TIẾNG ANH NÂNG CAO

In future it could identify you to everyone who needs to know who you are and would
enable you to carry out transactions which require a form of identification…. To pay
for a meal, say, you will use the phone to transfer money through the phone network to
a restaurant's computer. There will be no payment slip to sign because your SIM will
do it for you. Likewise, when you board a plane you won't have to wait in line for a
boarding pass and seat number.
26. A
And there's no reason why it couldn't unlock your health records, social security
details and other personal information. One click and a hospital would know exactly
who it's dealing with… But for these dreams to become reality, there'll have to be a
revolution in public attitudes.
27. C
People can loge or mislay their phones, and they are a tempting target for thieves,
who can easily dispose of them on the black market… The solution, according to
experts in the field, is to share precious information on secure servers accessible via a
WAP connection on the web.
28. F
The beauty of this system is that the identifier would act as one half of what's called a
public key encryption system. The identifier, kept safe inside the phone, acts as a key,
known to no one else. To read a message locked with this private key requires a
second, public key, which can be freely distributed… You might use this set-up to
send a request to a bank using its public key to see the details of your account, which it
would decrypt using its private key.
29. H
This has unleashed a flood of encryption systems, and the problem now is to get
governments and companies to agree on a standard…"The Finnish government has
taken the initiative with a national standard that companies can use free of charge," says
Vesa Vatka of the Finnish Population Register Centre in Helsinki.
30. D
But since it is already taking place and this system is more secure, businesses will
probably be keen to adopt it… Pearson thinks consumers, too, will learn to trust a chip
with their identity, not least because it will make life so much easier.
Part 4.
31. B
FANPAGE TÀI LIỆU TIẾNG ANH NÂNG CAO

Criminal law weighs very lightly in terms of volume when measured against non-
criminal (that is, civil) law. There are more rules of civil law than there are of criminal
law; more court cases involve breach of the civil law than that of the criminal law.

32. A

In other cases privately-instituted adjudication bodies are established, a third party


being appointed to decide whether an agreement or rule has been broken or not.

33. A

Every time a purchase is made, a contract is made. Both parties make promises about
what they will do; one to hand over the goods, one to pay the price.

34. D

Such confirmation is important; without it we have little to establish the impact the law
has, being forced to rely on individual instances of its application and having to assume
that these have general truth. Empirical study of the operation of law may reveal areas of
improvement. It can also confirm that, measured by particular standards, the courts
are working well.

35. D

Changes in these from year to year may indicate some effects of changes in the law and
practice. Statistical tests can establish that there is a relationship, a correlation, between
different things

36. B

Newcomers to the study of law tend to assume that criminal law occupies the bulk of a
lawyer's caseload and of a law student's studies. This is an interesting by-product of the
portrayal of the legal system by the media.

37. C

Disputes about the scope and interpretation of international law are rarely resolved by the
use of international courts or binding arbitration procedures of an international
organisation. This is because submission to an international court or similar process
is entirely voluntary and few states are likely to agree to this if there is a serious risk
of losing their case or where important political or national interests are at stake.

38. C
FANPAGE TÀI LIỆU TIẾNG ANH NÂNG CAO

The international legal order has no single governing body and operates by agreement
between states. This means that the creation, interpretation and enforcement of
international law lie primarily in the hands of states themselves. Its scope and
effectiveness depend on the sense of mutual benefit and obligation involved in
adhering to the rules.

39. A

Most laws are not about something spectacular but, rather, about the details of
ordinary life.

40. C

There is no world government or legislature issuing and enforcing laws to which all
nations are subject. The international legal order has no single governing body and
operates by agreement between states.

TEST 15
Part 1. For questions 1-13, read the following passage and do the tasks that follow.

Cars that can drive themselves


A. The idea of the self-driving car dates back to before the Second World War. Visitors
to General Motors' 'Futurama' at the 1939 World Fair in New York were shown a vision
of the future in 1960, in which radio-controlled automobiles would zip along perfectly
FANPAGE TÀI LIỆU TIẾNG ANH NÂNG CAO

safe highways. In the 1950s, General Motors and the Radio Corporation of America
began experimenting with such a system, digging radio-transmitting cables into roads at a
cost of around $100,000 per mile.
B. The system worked. In 1958, a Chevrolet impala made it round a special course
'without the driver's hands on the steering wheel' but it joined the list of other expensive
automated dreams. The first real breakthroughs for self-driving cars took place in the late
1980s, in the £800m Eureka Promethesus Project, led by Ernst Dickmanns of the
Bundeswehr University Munich, a pioneer of computer vision. Using just four black and
white cameras, Dickmanns's cars managed two road trips of more than 1,000 kilometres
through Europe in 1994 and 1995, driving without human intervention 95% of the time.
Automated cars have made huge leaps and bounds in recent years. Starting in 2004,
Darpa, the high-tech research wing of the US military, ran a series of competitions for
driverless vehicles. The winning team was hired by Google, and for years the technology
company ran a fleet of automated cars around the highways of California. So far the cars
have clocked up 200,000 miles and have been involved in one minor human-caused
accident. Google is not on its own. Other car manufacturers also have some kind of
automated car in the works, with research and testing also going on in the UK and
Germany.
C. Most driverless cars share the same 'Velodyne' laser system on their roofs. 64 spinning
lasers provide a constant, 3D view of the environment 40 metres around the car, while
radars on the bumpers and a rear-view camera also feed in information. The data is sorted
by algorithms that distinguish between cars, pedestrians, plastic bags and cats, and tell the
car what to do. Automated cars are programmed to be model drivers: they stop when
pedestrians step onto the road; they give way when they should; they stay out of other
cars' blindspots and nudge forward when other cars should be letting them through.
D. Across the world, 1.2 million people are killed or injured on the roads each year with
human error to blame 90% of the time. 'It's amazing to me that we let humans drive cars,'
says Eric Schmidt, the executive director of Google. Paul Newman, a robotics engineer at
Oxford University, says it is only a matter of time before we hand over the wheel. 'It's
crazy to imagine that in ten to twenty years we'll still have to sit behind a wheel,
concentrating hard, not falling asleep and not running over people' , he says. Computer-
controlled cars offer the benefits of safety, fuel efficiency and speed. Roads full of
automated cars, all communicating with one another, will see vehicles going bumper-to-
bumper at 70mph, eliminating traffic jams.
E. In spite of the encouraging pace of development, however, enormous hurdles still
remain, notably how to engineer human common sense. 'Imagine a situation where a box
falls onto the road in front of you,' says John Leonard, a mechanical engineering
professor at the Massachusetts Institute of Technology. 'The system needs to make a
FANPAGE TÀI LIỆU TIẾNG ANH NÂNG CAO

split-second decision to either go straight through it or to swerve left or right - which


might have worse consequences than just going forward'. Other challenges include
recognising the weaknesses of automated cars: how do their sensors respond to glare,
poor weather, or damage?
F. However, no matter how good, their guidance systems are, self-driven cars will still
have accidents. In which case, who will be responsible - the car manufacturer, the
software developer or the human driver - for failing to override the computer at the
critical moment? But will the cars even have people on board? One of the attractions,
surely, will be in ordering them to come and pick us up: in which case will it be empty
car A, or car B with driver, which is to blame for a driving mistake? In whatever form
they emerge, automated cars will require the greatest overhaul of the law of the roads
since the rise of the automobile in the first half of the 20th century.
G. It seems unlikely, at least for the time being, that drivers will hand over their car keys.
A lot of people actually like to drive and hate being passengers. More likely we will see a
continuation of the gradual automation of cars that have been under way for two decades,
during which automatic lane-keeping, cruise control and parking aids have been
gratefully taken up. A new Volvo, for example, now maintains safe distances in heavy
traffic without human intervention, and Nissan is working on software that anticipates a
driver's next move, adjusting the car ahead of time. Piece by piece, radars, lasers, car to
car communication, and the warning of dangers ahead will be added as well, slowly
easing the wheel out of our hands.
Questions 1-7
The reading passage has seven paragraphs A-G. Choose the correct heading for
each paragraph from the list of headings below. Write the correct numbers i- x in
boxes 1-7.

List of headings
i The legal implications of automated driving
ii The limitations of the automated car
iii Towards a partial public acceptance of automated cars
iv The long dream of the automated car
v The downward trend in the demand for automated cars
vi The definition of an automated car
vii The current financial cost of developing automated car technology
FANPAGE TÀI LIỆU TIẾNG ANH NÂNG CAO

viii The rationale behind the concept of the automated car


ix Common technological features of automated cars
x Remarkable success of automated cars on the road

Your answers
1. 2. 3. 4. 5. 6. 7.

Question 8-13
Do the following statements agree with the information given in the Reading Passage?
In boxes 8-13, write
TRUE if the statement agrees with the information.
FALSE if the statement contradicts the information.
NOT GIVEN if there is no information on this.
8. Google’s driverless cars have recorded no technical hitches since their
deployment.
9. It is claimed that human drivers inevitably cede their role to automated cars.
10. The technology could allow cars to drive with a greater degree of safety when
they are closer together.
11. Common sense reasoning would be a nice-to-have for self-driving cars but is not
truly required.
12. The limited capacity to cope with emergencies is a major setback of autonomous
cars.
13. The upgrade of driverless cars would guarantee no car crashes occurring in the
future.
Your answers.

8. 9. 10. 11. 12. 13.


FANPAGE TÀI LIỆU TIẾNG ANH NÂNG CAO

Part 2. For questions 14-23, read an extract from an article and choose the answer
A, B, C or D that fits best according to the text. Write your answers in the
corresponding numbered boxes provided.

Teenage brains
Journalist Martin Baines talks to neuroscientist Professor Sarah-Jayne Blakemore about
the development of the brain during adolescence.

Until I read Inventing Ourselves by Professor Sarah-Jayne Blakemore, I'd always


assumed that what we think of as teenage behaviour is largely an invention of
contemporary western society. I hadn't imagined, for example, that 15-year-olds in the
Kalahari Desert also complain about having to get up early — but they do. It was for
people like me — there are lots of us — that Blakemore wrote her book explaining the
science of everything from why teenagers can't get out of bed in the morning to why they
sometimes appear to be irresponsible narcissists. `We demonise teenagers more than any
other section of society,' she told me. 'And it's not right. They're going through an
essential stage of their development. Most adults don't realise this.'

This is arguably inevitable. Blakemore says that until 20 years ago, it was assumed that
teenage behaviour was largely down to hormonal changes in puberty and that children's
brains were more or less fully developed. The findings of brain scans and psychological
experiments have now revealed that the reality is very different, however. In fact, the
brain continues to change all through the teenage years and well into adulthood, and
important neurodevelopmental processes enable it to be moulded by the environment. So
adolescence is a critical period of neurological change, much of which is responsible for
adolescent behaviour.

Blakemore quotes from a teenager's diary dated 20 July 1969: 'I went to arts centre (by
myself!) in yellow cords and blouse. Ian was there but didn't speak to me. Got a rhyme
put in my handbag from someone who's apparently got a crush on me. It's Nicholas, I
think. UGH. Man landed on the moon.' This may look like amazing — even jaw-
dropping — self-absorption. But Blakemore says it's essential neurological development,
because the biological function of adolescence, today and in the past, is the creation of a
sense of self. Teens achieve this through creating new allegiances, independent of their
parents — which is why their friendships suddenly become extremely important. What is
known on social media as FOMO — fear of missing out — may look like an irrational
sense of priorities if it means skipping revision to attend a run-of-the-mill get-together,
but at that age, nothing matters more than peer approval.

Blakemore designed an experiment to explore this. Adolescents were asked to play an


online ball game, with what they believed to be two other players of their own age. In
fact, the game was with a computer programmed to ignore the human participants; these
FANPAGE TÀI LIỆU TIẾNG ANH NÂNG CAO

found themselves watching the ball being passed between two players on the screen who
chose not to include them. She repeated the experiment with adults, and found that, while
the game lowered the mood and increased the anxiety levels of all participants, the effect
was dramatically greater for the teenagers.

In her book, Blakemore also discusses how the neurologically driven preoccupation with
peer approval underlies adolescent risk-taking and acute self-consciousness, and how it
determines adolescent sleep patterns. It's fascinating, and I'm curious about the
implications. What could be done to accommodate the changes adolescents go through?

Blakemore suggests we might harness the power of peer pressure by getting adolescents
to run educational campaigns — for example, on healthy eating. She also mentions
schools which have altered their start times to fit in with teenage sleep patterns, though
she points out there may be practical issues about implementing this on a wider scale. But
generally, she's wary about putting forward concrete solutions. This seems regrettable,
but she insists she's a scientist rather than a consultant. She doesn't even like to be asked
for parenting advice, although she admits her work does inform her own parenting. When
she recently visited her teenage son's school, he asked her to pretend not to know him. 'I
could've been so offended by that. But I thought: "That's absolutely normal."'

[■] One thing that makes Blakemore's empathy and affection for teenagers so striking is
its rarity. [■] But why does she think so many other adults feel differently? [■] She often
thinks about why we find it hilarious to 'take the mickey out of teenagers'. [■] She
points out that there are whole comedy shows mocking their behaviour. She wonders if
adults do it to cope with their rejection; small children obey adults and want to be with
them, but teenagers, through necessity, look for independence, and the older generations
feel hurt about this. They resent adolescents' desire to rebel and their sense of
embarrassment when they're with parents. 'Our way of dealing with these changes is to
sneer at them.'

14. What is the writer doing in the first paragraph?


A outlining how he feels about the way teenagers are generally perceived
B giving his reasons for writing a profile of Professor Blakemore
C pointing out international variations in teenage behaviour
D acknowledging his relative ignorance about teenagers

15. The writer suggests that the way teenage behaviour is commonly regarded is
unsurprising because
A behavioural development is a complex subject.
B significant progress in relevant areas of study is relatively recent.
C people's outlook on life is partly determined by neurological factors.
D differences between generations will always cause misunderstanding.
FANPAGE TÀI LIỆU TIẾNG ANH NÂNG CAO

16. What does the writer suggest is `jaw-dropping' about the teenager's diary entry?
A the strong desire for independence that comes across
B the similarity with what young people talk about today
C the negative comments about two acquaintances
D the focus on personal rather than wider events

17. The experiment described in the fourth paragraph was designed to provide evidence
of
A how competitive teenagers tend to be.
B how easily teenagers can be deceived.
C how sensitive teenagers are to social exclusion.
D how immersed teenagers can become in video games.

18. In the sixth paragraph, the writer expresses


A disappointment at Blakemore's reluctance to advocate specific policies.
B enthusiasm for the idea of giving teenagers more responsibility.
C amusement at Blakemore's difficulties with her own children.
D doubt regarding the feasibility of changing school hours.

19. Which words in the final paragraph echo the phrase ‘take the mickey out of’?
A cope with
B feel hurt about
C resent
D sneer at

20. Look at the four squares [■] that indicate where the following sentence could be
added to the passage.
'Yes, I'm a champion of them,' she agrees.
Where would the sentence best fit?
A. First square
B. Second square
C. Third square
D. Fourth square

21. What does the word “It” in paragraph 1 refer to?


A. teenage behaviour B. book C. invention D. Kalahari Desert
22. The word ‘demonise’ in paragraph 1 is closest in meaning to:
A. vilify B. eulogize C. overlook D. notice

23. The word ‘run-of-the-mill’ in paragraph 3 is closest in meaning to:


A. distinguished B. ordinary C. tedious D. provocative

Your answers
FANPAGE TÀI LIỆU TIẾNG ANH NÂNG CAO

14. 15. 16. 17. 18.

19. 20. 21. 22. 23.

Part 3. You are going to read an extract from an article. Seven paragraphs have
been removed from the extract. Choose from paragraphs A—H the one which fits
each gap (24-30). There is one extra paragraph which you do not need to use.

My digital detox
Spending a weekend without access to communications technology was an eye-opener.

We were brushing through wet grass in the early morning when we saw it - a flash of
white drifting behind a small patch of trees, backlit by the sun. Crouching down next to
Artley, our twenty-one-month-old son, my partner Will and I watched the unmistakable
shape of a barn owl until it disappeared into the wood. The look on Artley’s face was part
of a brief moment of magic, the kind of memory that we live for. Ordinarily, my next
thought would have been to pull out my phone, take a photo or video and send a message.
Connecting is something I do unconsciously now, and sharing such moments has become
second nature.
24.

That weekend, however, the three of us were, by our own choice, offline. We were
camping at a rural site calied Swallowtail Hill in southern England, which offers visitors
the option of leaving all their electronic devices in the safe keeping of the owner for the
duration of their stay - a kind of digital detox, you might say. We had been inspired by
William Powers' book Hamlet’s BlackBerry, an imaginative and thoughtful work that
explores reactions to new technologies throughout history and the lessons we should have
learnt from them. Blessed with two days of good weather and some delicious local food, I
barely noticed I wasn’t online.

25.

Take equal responsibility for our digital obsession - magnetically drawn, as I am, to any
screen that can feed my addiction. Nonetheless, any objections of mine to this specific
vice are usually swiftly defended by an explanation of the importance of dealing with
whatever it is now, though it never seems anything that couldn’t wait half an hour.
Suddenly, however, we had his full attention - well almost. There was a moment when he
was distracted by a buzzing sensation and automatically reached for his phone, before
realising it was a bee.
26.
FANPAGE TÀI LIỆU TIẾNG ANH NÂNG CAO

By breaking away from my connected life, however, I came to appreciate just how much
it had permeated my way of being. So-called ‘early adopters', the heavy technology users
who throw themselves at every new device and service, will admit to an uncontrollable
impulse to check their email accounts or social networking sites. Researchers have called
this 'variable interval reinforcement schedule'. Such people have in effect been drawn into
digital message addiction because the most exciting rewards are unpredictable.
27.

A study by the University of California concluded that such constant multi-tasking


gradually erodes short-term memory. It also discovered that interruptions to any task
requiring concentration are a massive problem, as it takes us much longer to get back into
them than it does to deal with the interruption itself.
28.

In other words, what was once exterior and faraway is now easily accessible and this
carries a sense of obligation or duty. He sees the feeling that we sh ould be reaching out, or
be available to be reached out to as tied to the self-affirmation that the internet, and all that goes
with it, provides us with.
29.

One practical suggestion, for example, is to use paper as a more efficient way of
organising our thoughts. The theory of ‘embodied interaction' asserts that physical objects
free our minds to think because our hands and fingers can do much of the work, unlike
screens where our brains are constantly in demand.
30.

As we left Swallowtail Hill, we seemed to have achieved that. The real work was just
starting, however, trying to put this and other ideas into regular practice in an attempt to
balance work and home life. Powers also talks about ‘vanishing family trick’, where a
seemingly sociable family gradually dissolves away to screens in different corners of the
house. It's clearly a situation to be avoided. Our digital detox had been something of a
wake-up call. And guess what? When the owner handed back our phones, we didn't have
a missed call or message between us.

The misssing paragraphs


A. At home, those concerns about my digital addiction are most acute when I catch my
son looking at me while I'm checking a screen. It's reinforcing how much more important
the screen is than him, as if I'm teaching him that obeying these machines is what he
needs to do.
B. In truth, he wasn't at one in such lapses. Without our hand-held devices, neither of us
had much idea what the time was. Then, I reached for mine when I wondered about local
shopping facilities and whether it is normal to see a barn owl during the day. And the
FANPAGE TÀI LIỆU TIẾNG ANH NÂNG CAO

magical moment when Artley was being read his bedtime story in front of an open fire,
I've had to try and commit to my own fallible memory.
C. For those of us compelled to check email every few minutes, that finding accounts for
those days which seem to pass so quickly with so little getting done. And this is part of a
wider trend. 'The more we connect, the more our thoughts lean outward,’ Powers writes.
‘There's a preoccupation with what’s going on “out there" in the bustling otherworld,
rather than “in here" with yourself and those right around you.’
D. Like me, he is a true believer in the value and potential of digital technologies. He
concludes, however, that we need to find the discipline to restore control by reintroducing
a little disconnectedness.
E. More radical still is the idea of banning the internet at weekends on the grounds that
being away from it on a regular basis allows us to grasp its utility and value more fully,
Hopefully, it also brings about a shift to a slower, less restless way of thinking, where
you can just be in one place, doing one particular thing, and enjoy it.
F. The meaningful and the mundane have thus merged into one, all dutifully and
habitually recorded - my enjoyment split between that technological impulse and the
more delicate human need to be in the moment. This is how we live these days.
G. What did strike me though was the change in Will’s behaviour. If my worst habit is
incessant messaging, his is allowing his phone to take precedence over everything else.
Country walks, dinner, bathing our son - no moment is safe from the seemingly
irresistible ringing, vibrating, nagging phone, that demands - and wins - his attention
when he should be enjoying the moment with us.
H. What’s more, the hustle they develop as they struggle to keep up with the pace of all
that incoming information has produced a restless, anxious way of engaging with the
world. Desperate for efficiency, this seeps into people's physical lives. Perhaps that's why
I feel compelled to tidy while on the phone, to fold the washing while brushing my teeth,
and no single job has my undivided attention.

Your answers
24. 25. 26. 27. 28. 29. 30.

Part 4. The passage below consists of four sections marked A-D. For questions 31-
40, read the passage and do the task that follows. Write your answers (A-D) in the
corresponding numbered boxes provided.

Is charity as important as we are told it is?


A Mark
No matter how convinced we may be that we are members of a caring, responsible
society that has a well-established safety net, the fact remains that there are still plenty of
people who fall through the cracks. Too many of us sit back, supposedly safe in the
knowledge that the authorities are living up to the duty of care they have for the most
FANPAGE TÀI LIỆU TIẾNG ANH NÂNG CAO

vulnerable members of society. In fact, even a not particularly rigorous examination of


the trials and tribulations faced by some people will reveal plenty of situations in which
they are enduring difficulties that are not even officially acknowledged, never mind
addressed. The nub of the matter is that, in too many cases, people only become aware of
some of the extreme difficulties faced by fellow citizens when they themselves become
victims of the same circumstances - and in the process discover that there is actually no
official they can turn to for help.
B Alice
Charities are organisations set up in an attempt to offer a lifeline to those who, often
through no fault of their own, find themselves in extremely difficult circumstances for
which the state can't or won't give them any meaningful or effective assistance. In the
same way, we must consider situations where we, personally, could step in to give a
hand. For the fortunate amongst us, when we consider our own lives, we will likely
become aware of the extent to which our family members and other people close to us
help us to overcome our problems. Sadly, a lot of people are not blessed with caring
relatives or friends, and this is where we ought to consider how we can be good
neighbours. Of course, we can't do much in this regard if we have little contact with our
neighbours and even less awareness of anything going on in their lives. However, there
are plenty of charities that do try to help people and could do a better job if they had more
assistance or support from us. I believe at we can provide some useful help to people in
our communities if we make the effort to contact a charity that interests us and offer our
support in whatever way we can.
C Julian
In an ideal world, far fewer people would be reliant on charities. The sad reality reflects
poorly on society, particularly as regards the extent to which the authorities are able to
relieve the burdens which some people end up facing alone. I know of regrettable
instances where children are given detention at school for 'offences' that arise simply
because they have to take time out to look after seriously ill or disabled parents. As a
result of these detentions, they then have even less time to support their parents.
Obviously, this creates a vicious circle that will lead the child to missing out on a
significant part of their education and could quite possibly have even worse
consequences. As a result of finding out about someone in such circumstances, I am now
shouldering some of that child's burden, but I have not yet found a charity that I can turn
to for more help.
D Beth
On the whole, I think we're fairly spoilt compared with people who live in certain other
parts of the world. We live in a democracy which has operated a welfare state for several
generations. I appreciate that the budget doesn't always allow enough funding to ensure
that everyone gets adequate support, but we also have a long history of philanthropy.
Personally, I am lucky to have a great network of friends and relatives whom I can turn to
for assistance, but I know that lots of people must get by without this advantage. To tell
the truth, there are quite a few charities that I don't consider to be particularly important
or useful, but that is none of my concern, since I'm not forced to give them donations.
FANPAGE TÀI LIỆU TIẾNG ANH NÂNG CAO

However, there are others that I thoroughly approve of, and I actually volunteer for one
of them in my free time.
Which person gives each of these opinions about charities?
Which person mentions Answers
31. It's difficult to help the people around us if we don't know what
problems they are facing.
32. I value the work of some charitable organisations more highly than that
of others.
33. Often, people only come to understand others' problems through
personal experience.
34. It would be preferable if the work of charities was not so necessary.
35. It is understandable why the public sector can't take on more
responsibility.
36. Quite often an assumption people make results in them being less likely
to take the initiative to help people.
37. It is likely that problems not receiving adequate attention will be
exacerbated.
38. We can all help charities to work more effectively.
39. Someone's personal circumstances should be considered before they are
punished for something that they felt they had to do.
40. It isn't hard to pinpoint examples of people with hardships who are
receiving no help from the authorities.

Key and explanations:


Part 1.
1. iv
2. x
3. ix
4. viii
5. ii
FANPAGE TÀI LIỆU TIẾNG ANH NÂNG CAO

6. i
7. iii
8. NOT GIVEN
Thông tin này có thể gây nhầm lẫn “So far the cars have clocked up 200,000 miles and
have been involved in one minor human-caused accident.” Nhưng chúng ta chỉ biết đây là
lỗi do con người gây ra, không xác định được có phải do vấn đề liên quan đến kĩ thuật
hay không.
9. TRUE
Paul Newman, a robotics engineer at Oxford University, says it is only a matter of time
before we hand over the wheel.
10. TRUE
Roads full of automated cars, all communicating with one another, will see vehicles
going bumper-to-bumper at 70mph, eliminating traffic jams.
11. FALSE
In spite of the encouraging pace of development, however, enormous hurdles still
remain, notably how to engineer human common sense.
12. TRUE
'Imagine a situation where a box falls onto the road in front of you,' says John Leonard, a
mechanical engineering professor at the Massachusetts Institute of Technology. 'The
system needs to make a split-second decision to either go straight through it or to
swerve left or right - which might have worse consequences than just going
forward'. Other challenges include recognising the weaknesses of automated cars: how
do their sensors respond to glare, poor weather, or damage?
13. FALSE
However, no matter how good their guidance systems are, self-driven cars will still have
accidents.
Part 2.
14. D
Until I read Inventing Ourselves by Professor Sarah-Jayne Blakemore, I'd always
assumed that what we think of as teenage behaviour is largely an invention of
contemporary western society. I hadn't imagined, for example, that 15-year-olds in the
Kalahari Desert also complain about having to get up early — but they do.
FANPAGE TÀI LIỆU TIẾNG ANH NÂNG CAO

15. B
This is arguably inevitable. Blakemore says that until 20 years ago, it was assumed that
teenage behaviour was largely down to hormonal changes in puberty and that children's
brains were more or less fully developed. The findings of brain scans and
psychological experiments have now revealed that the reality is very different,
however.
16. D
Blakemore quotes from a teenager's diary dated 20 July 1969: 'I went to arts centre (by
myself!) in yellow cords and blouse. Ian was there but didn't speak to me. Got a rhyme
put in my handbag from someone who's apparently got a crush on me. It's Nicholas, I
think. UGH. Man landed on the moon.' This may look like amazing — even jaw-
dropping - self-absorption. But Blakemore says it's essential neurological development,
because the biological function of adolescence, today and in the past, is the creation of a
sense of self.
17. C
Blakemore designed an experiment to explore this. Adolescents were asked to play an
online ball game, with what they believed to be two other players of their own age. In
fact, the game was with a computer programmed to ignore the human participants; these
found themselves watching the ball being passed between two players on the screen who
chose not to include them.
18. A
But generally, she's wary about putting forward concrete solutions. This seems
regrettable, but she insists she's a scientist rather than a consultant.
19. D
20. B
21. B
22. A
23. B
Part 3.
24. F

Ordinarily, my next thought would have been to pull out my phone, take a phone or
video and send a message. Connecting is something I do unconsciously now, and
FANPAGE TÀI LIỆU TIẾNG ANH NÂNG CAO

sharing such moments has become second nature…My enjoyment split between that
technological impulse and the more delicate human need to be in the moment.

25. G

We had been inspired by William Powers' book Hamlet's BlackBerry, an imaginative and
thoughtful work that explores reactions to new technologies throughout history and the
lessons we should have learnt from them. Blessed with two days of good weather and
some delicious local food, I barely noticed I wasn't offline…What did strike me
though was the change in Will's behaviour. If my worst habit is incessant messaging,
his is allowing his phone to take precedence over everything else.

26. B

There was a moment when he was distracted by a buzzing sensation and


automatically reached for his phone, before realising it was a bee…In truth, he wasn't
alone in such lapses.

27. H

Perhaps that's why I feel compelled to tidy while on the phone, to fold the washing
while brushing my teeth, and no single job has my undivided attention…A study by
the University of California concluded that such constant multi-tasking gradually
erodes short-term memory.

28. C

It also discovered that interruptions to any tasks requiring concentration are a massive
problem, as it takes us much longer to get back into them than it does to deal with the
interruption itself…For those of us compelled to check email every few minutes, that
finding accounts for those days which seem to pass so quickly with so little getting done.

29. D

He concludes, however, that we need to find the discipline to restore control by


reintroducing a little disconnectedness…One practical suggestion, for example, is to
use paper as a more efficient way of organising our thoughts.

30. E

One practical suggestion, for example, is to use paper as a more efficient way of
organising our thoughts…More radical still is the idea of banning the internet at
FANPAGE TÀI LIỆU TIẾNG ANH NÂNG CAO

weekends on the grounds that being away from it on a regular basis allows us to grasp its
utility and value more fully.

Part 4.

31. B
Of course, we can't do much in this regard if we have little contact with our neighbours
and even less awareness of anything going on in their lives.
32. D
To tell the truth, there are quite a few charities that I don't consider to be
particularly important or useful, but that is none of my concern, since I'm not forced to
give them donations. However, there are others that I thoroughly approve of, and I
actually volunteer for one of them in my free time.
33. A
The nub of the matter is that, in too many cases, people only become aware of some of
the extreme difficulties faced by fellow citizens when they themselves become victims
of the same circumstances - and in the process discover that there is actually no official
they can turn to for help.
34. C
In an ideal world, far fewer people would be reliant on charities.
35. D
I appreciate that the budget doesn't always allow enough funding to ensure that
everyone gets adequate support, but we also have a long history of philanthropy.
36. A
Too many of us sit back, supposedly safe in the knowledge that the authorities are
living up to the duty of care they have for the most vulnerable members of society.
37. C
Obviously, this creates a vicious circle that will lead the child to missing out on a
significant part of their education and could quite possibly have even worse
consequences.
38. B
However, there are plenty of charities that do try to help people and could do a better
job if they had more assistance or support from us.
FANPAGE TÀI LIỆU TIẾNG ANH NÂNG CAO

39. C
I know of regrettable instances where children are given detention at school for
'offences' that arise simply because they have to take time out to look after seriously
ill or disabled parents.
40. A
In fact, even a not particularly rigorous examination of the trials and tribulations faced by
some people will reveal plenty of situations in which they are enduring difficulties
that are not even officially acknowledged, never mind addressed.

TEST 16
Part 1. For questions 1-13, read the following passage and do the tasks that follow.

Gesture
A Gesture is any action that sends a visual signal to an onlooker. To become a gesture, an
act has to be seen by someone else and has to communicate some pieces of information to
them. It can do this either because the gesturer deliberately sets out to send a signal or it
can do it only incidentally. The hand-wave is a Primary Gesture, because it has no other
existence or function. Therefore, to make it a gesture, first, it should be clear and
unambiguous. Others would be able to understand it instantly when it is shown to them.
Nor may any component of a gesture, its force, its direction and amplitude of movement,
be altered: otherwise, confusion or misunderstanding may occur.
FANPAGE TÀI LIỆU TIẾNG ANH NÂNG CAO

B Most people tend to limit their use of the term “gesture” to the primary form the hand-
wave type—but this misses an important point. What matters with gesturing is not what
signals we think we are sending out, but what signals are being received. The observers
of our acts will make no distinction between our intentional primary gestures and our
unintentional, incidental ones. This is why it is preferable to use the term “gesture” in its
wider meaning as an “observed action”. This can be compared to the ring of a telephone.
The speed, tone and intensity of a telephone remain the same for any phone call. Even the
length of time before being told that the number you are dialing is not answering, unless
the caller hangs up, is the same.
C Some gestures people use are universal. The shoulder shrug is a case in point. The
shrug is done by bringing the shoulders up, drawing the head in, and turning the palms
upwards so as to reveal that nothing is hidden. The shoulder shrug can also demonstrate
submission or that what is being said isn’t understood. Another example is that an angry
person usually expresses his rage by waving his clenched fist rapidly and forcefully.
Surprisingly, you may find that people of different cultures will do the same when they
are offended. That is to say, a commonly accepted gesture is shared by them. But if the
way the hand is clenched changes, or the amplitude of force and the direction the fist is
waved alters, the gesture no longer means the same.
D So, is gesture born with us or is it developed as we grow up? Recent research found
that gesture is more like a spontaneous reaction when we face certain situations. And we
just do that automatically. When people talk, they almost always gesture with their hands.
This expressive movement can be coaxed into a choreographic form if observed
carefully. People can practice spontaneous gesture by forming pairs, then observing and
questioning each other. They then show the group what they have collected from their
partners. It is fun to surprise a group using this technique. Because spontaneous gestures
are often unconscious, people will sometimes be surprised to have their gestures mirrored
back to them, saying “Did I really do that?”
E The attention of research was also drawn to cultural themes. Researchers discovered
that if a person has a good set of teeth, he or she would be prone to have a bigger smile
than he or she should when good things happen. And if a person possesses a bad set of
teeth, he or she would tend to have his or her mouth shut when being teased. And
people’s reaction to the same joke also varies: some laugh out loud while others titter.
However, this does not cause confusion and it helps to develop our “behavioural”, which
is an important aspect of our identity. It was referred to as a Gesture Variant, which
indicates that individuals’ gesture production is a complex process, in which speakers’
internal and external factors and interactions could play a role in multi-modal
communication.
FANPAGE TÀI LIỆU TIẾNG ANH NÂNG CAO

F During the research, an interesting phenomenon soon caught researchers’ attention. A


hand purse gesture, which is formed by straightening the fingers and thumb of one hand
and bringing them together so the tips touch, pointing upwards and shaping like a cone,
carries different meanings in different countries. In Malta, it means heavy sarcasm: “you
may seem good, but you are really bad.”; in Tunisia, it is against recklessness, saying
“slow down”; in Italy, it means “What’s the matter?” or “What are you trying to say?”; in
France, it means “I am afraid”. However, this gesture has no clear meaning in American
culture. And of course, the way the gesture is conducted is similar in different countries.
G But what will happen if the gestures of different countries confront each other? The
situation is further complicated by the fact that some gestures mean totally different
things in different countries. To take one example, in Saudi Arabia, stupidity can be
signalled by touching the lower eyelid with the tip of the forefinger. But this same
gesture, in various other countries, can mean disbelief, approval, agreement, mistrust,
scepticism, alertness, secrecy, craftiness, danger, or criminality. So people are faced with
two basic problems where certain gestures are concerned: either one meaning may be
signalled by different actions, or several meanings may be signalled by the same action,
as we move from culture to culture. The only solution is to approach each culture with an
open mind and learn their gestures as one would learn their vocabulary. These all require
considerable skill and training and belong in a totally different world from the familiar
gestures we employ in everyday life.
Questions 1-7
The passage has seven paragraphs, A-G.
Choose the correct heading for paragraph A-G and from the list of headings below.
Write the correct number, i-x, in boxes 1-7.
List of Headings
i The subconscious nature of gestures-D
ii The example of regional differences-F
iii The key factors of gestures-A
iv Sending out important signals
v How a well-known gesture loses its meaning-C
vi Performance in a specific setting-
vii Recent research of Gesture Variant-
viii Comparison to an everyday-use object-B
FANPAGE TÀI LIỆU TIẾNG ANH NÂNG CAO

ix How will conflict be handled-G


x Individual deviation of cultural norms-E

1 Paragraph A
2 Paragraph B
3 Paragraph C
4 Paragraph D
5 Paragraph E
6 Paragraph F
7 Paragraph G
Your answers

1. 2. 3. 4. 5. 6. 7.

Questions 8-13
Do the following statements agree with the information given in the passage.
In boxes 8-13, write
TRUE if the statement agrees with the information
FALSE if the statement contradicts the information
NOT GIVEN if there is no information on this
8. A gesture is a form of communication in which imperceptible bodily actions
communicate particular messages.-T-> F
9. Gestures can be likened to the unchanging sound of the ringing. -T
10. Angry people are often in the same age range or group. -NG
11. A Gesture Variant can still be understood by the members of the same culture.-T
12. In Malta, the gesture “Hand Purse” should be treated with caution.-T
FANPAGE TÀI LIỆU TIẾNG ANH NÂNG CAO

13. The main aim of the writer in writing this passage is to clarify the origin of gesture-
based communication. -F
Your answers
8. 9. 10. 11. 12. 13.

Part 2. For questions 14-23, read an extract from an article and choose the answer
A, B, C or D that fits best according to the text. Write your answers in the
corresponding numbered boxes provided.
Line It has come to pass that working men have been surrendered, isolated
and helpless, to the hard heartedness of employers and the greed of
unchecked competition. The mischief has been increased by rapacious
usury, which, although more than once condemned by the Church, is
5 nevertheless, under a different guise, but with like injustice, still
practiced by covetous and grasping men. To this must be added that
the hiring of labor and the conduct of trade are concentrated in the
hands of comparatively few; so that a small number of very rich men
have been able to lay upon the teeming masses of the laboring poor a
10 yoke little better than that of slavery itself.

To remedy these wrongs the socialists, working on the poor man's envy
of the rich, are striving to do away with private property, and contend
that individual possessions should become the common property of all,
to be administered by the State or by municipal bodies. They hold that
15 by thus transferring property from private individuals to the community,
the present mischievous state of things will be set to rights, inasmuch as
each citizen will then get his fair share of whatever there is to enjoy.
But their contentions are so clearly powerless to end the controversy
that were they carried into effect the working man himself would be
20 among the first to suffer. They are, moreover, emphatically unjust, for
they would rob the lawful possessor, distort the functions of the
State, and create utter confusion in the community.

It is surely undeniable that, when a man engages in remunerative labor, the


impelling reason and motive of his work is to obtain property, and
25 thereafter to hold it as his very own. If one man hires out to another his
strength or skill, he does so for the purpose of receiving in return what is
necessary for the satisfaction of his needs; he therefore expressly intends to
FANPAGE TÀI LIỆU TIẾNG ANH NÂNG CAO

acquire a right full and real, not only to the remuneration, but also to the
disposal of such remuneration, just as he pleases. Thus, if he lives
30 sparingly, saves money, and, for greater security, invests his savings in
land, the landy in such case, is only his wages under another form; and,
consequently, a working man's little estate thus purchased should be as
completely at his full disposal as are the wages he receives for his labor.
But it is precisely in such power of disposal that ownership obtains,
35 whether the property consist of land or chattels. Socialists, therefore, by
endeavoring to transfer the possessions of individuals to the conununity at
large, strike at the interests of every wage-earner, since they would deprive
him of the liberty of disposing of his wages, and thereby of all hope and
possibility of increasing his resources and of bettering his condition in life.

40 What is of far greater moment, however, is the fact that the remedy they
propose is manifestly against justice. For, every man has by nature the right
to possess property as his own. This is one of the chief points of distinction
between man and the animal creation, for the brute has no power of self
direction. It is the mind, or reason, which is the predominant element in us
45 who are human creatures; it is this which renders a human being human,
and distinguishes him essentially from the brute. And on this very account
—that man alone among the animal creation is endowed with reason—it
must be within his right to possess things not merely for temporary and
momentary use, as other living things do, but to have and to hold then in
50 stable and permanent possession; he must have not only things that perish
in the use, but those also which, though they have been reduced into use,
continue for further use in after time.

14. Based on the passage, the author would be most likely to agree that the economic
conditions of his era attest to which basic relationship?
A. Modifications to the laws regulating the economy heighten tensions between
employers and workers
B. Social inequality worsens as the economy increasingly comes under the control of a
select group of businesspeople
C. Competition between businesses in a particular sector of the economy influences the
morale of the workers in that sector
D. Measures to protect workers from economic exploitation have the unintended effects
of encouraging such exploitation
FANPAGE TÀI LIỆU TIẾNG ANH NÂNG CAO

15. Which choice provides the best evidence for the answer to the previous question?
A. Lines 1 – 3 (“It has … competition”)
B. Lines 3 – 6 (“The … men”)
C. Lines 6 – 10 (“To this … itself”)
D. Lines (“To remedy … bodies”)
16. As used in line 13, “common” most nearly means
A. shared
B. inferior
C. typical
D. frequent
17. According to the author, the desire to acquire property prompts individuals to
A. feel envious of others’ possessions
B. seek work from an employer
C. view work as more dignified than they otherwise would
D. resist sources of short-lived gratification
18. In lines 31 -32, the phrase “little estate” most directly refers to the
A. land or other property owned by a wage earner
B. status of a wage earner within the community
C. neighborhoods where wage earners typically live
D. standard amount of pay that a wage earner receives
19. It can reasonably be inferred from the passage that the author views socialism as
being threatening because it
A. discourages disempowered groups from seeking greater contribution in society
B. contributes to widespread discontent over the pace of societal change
C. decreases the individual’s willingness to make sacrifices benefiting society as a whole
D. endangers certain conventions and institutions that are indispensable to society
FANPAGE TÀI LIỆU TIẾNG ANH NÂNG CAO

20. According to the author, what is one essential aspect of the concept of private
property?
A. The ease with which land holdings can be exchanged for other forms of property
B. The belief that people can amass a great deal of property through living frugally
C. The freedom of property holders to determine what to do with their property
D. The likelihood that those who purchase property will profit from its resale
21. The author implies that those who wish to abolish the ownership of private property
fail to recognize that such ownership is beneficial in that it
A. promotes a single set of values among the wealthy and the working class alike
B. provides members of the working class a means to improve their circumstances
C. induces workers to feel content with the compensation they receive for their labor
D. helps to ensure that the various social classes live in harmony with one another
22. Which choice provides the best evidence for the answer to the previous question?
A. Lines 14-17 ("They . . . enjoy")
B. Lines 18-20 ("But their . . . suffer")
C. Lines 23-25 ("It is . . . own")
D. Lines 35-39 ("Socialists . . . life")
23. In the last paragraph, the discussion of "animal creation" serves mainly to
A. assert a central difference between humans and animals.
B. underscore humans' responsibilities toward animals.
C. consider humans' right to keep animals as property.
D. draw attention to the basic needs of humans and animals.
Your answers
14. 15. 16. 17. 18.

19. 20. 21. 22. 23.


FANPAGE TÀI LIỆU TIẾNG ANH NÂNG CAO

Part 3. In the passage below, seven paragraphs have been removed. For questions
24-30, read the passage and choose from the paragraphs A-H the one which fits each
gap. There is ONE extra paragraph which you do not need to use. Write your
answers in the corresponding numbered box provided.

Women in the Sciences

The expression 'behind every great man is a great woman' has been in use since the mid-
1940s, but undoubtedly the meaning behind this saying has been true for centuries. This
phrase more literally refers to both the practical and emotional support women can give
to their significant others who toil for success, yet it also infers the disheartening idea that
perhaps women haven't previously had the opportunity to revel in their own successes.

24.

Rosalind Franklin is a name you might recognise now, but just seventy years ago her
research was overlooked in terms of its instrumental contribution to the discovery of the
structure of DNA. The acclaim for the discovery, however, and even the Nobel Prize,
went to three men, Francis Crick, James Watson and Maurice Wilkins, without any
reference to Franklin's input.

25.

Incidents like this have occurred so often within the field that the term 'the Matilda Effect'
was coined in 1993 by Margaret W. Rossiter, a scientist who has devoted her career to
shining a light on the generally overlooked female scientists who were rather brazenly
excluded from the history books. One of Rossiter's aims has b een that a renewed focus on
successes of female scientists in history may encourage more women to enter the field of
science.
26.

Still, efforts continue to publicise the overlooked female scientists in history. From a
series of pop art posters showcasing pioneering scientists like Ada Lovelace and Chien-
Shiung Wu, to even rectifying past injustices by amending search-engine results to reflect
real contributions, the cause of female scientists is gradually strengthening.

27.
FANPAGE TÀI LIỆU TIẾNG ANH NÂNG CAO

However, perhaps scientific traditions need to be rectified not just by the gender divide
but by using a broader lens. There is plenty of discussion nowadays about how women
have frequently been underestimated and overlooked by their male colleagues, as
illustrated by the examples above. However, rather than this being a pure case of gender
inequality, perhaps the defining factor is simply the historically unequal power
relationship between parties.

28.

While we see efforts to shine a light on the females whose contributions have been
unnoticed by the general scientific community, maybe it will be a little harder to find
those men who were overlooked in favour of someone with a greater stature. After all, it
is near impossible to explore these kinds of power relationships so long after the event.

29.

Perhaps science will never reach an ideal world of an individual's contribution being
equal to their reward, but opening this subject up for reflection and discussion is
essential, and equal input for equal credit is just one aim to strive for. It has encouraged
females into the sciences already, and both the terms 'the Matilda Effect' and 'the
Matthew Effect' by their very use are making inroads into highlighting inequalities.

30.

The missing paragraphs


A The Matthew Effect can be summarised as the way in which disproportionate
recognition is attributed to someone who is more famous or in a position of power. So,
for example, an acclaimed scientist will naturally get more credit than a lesser-known
researcher, even if their work is comparable. This may seem a given, as the most
powerful or famous team member often leads the research, but that doesn't mean there
isn't a talent in the background contributing the lion's share of the scientific endeavour.

B Due to this growing number of complaints, especially those from female research
scientists at universities, some institutions are now addressing the issue. They are doing
this in a twofold manner: by shining a light on incidences in their own institutions that
have been caused by a gender power inequality and offering further opportunities and
support to women in the sciences. This has been heartily welcomed by the wider
scientific community.
FANPAGE TÀI LIỆU TIẾNG ANH NÂNG CAO

C One effort we can all make is to encourage a shift in people's attitudes in general and
ensure that this shift is also reflected in the world of science. We can see flaws in the past
and the present, as illustrated, and research shows that male and female stereotypes still
exist when it comes to the perceived quality of female work, and common sense can tell
us that the most powerful names get the most credit. However, that doesn't mean things
should remain the same in the years to come.

D However, perhaps the views on female scientists are too deeply embedded in the
scientific community for the situation to be rectified overnight. A 2013 paper found that
male scientists and more masculine topics, regardless of who wrote them, are perceived
as being of higher scientific quality. In the investigation, graduate students of both sexes
rated abstracts of papers that were assigned a fake male or female name, and the fake
male names were more highly rated overall. In addition, the same study indicated that
men are more desirable as collaboration partners.

E Never has this bleak interpretation been more accurate than in the field of science,
where women have usually taken a back seat, and not by choice. Examples of this can be
observed throughout history, as far back as the 12th century, when physician Trotula of
Salerno had her ground-breaking work attributed to men, and in the modern day where
female science professionals still sometimes struggle for appropriate recognition.

F For instance, try finding out about 'the Dean Method' online. A quick google will offer
you little mention of that particular term, but instead it will return a multitude of results
for scientist Alice Augusta Ball. The Dean Method, published by chemist and academic
Arthur L. Dean, was widely known as the cure for leprosy, an infectious condition that
used to cause severe skin sores and often resulted in limbs withering. However, it
eventually transpired that, upon the death of Alice Augusta Ball in 1916, Arthur Dean
had taken her efforts and claimed them as his own.

G You may be reading this article thinking that this is an issue you are powerless to
change, but you make up part of the world we share, and a sea change only happens
through the individual shifts in people's opinions. So, the next time you read about the
latest greatest discovery or the history of science, it might well be worth remembering
that behind every big name, there are many others who make valuable, if not the most
valuable contributions to research.

H Even in one of the winner's memoirs, we can see the attitude displayed towards her,
with the mentioned colleague even failing to call her by her proper name, preferring
nicknames that served to belittle her role in this ground-breaking finding. The author did
acknowledge her achievements in his book, but this admission was fifteen years too late.

Your answers
FANPAGE TÀI LIỆU TIẾNG ANH NÂNG CAO

24. 25. 26. 27. 28. 29. 30.

Part 4. The passage below consists of five sections marked A-E. For questions 31-40,
read the passage and do the task that follows. Write your answers (A-E) in the
corresponding numbered boxes provided.

The Modern City


Jacob Moore spoke with five city dwellers to find out what they think are the problems
with modern cities.
A Iain Bracewell
It goes without saying that modern cities are somewhat problematic, simply because we
don’t have the capacity for all the people who already live here, let alone the millions
who see cities as a potential destination. Therefore, in my mind, it’s vital that we become
a bit more imaginative about how we utilise city space. We’re going to be somewhat
reliant on technology to help us with this, by, for example, developing materials that we
can use to build higher, slimmer and underground. This might seem less than ideal for the
average city dweller, especially the notion having to spend a proportion of time below
ground, but it might be the only practical solution to what the data suggest if we want to
avoid cities growing at an uncontrolled rate across our countryside. And time is of the
essence; we can’t put this kind of research and development off it while the population
growth remains uncontained.
B Raphael Arco
Cities are often seen in a bad light, but I think this is undeserved because they offer so
much to so many. The fact is that cities are synonymous with opportunity, for
employment, culture, you name it! That’s not to say they are utopias without any room
for improvement, but I think we can solve a great majority of the issues affecting cities
by addressing their infrastructure. People often cite their bugbears as being issues of
convenience such as streets that aren’t walkable or road networks that are too dense, or
even lack of space for increased public transport. Devote serious attention on improving
these elements and cities will become far more liveable places with, in turn, generally all-
round happier residents! This might consist of tweaks or alternatively, in certain contexts,
starting from scratch to fundamentally redesign systems, but the benefits outweigh the
sacrifices as they’d offer valuable solutions to how modern city life affects the natural
environment and how well people gel together as a community.
C Jenna Crawford
We need to face up to the fact that most cities aren’t the glorious places that they are
painted to be, and that, for the majority of the inhabitants, the streets aren’t paved with
gold. There is a big difference between the haves and have-nots, and while city life is a
consumer paradise for the former, members of the latter category are completely locked
out of the benefits cities bring and often lack fundamental services such as clean water or
FANPAGE TÀI LIỆU TIẾNG ANH NÂNG CAO

sanitation. Why this is still allowed in the modern world I’ll never know, yet the powers
that be seem disinclined to do anything but sweep the issue under the carpet. Property
rental prices are also excessive, and this just gets to the point where you’ve got huge
families living cramped in just two rooms, or people receiving full-time salaries with
little to no chance of getting on the housing ladder. I understand that people think there
are valuable opportunities to be had in cities, but, let’s face it, there are still plenty of
people for whom opportunity has passed by.
D Caroline Birkenstein
Our cities right now are in dire straits. We’ve got an affordable housing and ecological
crisis in nearly every city on Earth, and it’s crucial that we concentrate our efforts on
these matters if we want our cities to continue to thrive. We can accomplish this by
creating and promoting more sharing and communal practices, like coworking spaces or
apartment buildings with common spaces for eating, socialising and exercising, and
these, of course, shouldn’t be extravagantly priced. It might not seem obvious, but it’s
initiatives like these that help people form communities, and this community atmosphere
encourages people to care more about their surroundings. Cities are also a massive drain
on resources, and we need to identify strategies to counter this and close the loop when it
comes to this. With this in mind, we should ask ourselves how one excess can be used to
give power to something else. This kind of sustainability could be the key to making our
cities much healthier places for individuals, the community and the surroundings we live
in.
E Doug de Souza
Cities today have one major problem that we need to curb, and that is urban sprawl. At
the moment, cities are like these huge sprawls, just spreading and spreading, and the
further out you go, the bigger plot each homeowner has and the more spacious all the
services are. This really has a negative effect on so many elements of our lives. Firstly, it
makes us more isolated; we’re behind fences, and this is where feelings of difference and
fear can stem from. We need integration to help people consider themselves a part of
something, but, furthermore, sprawl increases the urban footprint significantly, and
people start becoming dependent on their cars, simply because it’s not convenient to go
anywhere on foot – rather, driving becomes the preferred option. I mean, I don’t think it
takes a scientist to see the environmental problems that can arise from that.

In which sections are the following mentioned?


31. Cities need to focus on how they can reduce one-time consumption.
32. A fix that will take a varying amount of effort.
33. Cities have a common reputation that overlooks their positive aspects.
34. An acknowledgement that the solution may be disagreeable to some.
35. Our cities are designed in a way that makes us feel detached from others.
36. How people can become more integrated in cities.
37. Some people are ignoring problems that we should be tackling.
38. Cities have been left to grow virtually unchecked.
FANPAGE TÀI LIỆU TIẾNG ANH NÂNG CAO

39. Future enhancements will be determined by fundamental elements of construction.


40. The reputation of cities and the reality of cities are different.

Your answers
31. 32. 33. 34. 35.

36. 37. 38. 39. 40.

Key and explanations:

Part 1.
1.iii
2. viii
3. v
4. i
FANPAGE TÀI LIỆU TIẾNG ANH NÂNG CAO

5. x
6. ii
7. ix
8. FALSE
Gesture is any action that sends a visual signal to an onlooker. To become a gesture, an
act has to be seen by someone else and has to communicate some pieces of information
to them.
9. TRUE
This can be compared to the ring of a telephone. The speed, tone and intensity of a
telephone remain the same for any phone call.
10. NOT GIVEN
11. TRUE
However, this does not cause confusion and it helps to develop our “behavioural”,
which is an important aspect of our identity. It was referred to as a Gesture Variant,
which indicates that individuals’ gesture production is a complex process, in which
speakers’ internal and external factors and interactions could play a role in multi-modal
communication.
12. TRUE
In Malta, it means heavy sarcasm: “you may seem good, but you are really bad.”;
13. FALSE
Part 2.
14. B
15. C
16. A
17. B
18. A
19. D
20. C
21. B
FANPAGE TÀI LIỆU TIẾNG ANH NÂNG CAO

22. D
23. A
Part 3.
24. E
The expression 'behind every great man is a great woman' has been in use since the mid-
1940s, but undoubtedly the meaning behind this saying has been true for centuries. This
phrase more literally refers to both the practical and emotional support women can give
to their significant others who toil for success, yet it also infers the disheartening idea
that perhaps women haven't previously had the opportunity to revel in their own
successes… Never has this bleak interpretation been more accurate than in the field of
science, where women have usually taken a back seat, and not by choice.
25. H
The acclaim for the discovery, however, and even the Nobel Prize, went to three men,
Francis Crick, James Watson and Maurice Wilkins, without any reference to Franklin's
input…Even in one of the winner's memoirs, we can see the attitude displayed towards
her, with the mentioned colleague even failing to call her by her proper name, preferring
nicknames that served to belittle her role in this ground-breaking finding
26. D
However, perhaps the views on female scientists are too deeply embedded in the
scientific community for the situation to be rectified overnight… Still, efforts
continue to publicise the overlooked female scientists in history.
27. F
From a series of pop art posters showcasing pioneering scientists like Ada Lovelace and
Chien-Shiung Wu, to even rectifying past injustices by amending search-engine results
to reflect real contributions, the cause of female scientists is gradually strengthening…
For instance, try finding out about 'the Dean Method' online.
28. A
However, rather than this being a pure case of gender inequality, perhaps the defining
factor is simply the historically unequal power relationship between parties… The
Matthew Effect can be summarised as the way in which disproportionate recognition is
attributed to someone who is more famous or in a position of power
29. C
FANPAGE TÀI LIỆU TIẾNG ANH NÂNG CAO

One effort we can all make is to encourage a shift in people's attitudes in general and
ensure that this shift is also reflected in the world of science… Perhaps science will never
reach an ideal world of an individual's contribution being equal to their reward, but
opening this subject up for reflection and discussion is essential, and equal input for
equal credit is just one aim to strive for.
30. G
Perhaps science will never reach an ideal world of an individual's contribution being
equal to their reward, but opening this subject up for reflection and discussion is
essential, and equal input for equal credit is just one aim to strive for… You may be
reading this article thinking that this is an issue you are powerless to change, but you
make up part of the world we share, and a sea change only happens through the
individual shifts in people's opinions.
Part 4.
31. D
With this in mind, we should ask ourselves how one excess can be used to give power to
something else. This kind of sustainability
32. B
This might consist of making tweaks or, in certain contexts, starting from scratch
33. B
Cities are often seen in a bad light, but I think this is undeserved, because they offer so
much to so many
34. A
This might seem less than ideal for the average city dweller
35. E
cities are like these huge sprawls, just spreading and spreading / the bigger plot each
homeowner has and the more spacious / this really has a negative effect / makes us more
isolated
36. D
creating and promoting more sharing and communal practices, like coworking spaces or
apartment buildings with common spaces
37. C
The powers that be seem disinclined to do anything but sweep this under the carpet
FANPAGE TÀI LIỆU TIẾNG ANH NÂNG CAO

38. E
Cities today have one fundamental problem that we need to curb, and that is urban
sprawl. / cities are like these huge sprawls, just spreading and spreading
39. A
We’re going to be somewhat reliant on technology to help us with this, by, for example,
developing materials that we can use to build higher…
40. C
We need to face up to the fact that most cities aren’t the glorious places that they are
painted to be, and that, for the majority of the inhabitants, the streets aren’t paved with
gold

TEST 17
Part 1. For questions 1-13, read the following passage and do the tasks that follow.

Follow your nose


A Aromatherapy is the most widely used complementary therapy in the National
Health Service, and doctors use it most often for treating dementia. For elderly patients
who have difficulty interacting verbally, and to whom conventional medicine has little to
offer, aromatherapy can bring benefits in terms of better sleep, improved motivation, and
less disturbed behaviour. So the thinking goes. But last year, a systematic review of
FANPAGE TÀI LIỆU TIẾNG ANH NÂNG CAO

health care databases found almost no evidence that aromatherapy is effective in the
treatment of dementia. Other findings suggest that aromatherapy works only if you
believe it will. In fact, the only research that has unequivocally shown it to have an effect
has been carried out on animals.
B Behavioural studies have consistently shown that odours elicit emotional
memories far more readily than other sensory cues. And earlier this year, Rachel Herz, of
Brown University in Providence, Rhode Island, and colleagues peered into people’s
heads using functional Magnetic Resonance Imaging (fMRI) to corroborate that. They
scanned the brains of five women while they either looked at a photo of a bottle of
perfume that evoked a pleasant memory for them, or smelled that perfume. One woman,
for instance, remembered how as a child living in Paris—she would watch with
excitement as her mother dressed to go out and sprayed herself with that perfume. The
women themselves described the perfume as far more evocative than the photo, and Herz
and co-workers found that the scent did indeed activate the amygdala and other brain
regions associated with emotion processing far more strongly than the photograph. But
the interesting thing was that the memory itself was no better recalled by the odour than
by the picture. “People don’t remember any more detail or with any more clarity when
the memory is recalled with an odour,” she says. “However, with the odour, you have this
intense emotional feeling that’s really visceral.”
C That’s hardly surprising, Herz thinks, given how the brain has evolved. “The way I
like to think about it is that emotion and olfaction are essentially the same things,” she
says. “The part of the brain that controls emotion literally grew out of the part of the brain
that controls smell.” That, she says, probably explains why memories for odours that are
associated with intense emotions are so strongly entrenched in us, because the smell was
initially a survival skill: a signal to approach or to avoid.
D Eric Vermetten, a psychiatrist at the University of Utrecht in the Netherlands, says
that doctors have long known about the potential of smells to act as traumatic reminders,
but the evidence has been largely anecdotal. Last year, he and others set out to document
it by describing three cases of post-traumatic stress disorder (PTSD) in which patients
reported either that a certain smell triggered their flashbacks, or that smell was a feature
of the flashback itself. The researchers concluded that odours could be made use of in
exposure therapy, or for reconditioning patients’ fear responses.
E After Vermetten presented his findings at a conference, doctors in the audience
told him how they had turned this association around and put it to good use. PTSD
patients often undergo group therapy, but the therapy itself can expose them to traumatic
reminders. “Some clinicians put a strip of vanilla or a strong, pleasant, everyday odorant
such as coffee under their patients’ noses, so that they have this continuous olfactory
stimulation,” says Vermetten. So armed, the patients seem to be better protected against
FANPAGE TÀI LIỆU TIẾNG ANH NÂNG CAO

flashbacks. It’s purely anecdotal, and nobody knows what’s happening in the brain, says
Vermetten, but it’s possible that the neural pathways by which the odour elicits the
pleasant, everyday memory override the fear-conditioned neural pathways that respond to
verbal cues.
F According to Herz, the therapeutic potential of odours could lie in their very
unreliability. She has shown with her perfume-bottle experiment that they don’t
guarantee any better recall, even if the memories they elicit feel more real. And there’s
plenty of research to show that our noses can be tricked, because being predominantly
visual and verbal creatures, we put more faith in those other modalities. In 2001, for
instance, Gil Morrot, of the National Institute for Agronomic Research in Montpellier,
tricked 54 oenology students by secretly colouring a white wine with an odourless red
dye just before they were asked to describe the odours of a range of red and white wines.
The students described the coloured wine using terms typically reserved for red wines.
What’s more, just like experts, they used terms alluding to the wine’s redness and
darkness—visual rather than olfactory qualities. Smell, the researchers concluded, cannot
be separated from the other senses.
G Last July, Jay Gottfried and Ray Dolan of the Wellcome Department of Imaging
Neuroscience in London took that research a step further when they tested people’s
response times in naming an odour, either when presented with an image that was
associated with the odour or one that was not. So, they asked them to sniff vanilla and
simultaneously showed them either a picture of ice cream or of cheese, while scanning
their brains in a fMRI machine. People named the smells faster when the picture showed
something semantically related to them, and when that happened, a structure called the
hippocampus was strongly activated. The researchers’ interpretation was that the
hippocampus plays a role in integrating information from the senses— information that
the brain then uses to decide what it is perceiving.

Questions 1-7
The passage has seven paragraphs, A-G.
Choose the correct heading for paragraph A-G and from the list of headings below.
Write the correct number, i-x, in boxes 1-7.
List of Headings
i Remembering the past more clearly
ii Bringing back painful memories
iii Originally an alarm signal
FANPAGE TÀI LIỆU TIẾNG ANH NÂNG CAO

iv The physical effects of scent versus image


v Checking unreliable evidence
vi Reinforcing one sense with another
vii Protection against reliving the past
viii The overriding power of sight and sound
ix Conflicting views

1 Paragraph A
2 Paragraph B
3 Paragraph C
4 Paragraph D
5 Paragraph E
6 Paragraph F
7 Paragraph G
Your answers

1. 2. 3. 4. 5. 6. 7.

Questions 8-13
Look at the following findings (Questions 8-13) and the list of researchers
Match each finding with the correct researcher, A-D.
Write the correct letter, A-D, in boxes 8-13.
NB You may use any letter more than once.
8 Smell can trigger images of horrible events.
9 Memory cannot get sharper by smell.
FANPAGE TÀI LIỆU TIẾNG ANH NÂNG CAO

10 When people are given an odour and a picture of something to learn, they will
respond more quickly in naming the smell because the stimulus is stronger when two or
more senses are involved.
11 Pleasant smells counteract unpleasant recollections.
12 It is impossible to isolate smell from visual cues.
13 The part of brain that governs emotion is more stimulated by smell than an image.
A Rachel Hertz
B Eric Vermetten
C Gil Morrot
D Jay Gottfried and Ray Dolan

Your answers
8. 9. 10. 11. 12. 13.

Part 2. For questions 14-23, read an extract from an article and choose the answer
A, B, C or D that fits best according to the text. Write your answers in the
corresponding numbered boxes provided.

The painted smile


Smiles make us smile, they are infectious. We are certain we understand what a smile
means. It's an expression of happiness or pleasure, easy to spot and easy to understand,
and it charms us, People seem at their most natural and spontaneous when they smile -
we all know what a forced smile looks like. We smile automatically when someone
points a camera at us; it’s a social and cultural reflex, and what we expect of a picture
portrait. Yet the smile, like everything else, has a history. If you walk around any art
gallery and look at the portraits you'll find very few genuine smiles. The same is true of
early photograph portraits. They were, it seems, unfashionable for a long period in the
history of portraiture.
Physically, a smile itself is not complex - it comes when we contract the zygornotic
major, a muscle in the face - but the meaning of smiles changes with the mood of the
times, and can even be different for men and women in the same culture. Still walking
around the gallery, you will find more smirks than smiles. One reason for this may be that
with a smile, what you see is what you get, a frozen moment of happy
FANPAGE TÀI LIỆU TIẾNG ANH NÂNG CAO

unselfconsciousness. A smirk is more ambiguous, allowing for several interpretations -


condescension, flirtatiousness, boredom, and so on - and forces a longer and deeper
engagement with the painting from the viewer.
The most famous smile/smirk in portraiture 15, of course, Leonardo's Morro Lisa. Artists
great and small have been obsessed by it, critics too. It became a cult in the 19th century,
and millions of words have been written about what it might mean and what she
represents. There is something in it that draws us back to either confirm or revise our
interpretation, always demanding further examination. At first look she seems to be
welcoming, inviting you to join her; look again and she's changed her mind about you.
Some she attracts, others she repels. To the critic John Ruskin the painting was merely a
caricature: to Walter Pater it contained everything. To some it is romantic, to others it can
illustrate that withering look women sometimes bestow on obtuse men that illustrates the
adage, 'He that will not when he may/Shall not when he would'.
So the smirk allows for a wider range of interpretation than the smile, but that doesn't
really explain why the smile was so rare for so long. One theory, is that back then
everybody had terrible teeth and didn't want to reveal a gap-toothed, black-toothed mouth
to the world. But can this really be the reason? At a time when everybody had bad teeth,
would it have made people particularly shy or embarrassed, or others less attractive? Yet,
both painters and sitters did have a number of reasons for being disinclined to encourage
the smile.
The first consideration is practical - smiles are hard to do, for both painter and sitter. For
the sitter's point of view, you can use your own experience of having your photograph
taken. [■] When someone gets out their camera and asks you to smile, you do so quite
easily; but if they start fiddling about with focus and angles and depth of field, the smile
freezes on your face. [■] Imagine having to produce the same smile on demand when you
go through the process of having your portrait painted. [■] Besides, a smile isn't really an
expression, it's a response, so cannot easily be maintained or recorded. [■]
We have to remember what a portrait was for; what it meant and represented. Nowadays,
we all take thousands of pictures of friends and family, and they of us, often smiling, so
we have a record of ourselves in almost every mood and context, and we don't have to
worry about being defined by one picture. We also have to remember that smiling has a
large number of discrete cultural and historical significances, few of them being in line
with our idea of it being a sign of warmth, enjoyment or happiness. In fact, by the 17th
century it was generally agreed that the only people who smiled, in life and in art, were
the poor, the simple, the drunk or the mad. You wouldn't catch an aristocrat doing it.
You needed money to have your portrait painted by a known artist, and the portraits that
hang in the galleries are usually of prominent people: royals, politicians, generals, high
society, Portraits are for posterity. A smile would detract from their gravitas - these are
FANPAGE TÀI LIỆU TIẾNG ANH NÂNG CAO

serious people. The idea was not to capture a mood, let alone a frivolous one, nor even a
record of a certain person at a certain time, but to provide an example, a moral certainty.
14. In the first paragraph, the writer suggests that
A. people in the past didn't smile as much as we do.
B. portraiture wasn't as popular as it is now
C. what makes us smile is culturally determined.
D. historically, the smile was a social reflex.
15. Why might an artist prefer to paint a smirk to a smile?
A. People will spend more time looking at the painting.
B. It is physically more complex to do than a smile.
C. A smile does not convey the personality of a sitter.
D. Because artists like to confuse their spectators.
16. Which statement best expresses the writer's view of the Mono Lisa?
A. Too much has been written about it.
B. It defies any fixed interpretations.
C. It has become a cult object.
D. It represents missed opportunities.
17. Why doesn't the writer believe the 'bad teeth' theory?
A. Smiling was generally frowned upon in those days.
B. People weren't so self-conscious in those days
C. In the past bad teeth weren't considered unattractive.
D. Sitters and artists had their own reasons for not smiling.
18. Why is it difficult to paint smiles?
A. Smiles elicit the wrong response from spectators.
B. They always look more natural in photographs.
C. People find it difficult to smile if not in the right mood.
D. It's hard to maintain a smile for the length of a sitting.
FANPAGE TÀI LIỆU TIẾNG ANH NÂNG CAO

19. What does the writer say about portrait painting in the last two paragraphs?
A. Artists had to depict a serious expression for future generations.
B. Poor people never used to be chosen as subjects.
C. Sitters sometimes smiled to show they had a warm personality.
D. People cannot be represented by only one picture.
20. Look at the four squares [■] that indicate where the following sentence could be
added to the passage.
Not as a smirk, but a grimace.
Where would the sentence best fit?
A. First square B. Second square C. Third square D. Fourth square
21. The word ‘cult’ in paragraph 3 is closest in meaning to:
A. mysterious portrait B. controversial topic
C. popular painting D. peerless masterpiece
22. A “withering look” in paragraph 3 is one that is intended to make someone feel
A. delighted B. exasperated C. enchanted D. ashamed
23. The word ‘discrete’ in paragraph 6 is closest in meaning to:
A. separate B. interdependent C. abysmal D. unfathomable
Your answers
14. 15. 16. 17. 18.

19. 20. 21. 22. 23.

Part 3. In the passage below, seven paragraphs have been removed. For questions
24-30, read the passage and choose from the paragraphs A-H the one which fits each
gap. There is ONE extra paragraph which you do not need to use. Write your
answers in the corresponding numbered box provided.

A Clothing Revolution
If you’re bored with your wardrobe and are looking for a new style, you might want to
think about one of the latest trends in fashion, and that is digital clothing. Unlike all other
fashion trends in the past, digital fashion is unique insofar as it does not actually exist. In
FANPAGE TÀI LIỆU TIẾNG ANH NÂNG CAO

this brave new world clothes are made from pixels rather than fabric and customers can
be far more imaginative; in fact, the sky is the limit when it comes to designs in the
digital arena.

24.

However, it is worth bearing in mind that this type of online image manipulation is not a
new idea. People have always paid attention to their profile pictures on a variety of online
platforms that they use in both their private and professional lives. In the same way that
appearance can be influential in real life, our digital appearance plays an important role in
how we are perceived by others, which means people pay attention to it. What is new,
though, is the growing number of advantages of digital over physical fashion.

25.

By only producing a digital version of an item of clothing at first, the costs that are
associated with making samples, having face-to-face meetings and other logistical
concerns can be drastically reduced. The designers can work on each item of clothing
using technology rather than having to produce countless physical items during the
design process. Improvements in technology have meant that seeing a physical garment is
not as essential as it used to be.

26.

This is likely to become more important in the near future as companies rush to meet
consumers’ needs. Already it is clear that fashion brands are adapting their collections to
meet the demands of new lifestyles, many of which are becoming more flexible in terms
of working arrangements. As more people work from home and attitudes towards formal
office attire shift, the fashion industry will need to continue to respond and create new
types of clothing that allow for more comfort and ease of movement as opposed to stiff
formal wear.

27.

They could also start asking for a garment to be copied using different material as a way
of developing a unique personal wardrobe. Many designers are already way ahead of
consumers in this regard and are experimenting with new materials or new ways of using
existing materials. Some current suggestions include clothing that is made entirely from
small lights or metal, or even from plants and flowers. The possibilities for creativity are
FANPAGE TÀI LIỆU TIẾNG ANH NÂNG CAO

endless and customers could end up with the clothes that they have always dreamed of
owning yet could never find.

28.

Despite this being a huge step in the right direction, it is unlikely that attitudes to fashion
and gender will change overnight as the technology is not quite as advanced as people
might think. While many brands have an online fitting room for customers to try out
clothes before they buy them, the body shapes used by this kind of software are fairly
generic and still based on traditional views of male and female bodies that rarely conform
to reality.

29.

As a result, digital clothing remains quite expensive, but this will change as the
technology improves, and the video game sector can shed some light on customer
engagement with digital fashion. For years, video game enthusiasts have been changing
the appearance of their avatars through outfits and weapons, and are clearly happy to pay
for this service, which has caused the costs of avatar clothing to fall as demand has
increased.

30.

The missing paragraphs

A. While some of the new trends have started to emerge, such as the rise in demand for
leisurewear, it is still unclear how the digital fashion market will develop. However, what
is obvious it that it allows for self-expression in a far more nuanced way. For example, a
customer only needs to buy one digital t-shirt, but they could change the image or slogan
on that one t-shirt for many different versions.

B. No one knows whether the cost of digital clothing will come down in a few years’
time. At present, the market appears to be growing, but some designers have suggested
that it is just a passing phase and its popularity could disappear in a flash. As such they
are cautious about investing time or money in something that may be around for just a
few years.
FANPAGE TÀI LIỆU TIẾNG ANH NÂNG CAO

C. Personalisation and creating an individual look contribute to being a key part of the
modern approach to identity, and we can do this by wearing original clothing in both the
real and unreal worlds. The fact that people already do this with imaginary online
characters means that it should come as no surprise that there is a growing desire to do
the same with our online selves too.

D. Costs can be decreased further through the reduction in the time to transport the
products to both bricks-and-mortar-stores and online shopping platforms. This will
enable companies to become more agile, to respond to consumers’ needs and potentially
to create more collections that will start to address individual lifestyle needs instead of
being based around the four seasons.

E. In contrast, the process of having clothing fitted to a specific body shape in a


customer’s photo is much more involved. Currently, this work is done by people and it is
still quite a time-consuming process. Typically, clients upload a photograph and then this
image is digitally dressed by using 3D-modelling software. From start to finish this
process can take up to a whole working day.

F. The most obvious of these is its sustainability. Given that that fashion industry is
responsible for around 10% of greenhouse-gas emissions, there is a clear case for the
sustainability of digital fashion. It can reduce waste considerably, both in terms of
making the physical product, but also by reducing the carbon footprint of the design
process, something that people generally think about far less.

G. Fashion designers have much more freedom with digital garments and can play around
in more creative ways to make clothing more customisable and individual. All this has
become possible because people have been prepared for digital fashion by a social-
media-driven, modern society that is obsessed with photos and online images.

H. By putting the individual at the heart of digital fashion, there is also the opportunity
for a less prescriptive approach to clothing for men and women. For many years the
fashion industry has been criticised for using thin models and causing body-image issues
among young people. Because digital fashion can be so unique, it challenges these ideas.

Your answers
24. 25. 26. 27. 28. 29. 30.

Part 4. The passage below consists of five sections marked A-E. For questions 31-40,
read the passage and do the task that follows. Write your answers (A-E) in the
corresponding numbered boxes provided.
FANPAGE TÀI LIỆU TIẾNG ANH NÂNG CAO

Answering Old and New Questions in Science


Understanding more about the muscles in the human body

A. For a long time, scientists have wondered why some people are better than others at
tolerating the cold, and recent research into why athletes from different parts of the globe
excel in different sports may have shed some light on the issue. Athletes from certain
countries are supremely successful long-distance runners and frequently beat all other
competitors in global competitions such the Olympics, while athletes who perform
exceptionally well in sports that require short bursts of energy often come from
completely different parts of the world. Research now suggests that athletes can be
categorised by muscle types. The human body has fast-twitch and slow-twitch muscles,
the word 'twitch' referring to the movement of the muscles, and, according to scientists,
both muscle types function differently. Fast-twitch muscles contract quickly and are used
for actions such as jumping and sprinting, whereas slow-twitch muscles are the opposite
and are used for slower actions such as walking or jogging.

B. Humans have different combinations of these two muscle types that are set at birth and
cannot be changed through exercise, which accounts for why some people are naturally
good at specific sports compared to others. Athletes who possess more fast-twitch
muscles will always have a competitive edge in sports where speed is a decisive factor,
and this is also true for long-distance running where those who have more slow-twitch
muscles will be more likely to win races. What is interesting is that in both cases the
presence of one of these muscle types is much higher in people who end up becoming
athletes, where the ratio of fast-twitch and slow-twitch muscles is disproportionate
compared to the rest of the population, where it is about 50:50. In essence, then, these
people are born to be athletes.

C. What makes these muscles different is the presence of a specific protein that appears
in the fast-twitch muscles and has therefore been labelled as the 'gene for speed'. The
discovery of this protein has also contributed to ongoing research over the last decade
into the ability of humans to tolerate cold climates. Researchers have observed that this
protein, which is present in fast-twitch muscles, is part of this phenomenon because when
human beings are cold these muscles contract repeatedly and quickly, which explains
why people shiver when they are exposed to lower temperatures. All of these tiny muscle
movements create heat and that is how people warm up their bodies.

D. However, one in five people in the world lack this protein and so do not shiver or
warm themselves with their fast-twitch muscles. Instead, they use their slow-twitch
muscles, which are more efficient and allow people to tolerate colder temperatures for
longer periods of time. The reason why so many people do not have this protein is down
to evolution. As homo sapiens began to move north from Africa over 40,000 years ago
they would have started to settle in colder climates, and this would have caused a change
in their metabolisms in order to manage living in these new conditions. Scientists suggest
FANPAGE TÀI LIỆU TIẾNG ANH NÂNG CAO

that a gene most likely mutated during this transition, causing the loss of this protein, the
evolutionary result of which was that people shivered less in the cold thus preserving
energy.

E. This research into muscles could prove useful for scientists and doctors working in
certain areas of healthcare, one of which being the treatment of muscle-wasting diseases.
These diseases are commonly inherited, and it could be interesting to see how people
with and without this protein respond to different kinds of medical interventions over
time. One study found that there is a connection between this 'gene for speed' and how
long it takes for some muscle-wasting diseases to progress and so, based on this finding,
scientists may be able to develop medication that is more effective for different groups of
patients. Another key area is in that of obesity, which is one of the most concerning
diseases of the modern world affecting millions of people in many cultures. If people
without the specific protein are able to maintain their body temperatures more easily, this
suggest that, unless they follow a healthy diet and exercise regularly, the risk of obesity
and associated problems such as diabetes may be considerably higher for them. It has
been suggested that this information could be beneficial for doctors when advising
patients on how to tackle obesity.

F. Finally, researchers have also noted that people with more slow-than fast-twitch
muscles (in other words, those with the gene mutation and therefore without the protein)
may injure themselves more frequently and easily as they get older. Given that fast-twitch
muscles handle explosive movements such as falling, it follows that older people who do
not have as many fast-twitch muscles may be more susceptible to accidents. The research
that set out to answer the age-old question as to why certain people are able to tolerate the
cold more than others has, like many scientific enquiries, provided innovation in
medicine by incorporating studies into a much newer field, that of sports science.
In which section are the following mentioned?
31. A connection between muscles and how humans combat cold climates has been
found.
32. A change in environment is highly likely to have been the trigger for the genetic
alteration.
33. Studies of talented athletes are helping scientists answer an age-old question.
34. New medicines have come out of combining research fields.
35. The proportion of muscle types in humans remains the same irrespective of physical
training.
36. People lacking the protein might be less able to maintain a healthy lifestyle.
37. Slow-twitch muscles are better for regulating body temperature.
38. Sporting excellence is due to the biological makeup of an individual.
39. A higher proportion of slow-twitch muscles may cause mobility problems for some
people.
40. Treatment could become more precisely targeted as a result of muscle research.
FANPAGE TÀI LIỆU TIẾNG ANH NÂNG CAO

Your answers
31. 32. 33. 34. 35.

36. 37. 38. 39. 40.

Key and explanations:

Part 1.
1. ix
2. iv
3. iii
4. ii
5. vii
6. v
7. vi
8. B
FANPAGE TÀI LIỆU TIẾNG ANH NÂNG CAO

Eric Vermetten, a psychiatrist at the University of Utrecht in the Netherlands, says that
doctors have long known about the potential of smells to act as traumatic reminders, but
the evidence has been largely anecdotal.
9. A
“People don’t remember any more detail or with any more clarity when the memory is
recalled with an odour,” she says.
10. D
People named the smells faster when the picture showed something semantically related
to them, and when that happened, a structure called the hippocampus was strongly
activated.
11. B
“Some clinicians put a strip of vanilla or a strong, pleasant, everyday odorant such as
coffee under their patients’ noses, so that they have this continuous olfactory
stimulation,” says Vermetten. So armed, the patients seem to be better protected against
flashbacks.
12. C
In 2001, for instance, Gil Morrot, of the National Institute for Agronomic Research in
Montpellier, tricked 54 oenology students by secretly colouring a white wine with an
odourless red dye just before they were asked to describe the odours of a range of red and
white wines. The students described the coloured wine using terms typically reserved for
red wines. What’s more, just like experts, they used terms alluding to the wine’s redness
and darkness—visual rather than olfactory qualities.
13. A
That’s hardly surprising, Herz thinks, given how the brain has evolved. “The way I like to
think about it is that emotion and olfaction are essentially the same things,” she says.
“The part of the brain that controls emotion literally grew out of the part of the brain that
controls smell.”
Part 2.
14. C
We smile automatically when someone points a camera at us; it’s a social and cultural
reflex, and what we expect of a picture portrait.
15. A
FANPAGE TÀI LIỆU TIẾNG ANH NÂNG CAO

A smirk is more ambiguous, allowing for several interpretations - condescension,


flirtatiousness, boredom, and so on - and forces a longer and deeper engagement with the
painting from the viewer.
16. B
There is something in it that draws us back to either confirm or revise our interpretation,
always demanding further examination.
17. C
But can this really be the reason? At a time when everybody had bad teeth, would it have
made people particularly shy or embarrassed, or others less attractive?
18. D
Besides, a smile isn't really an expression, it's a response, so cannot easily be maintained
or recorded.
19. A
We have to remember what a portrait was for; what it meant and represented…, Portraits
are for posterity. A smile would detract from their gravitas - these are serious people.
20. B
21. C
22. D
23. A
Part 3.
24. G
In this brave new world clothes are made from pixels rather than fabric and customers
can be far more imaginative; in fact, the sky is the limit when it comes to designs in
the digital arena…Fashion designers have much more freedom with digital garments
and can play around in more creative ways to make clothing more customisable and
individual
25. F
What is new, though, is the growing number of advantages of digital over physical
fashion...The most obvious of these is its sustainability.
26. D
FANPAGE TÀI LIỆU TIẾNG ANH NÂNG CAO

costs that are associated with making samples, having face-to-face meetings and other
logistical concerns can be drastically reduced…Costs can be decreased further
through / more collections that will start to address individual lifestyle needs
27. A
This is likely to become more important in the near future as companies rush to meet
consumers’ needs /create new types of clothing that allow for more comfort and ease of
movement…While some of the new trends have started to emerge, such as the rise in
demand for leisurewear
28. H
They could also start asking for a garment to be copied using different material / The
possibilities for creativity are endless and customers could end up with the clothes that
they have always dreamed of owning yet could never find… By putting the
individual at the heart of digital fashion
29. E
the technology is not quite as advanced as people might think… Currently, this work is
done by people and it is still quite a time-consuming process…
30. C
As a result, digital clothing remains quite expensive / video game sector can shed some
light on customer engagement with digital fashion… imaginary online characters /
growing desire to do the same with online selves too
Part 4.
31. C
The discovery of this protein has also contributed to ongoing research over the last
decade into the ability of humans to tolerate cold climates.
32. D
As homo sapiens began to move north from Africa over 40,000 years ago they would
have started to settle in colder climates, and this would have caused a change in their
metabolisms in order to manage living in these new conditions.
33. A
For a long time, scientists have wondered why some people are better than others at
tolerating the cold, and recent research into why athletes from different parts of the
globe excel in different sports may have shed some light on the issue.
FANPAGE TÀI LIỆU TIẾNG ANH NÂNG CAO

34. F
…like many scientific enquiries, provided innovation in medicine by incorporating
studies into a much newer field, that of sports science.
35. B
Humans have different combinations of these two muscle types that are set at birth and
cannot be changed through exercise…
36. E
If people without the specific protein are able to maintain their body temperatures more
easily, this suggest that, unless they follow a healthy diet and exercise regularly, the risk
of obesity and associated problems such as diabetes may be considerably higher for
them.
37. D
…slow-twitch muscles, which are more efficient and allow people to tolerate colder
temperatures for longer periods of time
38. B
…people who end up becoming athletes, where the ratio of fast-twitch and slow-twitch
muscles is disproportionate compared to the rest of the population, where it is about
50:50. In essence, then, these people are born to be athletes.
39. F
…those with the gene mutation and therefore without the protein may injure themselves
more frequently and easily as they get older. Given that fast-twitch muscles handle
explosive movements such as falling, it follows that older people who do not have as
many fast-twitch muscles may be more susceptible to accidents.
40. E
…scientists may be able to develop medication that is more effective for different
groups of patients.
FANPAGE TÀI LIỆU TIẾNG ANH NÂNG CAO

TEST 18
Part 1. For questions 1-13, read the following passage and do the tasks that follow.

What is meaning?
Why do we respond to words and symbols in the ways we do?
Semantics, in general, is the subdivision of linguistics concerned with meaning.
Semantics attempts the systematic study of the assignment of meanings to minimal
meaning-bearing elements and the combination of these in the production of more
complex meaningful expressions. Elementary word groups may be combined in a
relationship of content, forming thematic groups and semantic and lexical “fields”. For
example, all the means of expressing the concept of joy in a given language constitute the
lexical-semantic field “joy”.
The end, product of education, yours and mine and everybody’s, is the total pattern of
reactions and possible reactions we have inside ourselves. If you did not have within you
FANPAGE TÀI LIỆU TIẾNG ANH NÂNG CAO

at this moment the pattern of reactions that we call “the ability to read”, you would see
here only meaningless black marks on paper. Because of the trained patterns of response,
you are (or are not) stirred to patriotism by martial music, your feelings of reverence are
aroused by symbols of your religion, you listen more respectfully to the health advice of
someone who has “MD” after his name than to that of someone who hasn’t. What I call
here a “pattern of reactions”, then, is the sum total of the ways we act in response to
events, to words, and to symbols.
Our reaction patterns or our semantic habits, are the internal and most important residue
of whatever years of education or miseducation we may have received from our parents’
conduct toward us in childhood as well as their teachings, from the formal education we
may have had, from all the lectures we have listened to, from the radio programs and the
movies and television shows we have experienced, from all the books and newspapers
and comic strips we have read, from the conversations we have had with friends and
associates, and from all our experiences. If, as the result of all these influences that make
us what we are, our semantic habits are reasonably similar to those of most people around
us, we are regarded as “normal,” or perhaps “dull.” If our semantic habits are noticeably
different from those of others, we are regarded as “individualistic” or “original.” or, if the
differences are disapproved of or viewed with alarm, as “crazy.”
Semantics is sometimes defined in dictionaries as “the science of the meaning of
words”— which would not be a bad definition if people didn’t assume that the search for
the meanings of words begins and ends with looking them up in a dictionary. If one stops
to think for a moment, it is clear that to define a word, as a dictionary does, is simply to
explain the word with more words. To be thorough about defining, we should next have
to define the words used in the definition, then define the words used in defining the
words used in the definition and so on. Defining words with more words, in short, gets us
at once into what mathematicians call an “infinite regress”. Alternatively, it can get us
into the kind of run-around we sometimes encounter when we look up “impertinence”
and find it defined as “impudence,” so we look up “impudence” and find it defined as
“impertinence.” Yet—and here we come to another common reaction pattern—people
often act as if words can be explained fully with more words. To a person who asked for
a definition of jazz, Louis Armstrong is said to have replied, “Man. when you got to ask
what it is, you’ll never get to know,” proving himself to be an intuitive semanticist as
well as a great trumpet player.
Semantics, then, seeks the “operational” definition instead of the dictionary Bridgman,
the 1946 Nobel Prize winner and physicist, once wrote, “The true meaning of a term is to
be found by observing what a man does with it, not by what he says about it.” He made
an enormous contribution to science by showing that the meaning of a scientific term lies
in the operations, the things done, that establish its validity, rather than in verbal
definitions. An example of operational definition of the term “weight” of an object,
FANPAGE TÀI LIỆU TIẾNG ANH NÂNG CAO

operationalized to a degree, would be the following: “Weight is the numbers that appear
when that object is placed on a weighing scale”. According to it, when one starts reading
the numbers on the scale, it would more fully make an operational definition. But if
people say—and revolutionists have started uprisings with just this statement “Man is
born free, but everywhere he is in chains!”—what operations could we perform to
demonstrate its accuracy or inaccuracy?
Next, if this suggestion of “operationalism” is pulled outside the physical sciences where
Bridgman applied it, what “operations” are people expected to perform as the result of
both the language they use and the language other people use in communicating to them?
Here is a personnel manager studying an application form. He comes to the words
“Education: Harvard University”, and drops the application form in the wastebasket
(that’s the “operation”) because, as he would say if you asked him, “I don’t like Harvard
men”. This is an instance of “meaning” at work—but it is not a meaning that can be
found in dictionaries.
So far as we know, human beings are the only creatures that have, over and above that
biological equipment which we have in common with other creatures, the additional
capacity for manufacturing symbols and systems of symbols. When we react to a flag, we
are not reacting simply to a piece of cloth, but to the meaning with which it has been
symbolically endowed. When we react to a word, we are not reacting to a set of sounds,
but to the meaning with which that set of sounds has been symbolically endowed. As a
matter of fact, how sound symbolism is processed in our brains has not yet been fully
explained in the field.
Simply put, the key point of semantics lies in, not the words definition, but our own
semantic reactions, which occur when we respond to things the way they “should” be,
rather than to the way they are. If a person was to tell a shockingly obscene story in
Arabic or Hindustani or Swahili before an audience that understood only English, no one
would blush or be angry; the story would be neither shocking nor obscene— indeed, it
would not even be a story. Likewise, the value of a dollar bill is not in the bill, but in our
social agreement to accept it as a symbol of value. If that agreement were to break down
through the collapse of our government, the dollar bill would become only a scrap of
paper. We do not understand a dollar bill by staring at it long and hard. We understand it
by observing how people act with respect to it. We understand it by understanding the
social mechanisms and the loyalties that keep it meaningful. Therefore, semantics
belongs to social studies and potentially underpins the integrity of the social sciences.
Questions 1-8
Do the following statements agree with the information given in the reading passage?
In boxes 1-8, write
FANPAGE TÀI LIỆU TIẾNG ANH NÂNG CAO

YES if the statement agrees with the views of the writer


NO if the statement contradicts the views of the writer
NOT GIVEN if it is impossible to say that the writer thinks about this
1. In linguistics, semantics is the subfield that studies meaning at the most basic level.
2. Printed words only carry meaning to those who have received appropriate ways to
respond.
3. People are judged by how conventional their responses are.
4. Louis Armstrong is an expert on language.
5. Operational definition is more effective than verbal definition in explaining the
meaning of a word.
6. Some statements are incapable of being proved or disproved.
7. Meaning that is unique to an individual is less worthy of study than shared meanings.
8. A story can be entertaining without being understood.
Your answers

1. 2. 3. 4. 5. 6. 7. 8.

Questions 9-13
Complete each sentence with the correct ending, A-H, below.
Write the correct letter, A-H, in boxes 9-13.
9 A comic strip
10 A dictionary
11 Bridgman
12 A story in a language the audience cannot understand
13 A dollar bill
A is meaningless.
B has lasting effects on human behaviors.
C is a symbol that has lost its meaning.
FANPAGE TÀI LIỆU TIẾNG ANH NÂNG CAO

D can be understood only in its social context.


E can provide inadequate explanation of meaning.
F reflects the variability of human behaviors.
G emphasizes the importance of analyzing how words were used.
H suggests that certain types of behaviors carry more meanings than others.

Your answers
9. 10. 11. 12. 13.

Part 2. For questions 14-23, read an extract from an article and choose the answer
A, B, C or D that fits best according to the text. Write your answers in the
corresponding numbered boxes provided.

True Love
Love is a force of nature as unpredictable and potent as a rising storm; when it hits you, it
sweeps you off your feet with such intensity that you can barely right yourself and
withstand its unrelenting power. [■] It draws you with tornadic force towards euphoria.
Just a moment's eye contact is enough to fire your senses into a blaze of passion. [■] One
glance and you know she is the one. [■] Forever more, you will devote yourself entirely
to the otherworldly apparition of perfection that has somehow materialised before you,
selflessly and unconditionally committed to her happiness. [■] Now you are completely
insane. Forgive the intervention, but this love is an animal quite so rare as to be status
critical on the endangered species list. All credit to Hollywood and its brethren for
fabricating and propagating this grand facetious notion, and my apologies for the
stereotypical gender depiction.
However, let's call a spade a spade; this is nothing other than a manifestation of desire,
or, to put it more kindly, attraction; a sensation which, of course, is not to be dismissed,
having both its merits and its part to play in the initial stages of relationship development;
however, the danger when we allow ourselves to be duped by the Hollywood depiction of
love is that the bar of our expectations is set so high as to all but guarantee three faults
and disqualification from the contest. Perhaps it is useful to continue with the storm
analogy a while, though, for what follows this initial burst of passion and impulse may
indeed be the blissful calm and perfection of the eye of the tempest; however, the
FANPAGE TÀI LIỆU TIẾNG ANH NÂNG CAO

moment is but fleeting and trailing the eye is a long tail that is arduous to navigate and
endure.
Running with the storm analogy, the real storm here is not one in a teacup by any means,
though, because when we buy into the media's fairy-tale portrait of love, we set ourselves
off down the road to perpetual misery. If we believe that when we find the one' our
certitude will be so great as to produce an undeniable sense of knowingness and bliss,
then the moment doubts start to creep in or cracks start to emerge in the relationship, we
feel a diminished sense of compatibility with our match and throw it all away before
we've really even given it a chance to work.
Although the bitter taste of reality presented here so far might sound a touch depressing
to some, personally, I find the notion that true love is reached through hard work — and
not simply fate-bestowed —refreshing, and I am far from a blind optimist. After all, it
means we don't have to leave it to chance, which should give us cause for hope. It means
that if we encounter problems in our relationship, we can take comfort in the notion that,
when we are prepared to work through them, there is light at the end of the tunnel. We
are the masters of our fate.
Let me put it this way: if you pilot the love plane for long enough, you will indubitably
encounter storms and turbulence from time to time, and there is no predicting when they
may happen. However, the more often you fly and the more familiar you become with
your crew and the route, the more likely you will be able to navigate through the
problems safely, so the impact of the turbulence should lessen over time. Furthermore,
there will also be, guaranteed, pure heavenly moments when the clouds disperse and you
are soaring as through perfection. These moments of magic make the challenges
worthwhile. On the other hand, if you press the ejector button prematurely, you will
never know such experiences. What's more, you might endanger yourself and you must
surely abandon any other persons on the plane. Note, though, that I said prematurely.
Sadly in life, not everything can work out as planned, but we must at least give it a
fighting chance and be sure we have done our best.
14. What is the writer's main reason for using evocative language in the first
paragraph?
A to convey the intensity of the emotion of love
B to criticise the influence of the media
C to highlight the rarity of true love
D to highlight a form of gender bias
15. Why does the writer use the example of a storm in the second paragraph?
FANPAGE TÀI LIỆU TIẾNG ANH NÂNG CAO

A to show that initial attraction has no relatedness to love


B to highlight the dangers of acting on desire
C to imply that a lasting relationship is full of challenges
D to suggest that love is a temporary emotion
16. What does the writer mean when he refers to a storm in a teacup?
A the effect the media has is very serious
B the media's influence should not be overestimated
C feelings of hurt do not last very long
D we make too much fuss about relationships
17. Why does the writer not find the sober view of love he depicts discouraging?
A because love is a tottery anyway
B because he is optimistic by nature
C because it means we control our own destiny
D because we can learn from failed relationships
18. What does the writer really mean when he talks about ejecting from the plane
prematurely?
A unless you work at a relationship, you will never experience the joys it can bring
B it is more dangerous to leave a relationship than to try to work on it
C wait until things are calm and you have a clear head before making big decisions
D even when a relationship fails, it is not the end of the world
19. What best sums up the writer's thoughts on love and relationships?
A He believes that even healthy relationships have their ups and downs
B He believes that love is not a very realistic concept and people should settle for less
C He is a true romantic who believes in love at first sight
D He is against people abandoning relationships in all but the most extreme
circumstances
20. Look at the four squares [■] that indicate where the following sentence could be
added to the passage.
FANPAGE TÀI LIỆU TIẾNG ANH NÂNG CAO

After all, could there be a purer cause?


Where would the sentence best fit?
A. First square B. Second square C. Third square D. Fourth square
21. The word ‘euphoria’ in paragraph 1 is closest in meaning to:
A. extreme happiness B. profound melancholy
C. mild lethargy D. complete nonsense
22. The word ‘facetious’ in paragraph 1 is closest in meaning to:
A. irrational B. insensitive C. unserious D. impossible
23. The word “certitude” in paragraph 3 refers to a state of being or feeling
A. delighted B. exasperated C. enchanted D. assured
Your answers
14. 15. 16. 17. 18.

19. 20. 21. 22. 23.

Part 3. In the passage below, seven paragraphs have been removed. For questions
24-30, read the passage and choose from the paragraphs A-H the one which fits each
gap. There is ONE extra paragraph which you do not need to use. Write your
answers in the corresponding numbered box provided.

MERGING ART&SCIENCE: A FALSE PREMISE

The current vogue is for believing that art and science should be brought together. This
obsession for showing that art - particularly the visual arts - is similar to science in
content and the creative processes is bemusing. I detect in it an element of social
snobbery - artists are envious of scientists and scientists want to be thought of as artists.

24.

If Watson and Crick had not got the structure of DNA we know that Franklin and Klug
would soon have had it. Indeed simultaneous discovery is a common feature of science. If
one could rerun the history of science and start again it would have a different history but
the end results would be the same: water would be H2O and genes would code for
proteins but the names would be different.
FANPAGE TÀI LIỆU TIẾNG ANH NÂNG CAO

25.

Whatever the feelings of the scientist these are absent from the final understanding of a
process. while art is a personal creation and contains the personal views of the artist. And
since science is a communal process a scientist has to be very aware of what is known
about the problem being investigated. There are strict criteria about lack of contradiction
and, of course, correspondence with reality. Science makes progress, we build on the
work of our current and earlier colleagues. To talk about progress in art makes no sense,
there is change but not progress.

26.

Thus, I cannot understand what is being referred to when there is reference to critical
thinking in art. In what sense can a painting be right or wrong? Anyone can have views
about a painting and engage in art discussions. Non-scientists can thrill to scientific ideas
but to make meaningful comments about them, and I exclude their application to
technology, one actually has to have detailed knowledge; science needs a much greater,
and quite different, intellectual effort.

27.

It is very rare for referees to recommend acceptance without changes. This can be a
complex procedure but in general authors are grateful for the careful reading and
criticism of their paper. Even so we reject about half of all papers we receive. Paintings,
however, are neither revised nor can be shown to be wrong.

28.

The idea of creativity makes scientists want to be thought of as artists and vice versa and
there may well be something similar in all human creativity, but that it is particularly
similar in scientists and artists is without foundation. The similarity between art and
science is even less than that between billiards and rugby, both of which at least use a
ball.

29.

It seems just poetic licence to suggest that this picture did much to convince European
scientists that the great mystery of life might be explained in terms of electrochemical
FANPAGE TÀI LIỆU TIẾNG ANH NÂNG CAO

forces. (Although it may be that Jan Vermeer did indeed discover that more compelling
illusions can be achieved through a kind of optical illusion that makes special use of the
perceptual system inside our brains, rather than through the details that reach our eyes).

30.

Art does not explain, but it broadens our experience in ways that are not clearly
understood. I value it in its own terms, but it has nothing to do with understanding how
the world works. To pretend that it does is to trivialise science and do nothing for art. We
should stop pretending that the two disciplines are similar, and instead rejoice in the very
different ways that they enrich our culture..

The missing paragraphs:


A. What are the criteria used by the director of a gallery and his or her advisers when
selecting for exhibition? Is he or she like the editor of a science journal? No, for there is
nothing in art like the peer review so fundamental to science; there are no art critics, just
art writers. As the editor of a scientific journal, it is extremely rare that my personal view
determines whether or not a paper gets published. My role is to choose a good editorial
board and to know to whom the papers to be reviewed should be sent.

B. Bringing visual artists and scientists together merely makes them feel elevated: it is
not a scientific experience. Although it must be said that science has had a strong
influence on certain artists - in the efforts to imitate nature and thus to develop
perspective or in the area of new technologies - art has contributed virtually nothing to
science.

C. Then of the hundreds of thousands of papers published each year, few have a lifetime
of more than a few years. Most disappear with little if any trace. The original papers, with
very rare exceptions, like those of Einstein, are never part of scientific culture and they
are not for sale. Science, unlike art, is not entertainment.

D. What intrigued me at the opening was how the exhibits were chosen. There is less of
a problem with well established artists such as Lucian Freud, Francis Bacon or Matisse. It
is the very modern works that present the problem.

E. How different from this are all the arts. No Shakespeare - no Hamlet; no Picasso - no
Guernica. Moreover a work of art is capable of many interpretations and has moral
content. There is but one correct scientific explanation for any set of observations and
reliable scientific understanding has no moral or ethical content; that is to say that the
scientist does not allow his own reactions to come into play.
FANPAGE TÀI LIỆU TIẾNG ANH NÂNG CAO

F. The Oxford University art historian Martin Kemp takes a very different view from
mine here. He claims that during the 'Scientific Revolution' some artists were able to play
an active role in the dialogue between seeing and knowing. He gives the fiery emissions
of Joseph Wright's volcanoes painted in the late eighteenth century as an example.
Wright's painting of Vesuvius erupting may be dramatic but it owes nothing to geology.

G. Art is not constrained by reality. It cannot be shown to be wrong. And of all the arts,
painting is the one least related to science as it does not deal with complex ideas or
explanations, is the easiest to appreciate, and the response is often an emotional one.
Ideas in the visual arts come from art critics and historians, not the works themselves.

H. Science is about understanding how the world works, there being only one right
description of any observed phenomenon. Unlike the arts it is a collective endeavour in
which the individual is ultimately irrelevant - geniuses merely speed up discovery.

Your answers
24. 25. 26. 27. 28. 29. 30.

Part 4. The passage below consists of five sections marked A-E. For questions 31-40,
read the passage and do the task that follows. Write your answers (A-E) in the
corresponding numbered boxes provided.

The Future of Interior Design


Five designers talk about the changes they see on the horizon
A Antonio Maroles
I'm optimistic about the future, actually, because no industry gets the luxury of staying
the same forever; otherwise innovation never happens. For me, much of what is changing
is exciting, especially how technology will open up the sector to more people. Interior
design used to only be accessible by a select few, whereas now it's much more egalitarian
and affordable, and this will bring opportunities for designers everywhere. Apps that
allow customers to visualise designs and 'walk through' 3D rooms are going to
revolutionise how we do things, and customers will also be able to recruit designers from
anywhere in the world. All this would have been unthinkable a decade ago when all these
image apps didn't exist, but they've really opened up the possibilities to interact with
clients and increase that level of personalised service. And this gives designers far more
scope to develop their own personal style, too. Years ago, everyone tended to follow the
same trends promoted in magazines and by fabric or wallpaper companies, but that
approach is dead now that anyone can create their own distinctive style, whichever walk
of life they come from, and I'm fully behind it.
B Jeannette Harrison
FANPAGE TÀI LIỆU TIẾNG ANH NÂNG CAO

So many interior designers are panicking that technology is going to put them out of a job
in the next few years, but I find all this gloomy talk quite depressing and quite unlikely in
the foreseeable future. The latest market analysis is not telling us that people want their
homes to go high-tech anytime soon. In fact. quite the opposite seems to be true if you
look carefully at the research. There are real concerns around privacy connected to smart
technology in our homes, so in my view the fear of technology is leading the profession
up a blind alley. What people are looking for, however, is individuality, both at the high
end of the market and the new middle-class customers who can afford to design their
homes more to their own tastes due to a wider range of prices. I am slightly concerned
about the impact on less technologically minded designers because, although it's not
going to take over our jobs, clients will start to expect a basic level of proficiency with
design apps.
C Bobbi Zarkowksi
As far as I'm concerned, there's no point in trying to predict the outcomes of this period
of change with any certainty. There are all sorts of people saying that the end is in sight
for interior designers because consumers can do everything themselves online, but this is
an oversimplification of the situation. There are considerable similarities to what went on
in the travel sector, so we would do well to analyse the impact of the internet to analyse
the impact of the internet for them before wringing our hands in despair. Of course, the
internet democratised travel for the masses, but parts of that meant that the type of
customer changed significantly. Likewise, interior design has always been seen as a
luxury for the wealthy, but not anymore. Whatever the effects over the coming decade
there will be winners and losers, and those who survive will harness the internet to their
advantage instead of railing against it.
D Martina Davis
I'm really hoping that the future heralds a return to creativity after this horrible period,
which I affectionately call the beige stage. I understand that it's important for more
people to have access to interior design because increased access reflects a society that
has improved living conditions for its citizens. But these people should also be given
choice and creativity too, and, unfortunately, what I see around me nowadays is
somewhat of a paradox; people have a desire to express their individuality at home in the
same way they do through fashion, but this self-expression is often identical. The retail
landscape is entirely uninspiring, and I hope the next few years will see a return to
innovation and creativity in both customers and designers. I am worried that this will be
at odds with sustainability, though, which I wholeheartedly believe must be at the heart of
what interior design represents going forward. Cheap furniture from unsustainable wood
sources is not the solution, so we've got to try to merge creativity, sustainability and
affordability, which is going to be quite a challenge.
E Francesca Cheng
My approach towards design has always drawn on traditional imagery that encapsulates
the idea of nature and then applying this inside the home, and this to me is where the
future lies because consumers are much more concerned with sustainability now. Based
on this I can see interior design having a period of re-evaluation that will likely result in
FANPAGE TÀI LIỆU TIẾNG ANH NÂNG CAO

rejecting certain materials or production practices and focusing on innovative ways to be


sustainable and stylish together. I predict a lot more design that tries to enhance
wellbeing, such as using plants for internal walls or LED lights that use a fraction of the
electricity used by other light bulbs. I also think that there'll have to be a lot more
collaboration between interior designers and engineers in order to turn these ideas into
actual objects or materials that function well at the same time as looking attractive.
Which person:
31. suggests that interior designers could upskill themselves to meet customer
expectations?
32. is not convinced that it is worth speculating on the future of interior design?
33. supports the movement of interior design towards a wider customer base?
34. thinks that there is a connection between interior design and social mobility?
35. is disappointed that interior design has become bland for everyone involved?
36. believes technology will completely transform the industry?
37. suggests that interior design will become more environmentally friendly?
38. believes that the future of interior design will be comparable to changes in another
industry?
39. predicts that designers will work alongside other professionals to create the interiors
of the future?
40. says that there is little evidence that technology will replace interior designers?
Your answers
31. 32. 33. 34. 35.

36. 37. 38. 39. 40.

Key and explanations:

Part 1.
1. NO
Semantics, in general, is the subdivision of linguistics concerned with meaning.
Semantics attempts the systematic study of the assignment of meanings to minimal
meaning-bearing elements and the combination of these in the production of more
complex meaningful expressions.
2. YES
If you did not have within you at this moment the pattern of reactions that we call “the
ability to read”, you would see here only meaningless black marks on paper.
3. YES
If, as the result of all these influences that make us what we are, our semantic habits are
reasonably similar to those of most people around us, we are regarded as “normal,” or
FANPAGE TÀI LIỆU TIẾNG ANH NÂNG CAO

perhaps “dull.” If our semantic habits are noticeably different from those of others, we
are regarded as “individualistic” or “original.” or, if the differences are disapproved of or
viewed with alarm, as “crazy.”
4. NOT GIVEN
5. NOT GIVEN
6. YES
But if people say—and revolutionists have started uprisings with just this statement “Man
is born free, but everywhere he is in chains!”—what operations could we perform to
demonstrate.
7. NOT GIVEN
8. NO
If a person was to tell a shockingly obscene story in Arabic or Hindustani or Swahili
before an audience that understood only English, no one would blush or be angry; the
story would be neither shocking nor obscene— indeed, it would not even be a story.
9. B
from all the books and newspapers and comic strips we have read, from the conversations
we have had with friends and associates, and from all our experiences. If, as the result of
all these influences that make us what we are
10. E
If one stops to think for a moment, it is clear that to define a word, as a dictionary does, is
simply to explain the word with more words. To be thorough about defining, we should
next have to define the words used in the definition, then define the words used in
defining the words used in the definition and so on. Defining words with more words, in
short, gets us at once into what mathematicians call an “infinite regress”.
11. G
Semantics, then, seeks the “operational” definition instead of the dictionary Bridgman,
the 1946 Nobel Prize winner and physicist, once wrote, “The true meaning of a term is to
be found by observing what a man does with it, not by what he says about it.” He made
an enormous contribution to science by showing that the meaning of a scientific term lies
in the operations, the things done, that establish its validity, rather than in verbal
definitions.
12. A
FANPAGE TÀI LIỆU TIẾNG ANH NÂNG CAO

If a person was to tell a shockingly obscene story in Arabic or Hindustani or Swahili


before an audience that understood only English, no one would blush or be angry; the
story would be neither shocking nor obscene— indeed, it would not even be a story.
13. D
We understand it by understanding the social mechanisms and the loyalties that keep it
meaningful.
Part 2.
14. B
All credit to.Hollywood and its brethren for fabricating and propagating this grand
facetious notion, and my apologies for the stereotypical gender depiction.
15. C
Perhaps it is useful to continue with the storm analogy a while, though, for what follows
this initial burst of passion and impulse may indeed be the blissful calm and perfection of
the eye of the tempest; however, the moment is but fleeting and trailing the eye is a long
tail that is arduous to navigate and endure
16. A
Running with the storm analogy, the real storm here is not one in a teacup by any means,
though, because when we buy into the media's fairy-tale portrait of love, we set ourselves
off down the road to perpetual misery
17. C
After all, it means we don't have to leave it to chance, which should give us cause for
hope. It means that if we encounter problems in our relationship, we can take comfort in
the notion that, when we are prepared to work through them, there is light at the end of
the tunnel. We are the masters of our fate.
18. A
Furthermore, there will also be, guaranteed, pure heavenly moments when the clouds
disperse and you are soaring as through perfection. These moments of magic make the
challenges worthwhile. On the other hand, if you press the ejector button prematurely,
you will never know such experiences.
19. A
20. D
21. A
FANPAGE TÀI LIỆU TIẾNG ANH NÂNG CAO

22. C
23. D
Part 3.
24. H

“Unlike the arts it is a collective endeavour in which the individual is ultimately


irrelevant - geniuses merely speed up discovery.” connects with “If Watson and Crick
had not got the structure of DNA we know that Franklin and Klug would soon have had
it. Indeed simultaneous discovery is a common feature of science.”

25. E

“If Watson and Crick had not got the structure of DNA we know that Franklin and Klug
would soon have had it.” connects with “How different from this are all the arts. No
Shakespeare - no Hamlet; no Picasso - no Guernica.”

26. G

“Art is not constrained by reality. It cannot be shown to be wrong.” connects with


“Thus, I cannot understand what is being referred to when there is reference to critical
thinking in art. In what sense can a painting be right or wrong? Anyone can have
views about a painting and engage in art discussions.”

27. A

“As the editor of a scientific journal, it is extremely rare that my personal view
determines whether or not a paper gets published. My role is to choose a good editorial
board and to know to whom the papers to be reviewed should be sent.” connects with
“It is very rare for referees to recommend acceptance without changes. This can be a
complex procedure but in general authors are grateful for the careful reading and
criticism of their paper.”

28. C

“It is very rare for referees to recommend acceptance without changes. This can be a
complex procedure but in general authors are grateful for the careful reading and
criticism of their paper.” connects with “Then of the hundreds of thousands of papers
published each year, few have a lifetime of more than a few years.

29. F
FANPAGE TÀI LIỆU TIẾNG ANH NÂNG CAO

“Wright's painting of Vesuvius erupting may be dramatic but it owes nothing to


geology.” connects with “It seems just poetic licence to suggest that this picture did
much to convince European scientists that the great mystery of life might be explained in
terms of electrochemical forces.”

30. B

“Although it must be said that science has had a strong influence on certain artists - in the
efforts to imitate nature and thus to develop perspective or in the area of new
technologies - art has contributed virtually nothing to science.” connects with “I value
it in its own terms, but it has nothing to do with understanding how the world
works.”

Part 4.
31. B
I am slightly concerned / impact on less technologically minded designers / clients /
expect a basic level of proficiency with design apps
32. C
no point in trying to predict the outcomes / with any certainty. / end is in sight /
oversimplification
33. A
what is changing is exciting / open up the sector to more people. / used to only be
accessible by a select few / now more egalitarian
34. D
important for more people to have access / because reflects a society that has improved
living conditions for its citizens
35. D
this horrible period / beige stage / desire to express their individuality / identical. The
retail landscape is entirely uninspiring
36. A
Apps / are going to revolutionise how we do things / image apps / really opened up the
possibilities
37. E
FANPAGE TÀI LIỆU TIẾNG ANH NÂNG CAO

period of re-evaluation / rejecting certain materials / focusing on innovative ways to be


sustainable
38. C
There are considerable similarities to what went on in the travel sector / Likewise,
interior design
39. E
more collaboration between interior designers and engineers
40. B
The latest market analysis is not telling us that people want / high-tech anytime soon. In
fact, quite the opposite seems to be true if you look carefully at the research.

TEST 19
Part 1. For questions 1-13, read the following passage and do the tasks that follow.

Does an IQ test prove creativity?


Everyone has creativity, some a lot more than others. The development of humans, and
possibly the universe, depends on it. Yet creativity is an elusive creature. What do we
mean by it? What is going on in our brains when ideas form? Does it feel the same for
artists and scientists? We asked writers and neuroscientists, pop stars and AI gurus to try
to deconstruct the creative process-and learn how we can all ignite the spark within.
A. In the early 1970s, creativity was still seen as a type of intelligence. But when more
subtle tests of IQ and creative skills were developed in the 1970s, particularly by the
father of creativity testing, Paul Torrance, it became clear that the link was not so simple.
Creative people are intelligent, in terms of IQ tests at least, but only averagely or just
above. While it depends on the discipline, in general beyond a certain level IQ does not
help boost creativity; it is necessary but not sufficient to make someone creative.
FANPAGE TÀI LIỆU TIẾNG ANH NÂNG CAO

B. Because of the difficulty of studying the actual process, most early attempts to study
creativity concentrated on personality. According to creativity specialist Mark Runco of
California State University, Fullerton, the “creative personality” tends to place a high
value on aesthetic qualities and to have broad interests, providing lots of resources to
draw on and knowledge to recombine into novel solutions. “Creatives” have an attraction
to complexity and an ability to handle conflict. They are also usually highly self-
motivated, perhaps even a little obsessive. Less creative people, on the other hand, tend to
become irritated if they cannot immediately fit all the pieces together. They are less
tolerant of confusion. Creativity comes to those who wait, but only to those who are
happy to do so in a bit of a fog.
C. But there may be a price to pay for having a creative personality. For centuries, a link
has been made between creativity and mental illness. Psychiatrist Jamison of Johns
Hopkins University in Baltimore, Maryland, found that established artists are
significantly more likely to have mood disorders. But she also suggests that a change of
mood state might be the key to triggering a creative event, rather than the negative mood
itself. Intelligence can help channel this thought style into great creativity, but when
combined with emotional problems, lateral, divergent or open thinking can lead to mental
illness instead.
D. Jordan Peterson, a psychologist at the University of Toronto, Canada, believes he has
identified a mechanism that could help explain this. He says that the brains of creative
people seem more open to incoming stimuli than less creative types. Our senses are
continuously feeding a mass of information into our brains, which have to block or ignore
most of it to save us from being snowed under. Peterson calls this process latent
inhibition, and argues that people who have less of it, and who have a reasonably high IQ
with a good working memory can juggle more of the data, and so maybe open to more
possibilities and ideas. The downside of extremely low latent inhibition may be a
confused thought style that predisposes people to mental illness. So for Peterson, mental
illness is not a prerequisite for creativity, but it shares some cognitive traits.
E. But what of the creative act itself? One of the first studies of the creative brain at work
was by Colin Martindale, a psychologist from the University of Maine in Orono. Back in
1978, he used a network of scalp electrodes to record an electroencephalogram, a record
of the pattern of brain waves, as people made up stories. Creativity has two stages:
inspiration and elaboration, each characterised by very different states of mind. While
people were dreaming up their stories, he found their brains were surprisingly quiet. The
dominant activity was alpha waves, indicating a very low level of cortical arousal: a
relaxed state, as though the conscious mind was quiet while the brain was making
connections behind the scenes. It’s the same sort of brain activity as in some stages of
sleep, dreaming or rest, which could explain why sleep and relaxation can help people be
creative. However, when these quiet minded people were asked to work on their stories,
FANPAGE TÀI LIỆU TIẾNG ANH NÂNG CAO

the alpha wave activity dropped off and the brain became busier, revealing increased
cortical arousal, more corralling of activity and more organised thinking. Strikingly, it
was the people who showed the biggest difference in brain activity between the
inspiration and development stages who produced the most creative storylines. Nothing
in their background brain activity marked them as creative or uncreative. “It’s as if the
less creative person can’t shift gear,” says Guy Claxton, a psychologist at the University
of Bristol, UK. “Creativity requires different kinds of thinking. Very creative people
move between these states intuitively.” Creativity, it seems, is about mental flexibility:
perhaps not a two-step process, but a toggling between two states. In a later study,
Martindale found that communication between the sides of the brain is also important.
F. Paul Howard-Jones, who works with Claxton at Bristol, believes he has found another
aspect of creativity. He asked people to make up a story based on three words and
scanned their brains using functional magnetic resonance imaging. In one trial, people
were asked not to try too hard and just report the most obvious story suggested by the
words. In another, they were asked to be inventive. He also varied the words so it was
easier or harder to link them. As people tried harder and came up with more creative
tales, there was a lot more activity in a particular prefrontal brain region on the right-hand
side. These regions are probably important in monitoring for conflict, helping us to filter
out many of combining the words and allowing us to pull out just the desirable
connections, Howard-Jones suggests. It shows that there is another side to creativity, he
says. The story-making task, particularly when we are stretched, produces many options
which we have to assess. So part of creativity is a conscious process of evaluating and
analysing ideas. The test also shows that the more we try and are stretched, the more
creative our minds can be.
G. And creativity need not always be a solitary, tortured affair, according to Teresa
Amabile of Harvard Business School. Though there is a slight association between
solitary writing or painting and negative moods or emotional disturbances, scientific
creativity and workplace creativity seem much more likely to occur when people are
positive and buoyant. In a decade-long study of real businesses, to be published soon,
Amabile found that positive moods relate positively to creativity in organisations, and
that the relationship is a simple linear one. Creative thought also improves people’s
moods, her team found, so the process is circular. Time pressures, financial pressures and
hard-earned bonus schemes on the other hand, do not boost workplace creativity: internal
motivation, not coercion, produces the best work.
H. Another often forgotten aspect of creativity is social. Vera John-Steiner of the
University of New Mexico says that to be really creative you need strong social networks
and trusting relationships, not just active neural networks. One vital characteristic of a
highly creative person, she says, is that they have at least one other person in their life
who doesn’t think they are completely nuts
FANPAGE TÀI LIỆU TIẾNG ANH NÂNG CAO

Questions 1-4
Do the following statements agree with the information given in Reading Passage?
In boxes 1-4, write
TRUE if the statement is true
FALSE if the statement is false
NOT GIVEN if the information is not given in the passage
1 High IQ guarantees better creative ability in one person than that who achieves an
average score in an IQ test.
2 In a competitive society, individuals’ language proficiency is more important than
other abilities.
3 A wider range of resources and knowledge can be integrated by more creative people
into bringing about creative approaches.
4 A creative person not necessarily suffers more mental illness.
Your answers
1. 2. 3. 4.

Questions 5-9
Use the information in the passage to match the people (listed A-F) with opinions or
deeds below.
Write the appropriate letters A-F in boxes 5-9 on your answer sheet.
A Jamison
B Jordan Peterson
C Guy Claxton
D Howard-Jone
E Teresa Amabile
F Vera John-Steiner
5 Instead of producing the negative mood, a shift of mood state might be the one
important factor of inducing a creative thinking.
6 Where the more positive moods individuals achieve, there is higher creativity in
organizations.
FANPAGE TÀI LIỆU TIẾNG ANH NÂNG CAO

7 Good interpersonal relationship and trust contribute to a person with more creativity.
8 Creativity demands an ability that can easily change among different kinds of thinking.
9 Certain creative mind can be upgraded if we are put into more practice in assessing and
processing ideas.
Your answers

5. 6. 7. 8. 9.

Questions 10-13
Complete the summary paragraph described below.
In boxes 10-13, write the correct answer with NO MORE THAN THREE WORDS.
But what of the creative act itself? In 1978, Colin Martindale made records of pattern of
brain waves as people made up stories by applying a system constituted of many
10…………………………………… The two phrases of creativity, such as
11………………………………. were found. While people were still planning their
stories, their brains shows little active sign and the mental activity was showed a very
relaxed state as the same sort of brain activity as in sleep, dreaming or rest.
However, experiment proved the signal of 12……………………………………went
down and the brain became busier, revealing increased cortical arousal, when these
people who were in a laidback state were required to produce their stories. Strikingly, it
was found the person who was perceived to have the greatest
13………………………………………..in brain activity between two stages, produced
storylines with highest level of creativity.
Your answers
10. 11. 12. 13

Part 2. For questions 14-23, read an extract from an article and choose the answer
A, B, C or D that fits best according to the text. Write your answers in the
corresponding numbered boxes provided.

GREEN ICEBERGS
Icebergs are massive blocks of ice, irregular in shape; they float with only about 12
percent of their mass above the sea surface. They are formed by glaciers—large rivers of
FANPAGE TÀI LIỆU TIẾNG ANH NÂNG CAO

ice that begin inland in the snows of Greenland, Antarctica, and Alaska—and move
slowly toward the sea. The forward movement, the melting at the base of the glacier
where it meets the ocean, and waves and tidal action cause blocks of ice to break o ff and
float out to sea.
Icebergs are ordinarily blue to white, although they sometimes appear dark or opaque
because they carry gravel and bits of rock. They may change color with changing light
conditions and cloud cover, glowing pink or gold in the morning or evening light, but this
color change is generally related to the low angle of the Sun above the horizon. [■]
However, travelers to Antarctica have repeatedly reported seeing green icebergs in the
Weddell Sea and, more commonly, close to the Amery Ice Shelf in East Antarctica.
[■] One explanation for green icebergs attributes their color to an optical illusion when
blue ice is illuminated by a near-horizon red Sun, but green icebergs stand out among
white and blue icebergs under a great variety of light conditions. [■] Another suggestion
is that the color might be related to ice with high levels of metallic compounds, including
copper and iron. [■] Recent expeditions have taken ice samples from green icebergs and
ice cores—vertical, cylindrical ice samples reaching down to great depths—from the
glacial ice shelves along the Antarctic continent. Analyses of these cores and samples
provide a different solution to the problem.
The ice shelf cores, with a total length of 215 meters (705 feet), were long enough to
penetrate through glacial ice—which is formed from the compaction of snow and
contains air bubbles—and to continue into the clear, bubble-free ice formed from
seawater that freezes onto the bottom of the glacial ice. The properties of this clear sea ice
were very similar to the ice from the green iceberg. The scientists concluded that green
icebergs form when a two-layer block of shelf ice breaks away and capsizes (turns upside
down), exposing the bubble-free shelf ice that was formed from seawater.
A green iceberg that stranded just west of the Amery Ice Shelf showed two distinct
layers: bubbly blue-white ice and bubble-free green ice separated by a one-meter-long ice
layer containing sediments. The green ice portion was textured by seawater erosion.
Where cracks were present, the color was light green because of light scattering; where
no cracks were present, the color was dark green. No air bubbles were present in the
green ice, suggesting that the ice was not formed from the compression of snow but
instead from the freezing of seawater. Large concentrations of single-celled organisms
with green pigments (coloring substances) occur along the edges of the ice shelves in this
region, and the seawater is rich in their decomposing organic material. The green iceberg
did not contain large amounts of particles from these organisms, but the ice had
accumulated dissolved organic matter from the seawater. It appears that unlike salt,
dissolved organic substances are not excluded from the ice in the freezing process.
FANPAGE TÀI LIỆU TIẾNG ANH NÂNG CAO

Analysis shows that the dissolved organic material absorbs enough blue wavelengths
from solar light to make the ice appear green.
Chemical evidence shows that platelets (minute flat portions) of ice form in the water and
then accrete and stick to the bottom of the ice shelf to form a slush (partially melted
snow). The slush is compacted by an unknown mechanism, and solid, bubble-free ice
isformed from water high in soluble organic substances. When an iceberg separates from
the ice shelf and capsizes, the green ice is exposed.
The Amery Ice Shelf appears to be uniquely suited to the production of green icebergs.
Once detached from the ice shelf, these bergs drift in the currents and wind systems
surrounding Antarctica and can be found scattered among Antarctica’s less colorful
icebergs.
14. According to paragraph 1, all of the following are true of icebergs EXCEPT:
A They do not have a regular shape.
B They are formed where glaciers meet the ocean.
C Most of their mass is above the sea surface.
D Waves and tides cause them to break off glaciers.
15. According to paragraph 2, what causes icebergs to sometimes appear dark or opaque?
A A heavy cloud cover
B The presence of gravel or bits of rock
C The low angle of the Sun above the horizon
D The presence of large cracks in their surface
16 According to paragraph 4, how is glacial ice formed?
A By the compaction of snow
B By the freezing of seawater on the bottom of ice shelves
C By breaking away from the ice shelf
D By the capsizing of a two-layer block of shelf ice
17 According to paragraph 4, ice shelf cores helped scientists explain the formation of
green icebergs by showing that
A the ice at the bottom of green icebergs is bubble-free ice formed from frozen seawater.
B bubble-free ice is found at the top of the ice shelf.
FANPAGE TÀI LIỆU TIẾNG ANH NÂNG CAO

C glacial ice is lighter and floats better than sea ice.


D the clear sea ice at the bottom of the ice shelf is similar to ice from a green iceberg.
18 Why does the author mention that “The green ice portion was textured by seawater
erosion”?
A To explain why cracks in the iceberg appeared light green instead of dark green
B To suggest that green ice is more easily eroded by seawater than white ice is
C To support the idea that the green ice had been the bottom layer before capsizing
D To explain how the air bubbles had been removed from the green ice
19 The passage supports which of the following statements about the Amery Ice Shelf?
A The Amery Ice Shelf produces only green icebergs.
B The Amery Ice Shelf produces green icebergs because its ice contains high levels
of metallic compounds such as copper and iron.
C The Amery Ice Shelf produces green icebergs because the seawater is rich in a
particular kind of soluble organic material.
D No green icebergs are found far from the Amery Ice Shelf.
20. Look at the four squares [■] that indicate where the following sentence could be
added to the passage.
Scientists have differed as to whether icebergs appear green as a result of light
conditions or because of something in the ice itself.
Where would the sentence best fit?
A. First square B. Second square C. Third square D. Fourth square
21. Which phrase best explain the word “textured” in the passage?
A having a surface that is not smooth
B covered with slime and mud
C hanging or moving downwards
D wet and soft, usually in a way that is unpleasant
22. The word “excluded” in the passage is closest in meaning to
A kept out
FANPAGE TÀI LIỆU TIẾNG ANH NÂNG CAO

B compressed
C damaged
D gathered together
23. The word “accrete” is closest in meaning to
A. become fused B. move upwards C. shake violently D. melt slowly
Your answers
14. 15. 16. 17. 18.

19. 20. 21. 22. 23.

Part 3. In the passage below, seven paragraphs have been removed. For questions
24-30, read the passage and choose from the paragraphs A-H the one which fits each
gap. There is ONE extra paragraph which you do not need to use. Write your
answers in the corresponding numbered box provided.

Ringing the Alarm for Earth

Peter Raven is a botanist. He knows about photosynthesis, primary productivity and


sustainable growth. He knows that all flesh is grass; that the richest humans and the
hungriest alike depend ultimately on plants for food, fuel, clothing, medicines and shelter,
and that all of these come from the kiss of the sun on warm moist soils, to quicken
growth and ripen grain.

24.

The global population is about to soar from six billion to nine billion in less than a
lifetime. Around 800 million humans are starving, and maybe two billion are
malnourished, while three billion survive on two dollars a day.

25.

By many, Raven means perhaps half to two thirds of all the other species on the planet in
the next 100 years. There could be ten million different kinds of fern, fungus, flowering
plant, arthropod, amphibian, reptile, bird, fish and mammal on Earth. Nobody knows.
People such as Raven, director of the Missouri Botanic Gardens in St Louis, are doing
their best to count and preserve them.
FANPAGE TÀI LIỆU TIẾNG ANH NÂNG CAO

26.

Some of these organisms are now being chased to oblivion by human population growth
at levels that ecosystems cannot sustain.

27.

There are ways of confirming species loss, even if it cannot be established how many
species there were in the first place. Look at the vertebrates and molluscs in fossil
records, Raven says, just for the past sixty-five million years or so. 'You find that the
average life of a species is two to three million years and you get about one species per
million becoming extinct per year in the fossil record. Those particular groups are a small
sample, but they are a real sample,' he says.

28.

That works out at hundreds of creatures per year over the past four centuries, and even
more when humans, rats and other invaders started colonising islands: 2,000 species have
vanished from the Pacific basin alone since the Polynesians got there 1,200 years ago.

29.

There are various wild creatures that get along with humans and follow them everywhere:
cockroaches, fleas, ticks, rats, cats, pigs, cattle, scavenger birds, lusty weeds. These
invade little islands of ancient biodiversity, take over, and see the natives off the
premises. And not just islands: one third of all endangered plants in the continental US
are threatened because of alien invaders, Raven says. In Hawaii, it is 100 percent.

30.

Ecosystems are not static. They change, naturally. They burn, are grazed or browsed, they
regenerate, flood and silt up. But left to themselves, they go on providing services that
humans and other creatures value. A mangrove swamp provides a habitat for shrimps. It
cannot be improved by draining it for a tourist beach, or building a large city on it. Its
natural value would be dissipated. 'An ecosystem itself undamaged is very, very resilient,
and the more simplified it gets, the less resilient. Globally, what we are doing is
simplifying them all, simultaneously, which is a very dangerous large-scale experiment,'
Raven says.
FANPAGE TÀI LIỆU TIẾNG ANH NÂNG CAO

The missing paragraphs:


A Ecosystems, Raven says, can be whatever you like. Hedgerows in Hampshire are an
ecosystem; so are weeds on a railway line at Hammersmith. Savannahs, grasslands,
prairies, rainforests, dry forests, pine forests, uplands, heathlands, downlands, wetlands,
mangrove swamps, estuaries, oxbow lakes and coral reefs are all ecosystems, and they
survive on diversity. The greater the variety of microbes, plants and animals in an
ecosystem, the more resilient it is and the better it works for all, including humans. So it
would not be a good idea to evict at least half of these creatures, especially if nothing is
known about them. But, Raven says, that is what is happening.

B 'Then you can start with the literature in about 1600, when people began to care enough
about organisms to be able to document them well, and for the groups that they were
documenting - birds, mammals, amphibians, reptiles, butterflies and plants - then you can
say, "What was the rate over the past 400 years? It's tens of times or hundreds of times
the level it was before." ,
C Global warming is not going to help, either. What happens to the unique assembly of
plants in the Cape region of Africa as the thermometer rises? They cannot migrate south.
There is no land south of the Cape. So many will perish.

D As he keeps pointing out, the human species is living as if it had more than one planet
to occupy. Forty years ago, he and colleagues tried to calculate the economic cost of
exporting humans to a star system likely to be orbited by habitable planets. They worked
out that it would cost the entire gross economic product of the planet to ship just twelve
people a year to Proxima Centauri or beyond. His message for the planet is, 'Think, look
at the big picture, and think again'.

E But the human population is growing at the rate of about 10,000 an hour, and each
human depends on a hectare or two of land and water for what economists now call
'ecosystem services' - the organisms that ultimately recycle waste and deliver new wealth
to provide oxygen, fresh food, clean water, fuel, new clothes, safe shelter and disposable
income.

F Valuable agricultural land is being poisoned or parched or covered in concrete, soils


eroded, rivers emptied and aquifers drained to feed the swelling numbers. Something has-
got to give, and the first things to go are many of the plants and animals.

G So botanists such as Raven begin with the big picture of sustainable growth and can
calculate to the nearest planet how much land and sea it would take to sustain the
population of the world if everybody lived as comfortably as the Americans, British or
French. The answer is three planets.
FANPAGE TÀI LIỆU TIẾNG ANH NÂNG CAO

H There is another way of checking, Raven says, pioneered by, among others,
sociobiologist and evolutionary psychologist Edward O. Wilson. There is a logarithmic
relationship between the area of habitat and the species that inhabit it. Measure a patch of
forest and count a sample of the species in it. Then compare it with another patch of
forest ten times smaller. The smaller one will have only half the sample species count.
This has been shown in thousands of individual observations, he says. So destroying
forests piecemeal is a way of extinguishing creatures.

Your answers
24. 25. 26. 27. 28. 29. 30.

Part 4. The passage below consists of five sections marked A-E. For questions 31-40,
read the passage and do the task that follows. Write your answers (A-E) in the
corresponding numbered boxes provided.

LIFTING HIGHER EDUCATION TO LOFTIER HEIGHTS?


Academic John Brennan asks whether universities should leave on-the-job training to
employers.

A There is a lot of emphasis nowadays placed on the need for universities and business
groups to get graduates “work ready” through vocational workplace training. This is to be
welcomed but it is also to be questioned – about what it should mean in practice and how
it should be applied. The concept is nothing new. I remember some years back being at a
meeting about higher education and employment, attended by a number of employer
representatives. I recall one employer remarking that of the many thousands of graduates
that he had hired what he really wanted and expected was for each of them to have
changed the nature of the job by the time they had left the role.

B Rather than being concerned with how recruits would fit into existing organisational
arrangements and master existing ways of doing things, here was an employer who
expected graduates to change existing arrangements and ways of working. Who, rather
than focusing on whether graduates had the right kinds of skills and competencies,
acknowledged that he didn’t know what skills and competencies his workers would need
in a few years’ time. The very point of hiring graduates was that he hoped to get people
who would themselves be able to work out what was required and be capable of
delivering it and a bold new future.
FANPAGE TÀI LIỆU TIẾNG ANH NÂNG CAO

C Of course, starting any job requires some work-specific knowledge and capability and
when recruiting staff, graduate or non-graduate, employers have a responsibility to
provide suitable induction and training. The responsibilities of higher education are
different. They are about preparing for work in the long term, in different jobs and, quite
possibly, in different sectors. This is preparation for work in a different world, for work
that is going to require learning over a lifetime, not just the first few weeks of that first
job after graduation. Current initiatives set out a perfectly reasonable set of objectives for
the ways in which higher education can help prepare students for their working lives. But
much will depend on the interpretation and on recognising who – higher education or
employer – is best equipped to contribute what.

D In the rush to focus on “vocational training to improve graduate employability”


academics need to remember that all higher education is vocational in the sense that it
can help shape a graduate’s capacity to succeed in the workplace. In this way higher
education is about life skills, not just job skills. Many years ago, Harold Silver and I
wrote a book entitled A Liberal Vocationalism. It was based on a project we had just
completed on the aims of degree courses in vocational areas such as accountancy,
business and engineering. The book’s title intentionally conveyed the message that even
vocational degree courses were about more than training for a job. There were
assumptions about criticality, transferability of skills, creating and adapting to change
and, above all, an academic credibility.

E Degree courses in subjects such as history and sociology are preparations for
employment as much as vocational degrees such as business and engineering. But the job
details will not be known at the time of study. Indeed, they may not be known until
several years later. Thus, the relevance of higher education to later working life for many
graduates will lie in the realm of generic and transferable skills rather than specific
competencies needed for a first job after graduation. The latter competences are not
unimportant but the graduate’s employer is generally much better equipped than a
university to ensure that the graduate acquires them. Work experience alongside or as
part of study can also help a lot. Higher education is for the long term. Universities,
employers and students should realise that.

In which paragraph is it stated that

31 new proposals require an appropriate level of scrutiny?


32 academic subjects have benefits beyond their syllabuses?
33 business is investing in an unknown quantity in the pursuit of an uncertain goal?
FANPAGE TÀI LIỆU TIẾNG ANH NÂNG CAO

34 responsibility for service provision needs to be correctly allocated?


35 educators need to make sure that they don’t lose sight of an important point?
36 the issues discussed are a recurring theme that is yet to be agreed upon?
37 beliefs about the key topics of a study were alluded to in the heading of a publication?
38 industry is better suited to cover some issues than educational institutions?
39 original thinking is key in finding solutions to future challenges?
40 while obligations vary, they are still present for both parties?
Your answers
31. 32. 33. 34. 35.

36. 37. 38. 39. 40.

Key and explanations:

Part 1.
1. FALSE
Creative people are intelligent, in terms of IQ tests at least, but only averagely or just
above. While it depends on the discipline, in general beyond a certain level IQ does not
help boost creativity; it is necessary but not sufficient to make someone creative.
2. NOT GIVEN
3. TRUE
the “creative personality” tends to place a high value on aesthetic qualities and to have
broad interests, providing lots of resources to draw on and knowledge to recombine
into novel solutions.
4. TRUE
The downside of extremely low latent inhibition may be a confused thought style that
predisposes people to mental illness. So for Peterson, mental illness is not a
prerequisite for creativity, but it shares some cognitive traits.
5. A
Psychiatrist Jamison of Johns Hopkins University in Baltimore, Maryland, found that
established artists are significantly more likely to have mood disorders. But she also
FANPAGE TÀI LIỆU TIẾNG ANH NÂNG CAO

suggests that a change of mood state might be the key to triggering a creative event,
rather than the negative mood itself.
6. E
according to Teresa Amabile of Harvard Business School. Though there is a slight
association between solitary writing or painting and negative moods or emotional
disturbances, scientific creativity and workplace creativity seem much more likely to
occur when people are positive and buoyant.
7. F
Vera John-Steiner of the University of New Mexico says that to be really creative you
need strong social networks and trusting relationships, not just active neural
networks.
8. C
“It’s as if the less creative person can’t shift gear,” says Guy Claxton, a psychologist at
the University of Bristol, UK. “Creativity requires different kinds of thinking. Very
creative people move between these states intuitively.”
9. D
Paul Howard-Jones, who works with Claxton at Bristol, believes he has found another
aspect of creativity. He asked people to make up a story based on three words and
scanned their brains using functional magnetic resonance imaging (…) So part of
creativity is a conscious process of evaluating and analysing ideas. The test also
shows that the more we try and are stretched, the more creative our minds can be.
10. scalpel electrodes
Back in 1978, he used a network of scalp electrodes to record an electroencephalogram,
a record of the pattern of brain waves, as people made up stories.
11. inspiration and elaboration
Creativity has two stages: inspiration and elaboration, each characterised by very
different states of mind.
12. alpha wave activities/alpha waves
The dominant activity was alpha waves, indicating a very low level of cortical arousal
13. difference / differences
Strikingly, it was the people who showed the biggest difference in brain activity between
the inspiration and development stages who produced the most creative storylines.
FANPAGE TÀI LIỆU TIẾNG ANH NÂNG CAO

Part 2.
14. C
The information in choice C is contradicted in sentence 1, which states that icebergs
“float with only about 12 percent of their mass above the sea surface.” The information
given in the other choices is stated in the paragraph.
15. B
The information in choice B is taken directly from sentence 1 in the paragraph, which
states that icebergs “sometimes appear dark or opaque because they carry gravel and bits
of rock.” Choice A is incorrect because, as sentence 2 states, cloud cover may result in
“pink or gold” colors, not dark colors. Choice C is incorrect because “the low angle of the
Sun above the horizon” is discussed as a possible cause of pink or gold colors. Choice D
is incorrect because the issue of large cracks in icebergs is not discussed in paragraph 2
16. A
The correct answer is choice A. Sentence 1 in the paragraph discusses “glacial ice—
which is formed from the compaction of snow.” Choice B is incorrect because the
information given describes sea ice, a different type of ice. Choice C is incorrect because
the information given describes the first step in the formation of green icebergs. Choice D
is incorrect because the information given describes the second step in the formation of
green icebergs.
17. D
The correct answer is choice D. Sentence 2 in the paragraph states that clear sea ice is
“very similar” to the ice from green icebergs. Choices A, B, and C do not answer the
question asked. Choice A is also incorrect because it mistakenly identifies green icebergs
as having frozen seawater at the bottom, whereas sentence 1 in the paragraph says that
frozen seawater is found on the bottom of glacial ice. Choice B is incorrect because the
information given is the opposite of what is stated in the passage, which is that bubble-
free ice is formed and found on the bottom of shelf ice. Choice C is incorrect because the
information given is not discussed in the passage at all.
18. C
This is a Rhetorical Purpose question. It tests why the author mentions that “The green
ice portion was textured by seawater erosion.” This sentence is highlighted in the
passage. The correct answer is choice C. The highlighted sentence is evidence that the
green ice part of the iceberg was once under water. The fact that this green ice is no
longer under water but is now exposed to air is evidence that the green icebergs are
formed from pieces of the ice shelf that have broken off and turned upside down. Choice
A is incorrect because the information given, while factual according to the passage, does
FANPAGE TÀI LIỆU TIẾNG ANH NÂNG CAO

not explain why the author includes the information that the green ice portion was
textured by seawater. Choice B is incorrect because there is no comparison made between
the erosion of green ice and white ice in the paragraph. Choice D is incorrect because,
while sentences 1 and 4 in the paragraph state that green ice has no bubbles, there is no
information in the paragraph indicating that green ice initially has bubbles and that they
are removed.
19. C
This is an Inference question asking for an inference that can be supported by the
passage. The correct answer is choice C. Sentences 5, 6, and 7 in paragraph 5 support this
information by indicating that the seawater around these icebergs contains the
decomposing material of green-pigmented organisms. This decomposing material
dissolves in seawater, which then freezes as part of the iceberg. The information in choice
A is incorrect because paragraph 7 says that the Amery Ice Shelf is well suited to the
production of green icebergs. This does not mean that the Amery Ice Shelf produces only
green icebergs. The information in choice B is incorrect because copper and iron are
mentioned in paragraph 3 only as possible color sources in green icebergs. The last
sentence in paragraph 3 states that a source other than copper and iron was found. The
information in choice D is incorrect because the passage gives no indication of where all
green icebergs are located. Paragraph 2 mentions the Weddell Sea in Antarctica, and
paragraph 7 states that green icebergs “drift” around Antarctica. Therefore green icebergs
can be found far from the Amery Ice Shelf.
20. D
Choice (B) is correct because the sentence provided introduces two possible explanations
for the color of green icebergs. Paragraph 3 is the first place in the passage where
explanations are offered for the color of green icebergs. The beginning of paragraph 3 is
the only appropriate place to introduce these possible explanations. Choice (A) is
incorrect because green icebergs are mentioned for the first time in the last sentence in
paragraph 2. It does not make sense to insert the given sentence, which introduces
explanations for the color of green icebergs, before the first mention of green icebergs.
Choice (C) is incorrect because its position is between the detailed discussions of the two
explanations introduced in the given sentence. The given sentence introduces the two
explanations; therefore it must come before the discussions. Choice (D) is incorrect
because its position is after the detailed discussions of the two explanations introduced in
the given sentence. The given sentence introduces the two explanations; therefore it must
come before the discussions.
21. A
22. A
FANPAGE TÀI LIỆU TIẾNG ANH NÂNG CAO

23. A
Part 3.
24. G
The first paragraph introduces Peter Raven and describes the kind of knowledge botanists
like him have. Paragraph G continues this theme by describing what conclusions Raven
and other botanists can come to using their knowledge. Verbal links are 'Peter Raven is a
botanist' and 'sustainable growth' in the paragraph before the gap and 'botanists such as
Raven' and 'the big picture of sustainable growth' in paragraph G.
25. F
There is a link between 'Something has got to give, and the first things to go are many of
the plants and animals' in paragraph F and' By many, Raven means perhaps half to two
thirds of all the other species on the planet' in the paragraph after the gap.
26. E
There is a link between 'the human population', 'each human depends on ... "ecosystem
services" , and 'the organisms that ultimately recycle waste and deliver new wealth to
provide oxygen, fresh food, clean water, fuel, new clothes, safe shelter and disposable
income' in paragraph E and the whole of the paragraph after the gap.
27. A
There is a link between the ecosystems mentioned in the paragraph before the gap and the
description of what an ecosystem is in paragraph A. Also, there is a link between 'The
greater the variety of microbes, plants and animals in an ecosystem, the more resilient it
is and the better it works ... So it would not be a good idea to evict at least half of these
creatures ... But, Raven says, that is what is happening' in paragraph A and the reference
to 'species loss' in the paragraph after the gap.
28. B
There is a link between the description of 'species loss' in the paragraph before the gap,
the description of species loss in paragraph B and the calculation of the number of
species lost per year 'over the last four centuries' in the paragraph after the gap. There is a
further link between the question posed in paragraph B 'What was the rate over the past
400 years?' and the answer in the paragraph after the gap: 'hundreds of creatures per year
over the past four centuries.
29. H
There is a link between 'There is another way of checking' (which refers to another way
of checking how many species have been lost) and 'There is a way of confirming species
FANPAGE TÀI LIỆU TIẾNG ANH NÂNG CAO

loss' (three paragraphs earlier). This idea is continued in the paragraph before the gap.
There is also a link between '2,000 species have vanished from the Pacific basin' in the
paragraph before the gap and 'So destroying forests piecemeal is a way of extinguishing
creatures' in paragraph H.
30. C
There is a link between 'one third oral endangered plants in the continental US are
threatened because of alien invaders' in the paragraph before the gap and 'Global
warming is not going to help, either' in paragraph C.
Part 4.
31 A

There is a lot of emphasis nowadays placed on the need for universities and business
groups to get graduates “work ready” through vocational workplace training. This is to be
welcomed but it is also to be questioned – about what it should mean in practice and
how it should be applied.

32 D

The book’s title intentionally conveyed the message that even vocational degree courses
were about more than training for a job. There were assumptions about criticality,
transferability of skills, creating and adapting to change and, above all, an academic
credibility.

33 B

Rather than being concerned with how recruits would fit into existing organisational
arrangements and master existing ways of doing things, here was an employer who
expected graduates to change existing arrangements and ways of working. Who, rather
than focusing on whether graduates had the right kinds of skills and competencies,
acknowledged that he didn’t know what skills and competencies his workers would
need in a few years’ time.

34 C

But much will depend on the interpretation and on recognising who – higher education or
employer – is best equipped to contribute what.

35 D
FANPAGE TÀI LIỆU TIẾNG ANH NÂNG CAO

In the rush to focus on “vocational training to improve graduate employability”


academics need to remember that all higher education is vocational in the sense that it
can help shape a graduate’s capacity to succeed in the workplace

36 A

The concept is nothing new. I remember some years back being at a meeting about
higher education and employment, attended by a number of employer representatives. I
recall one employer remarking that of the many thousands of graduates that he had hired
what he really wanted and expected was for each of them to have changed the nature of
the job by the time they had left the role.

37 D

Many years ago, Harold Silver and I wrote a book entitled A Liberal Vocationalism. It
was based on a project we had just completed on the aims of degree courses in
vocational areas such as accountancy, business and engineering.

38 E

The latter competences are not unimportant but the graduate’s employer is generally
much better equipped than a university to ensure that the graduate acquires them.

39 B

The very point of hiring graduates was that he hoped to get people who would themselves
be able to work out what was required and be capable of delivering it and a bold new
future.

40 C

Of course, starting any job requires some work-specific knowledge and capability and
when recruiting staff, graduate or non-graduate, employers have a responsibility to
provide suitable induction and training. The responsibilities of higher education are
different. They are about preparing for work in the long term, in different jobs and,
quite possibly, in different sectors.
FANPAGE TÀI LIỆU TIẾNG ANH NÂNG CAO

TEST 20
Part 1. For questions 1-13, read the following passage and do the tasks that follow.

E-training
A E-learning is the unifying term to describe the fields of online learning, web-based
training, and technology-delivered instruction, which can be a great benefit to corporate
e-learning. IBM, for instance, claims that the institution of its e-training program, Basic
Blue, whose purpose is to train new managers, saved the company in the range of $200
million in 1999. Cutting the travel expenses required to bring employees and instructors
to a central classroom accounts for the lion’s share of the savings. With an online course,
employees can learn from any Internet-connected PC, anywhere in the world. Ernst and
Young reduced training costs by 35 percent while improving consistency and scalability.
B In addition to generally positive economic benefits, other advantages such as
convenience, standardized delivery, self-paced learning, and variety of available content,
have made e-learning a high priority for many corporations. E-learning is widely believed
to offer flexible “any time, any place” learning. The claim for “any place” is valid in
principle and is a great development. Many people can engage with rich learning
materials that simply were not possible in a paper or broadcast distance learning era. For
teaching specific information and skills, e-training holds great promise. It can be
especially effective at helping employees prepare for IT certification programs. E-
learning also seems to effectively address topics such as sexual harassment education,5
safety training and management training — all areas where a clear set of objectives
can be identified. Ultimately, training experts recommend a “blended” approach that
combines both online and in-person training as the instruction requires. E-learning is not
an end-all solution. But if it helps decrease costs and windowless classrooms filled with
snoring students, it definitely has its advantages.
C Much of the discussion about implementing e-learning has focused on the technology,
but as Driscoll and others have reminded us, e-learning is not just about the technology,
FANPAGE TÀI LIỆU TIẾNG ANH NÂNG CAO

but also many human factors. As any capable manager knows, teaching employees new
skills is critical to a smoothly run business. Having said that, however, the traditional
route of classroom instruction runs the risk of being expensive, slow and, oftentimes,
ineffective. Perhaps the classroom’s greatest disadvantage is the fact that it takes
employees out of their jobs. Every minute an employee is sitting in a classroom training
session is a minute they’re not out on the floor working. It now looks as if there is a way
to circumvent these traditional training drawbacks. E-training promises more effective
teaching techniques by integrating audio, video, animation, text and interactive materials
with the intent of teaching each student at his or her own pace. In addition to higher
performance results, there are other immediate benefits to students such as increased time
on task, higher levels of motivation, and reduced test anxiety for many learners. A
California State University Northridge study reported that e-learners performed 20
percent better than traditional learners. Nelson reported a significant difference between
the mean grades of 406 university students earned in traditional and distance education
classes, where the distance learners outperformed the traditional learners.
D On the other hand, nobody said E-training technology would be cheap. E-training
service providers, on the average, charge from $10,000 to $60,000 to develop one hour of
online instruction. This price varies depending on the complexity of the training topic and
the media used. HTML pages are a little cheaper to develop while streaming-video
(presentations or flash animations cost more. Course content is just the starting place for
cost. A complete e-learning solution also includes the technology platform (the
computers, applications and network connections that are used to deliver the courses).
This technology platform, known as a learning management system (LMS), can either be
installed onsite or outsourced. Add to that cost the necessary investments in network
bandwidth to deliver multimedia courses, and you’re left holding one heck of a bill. For
the LMS infrastructure and a dozen or so online courses, costs can top $500,000 in the
first year. These kinds of costs mean that custom e-training is, for the time being, an
option only for large organizations. For those companies that have a large enough staff,
the e-training concept pays for itself. Aware of this fact, large companies are investing
heavily in online training. Today, over half of the 400-plus courses that Rockwell Collins
offers are delivered instantly to its clients in an e-learning format, a change that has
reduced its annual training costs by 40%. Many other success stories exist.
E E-learning isn’t expected to replace the classroom entirely. For one thing, bandwidth
limitations are still an issue in presenting multimedia over the Internet. Furthermore, e-
training isn’t suited to every mode of instruction or topic. For instance, it’s rather
ineffective imparting cultural values or building teams. If your company has a unique
corporate culture it would be difficult to convey that to first-time employees through a
computer monitor. Group training sessions are more ideal for these purposes. In addition,
there is a perceived loss of research time because of the work involved in developing and
FANPAGE TÀI LIỆU TIẾNG ANH NÂNG CAO

teaching online classes. Professor Wallin estimated that it required between 500 and
1,000 person-hours, that is, Wallin-hours, to keep the course at the appropriate level of
currency and usefulness. (Distance learning instructors often need technical skills, no
matter how advanced the courseware system.) That amounts to between a quarter and
half of a person-year. Finally, teaching materials require computer literacy and access to
equipment. Any e-Learning system involves basic equipment and a minimum level of
computer knowledge in order to perform the tasks required by the system. A student that
does not possess these skills, or have access to these tools, cannot succeed in an e-
Learning program.
F While few people debate the obvious advantages of e-learning, systematic research is
needed to confirm that learners are actually acquiring and using the skills that are being
taught online, and that e-learning is the best way to achieve the outcomes in a corporate
environment. Nowadays, a go-between style of the Blended learning, which refers to a
mixing of different learning environments, is gaining popularity. It combines traditional
face-to-face classroom methods with more modem computer-mediated activities.
According to its proponents, the strategy creates a more integrated approach for both
instructors and learners. Formerly, technology-based materials played a supporting role to
face-to-face instruction. Through a blended learning approach, technology will be more
important
Questions 1-6
The reading passage has seven paragraphs, A-F
Choose the correct heading for paragraphs A-F from the list below. Write the
correct number, i-xi in boxes 1-6.

List of Headings

i overview of the benefits for the application of E-training

ii IBM’s successful choice of training

iii Future direction and a new style of teaching

iv learners achievement and advanced teaching materials

v limitations when E-training compares with traditional class

vi multimedia over the Internet can be a solution


FANPAGE TÀI LIỆU TIẾNG ANH NÂNG CAO

vii technology can be a huge financial burden

viii the distance learners outperformed the traditional university learners in worldwide

ix other advantages besides economic consideration

x Training offered to help people learn using computers

1. Paragraph A
2. Paragraph B
3. Paragraph C
4. Paragraph D
5. Paragraph E
6. Paragraph F
Your answers

1. 2. 3. 4. 5. 6.

Questions 7-10
The reading Passage has seven paragraphs A-F.
Which paragraph contains the following information?
Write the correct letter A-F, in boxes 7-10.
7. Projected Basic Blue in IBM achieved a great success.
8. E-learning wins as a priority for many corporations as its flexibility.
9. The combination of the traditional and e-training environments may prevail.
10. Example of a fast electronic delivery for a company’s products to its customers.
Your answers

7. 8. 9. 10.
FANPAGE TÀI LIỆU TIẾNG ANH NÂNG CAO

Questions 11-13
Choose Three correct letters, among A-E
Write your answers in boxes 11-13.
A Technical facilities are hardly obtained.
B Presenting multimedia over the Internet is restricted due to the bandwidth limit.
C It is ineffective imparting a unique corporate value to fresh employees.
D Employees need block a long time leaving their position attending training.
E More preparation time is needed to keep the course at the suitable level.
Your answers

11. 12. 13.

Part 2. For questions 14-23, read an extract from an article and choose the answer
A, B, C or D that fits best according to the text. Write your answers in the
corresponding numbered boxes provided.
Line We are told that it is not within the “province of woman,” to discuss the
subject of slavery; that it is a “political question,” and we are “stepping out
of our sphere,” when we take part in its discussion. It is not true that it is
merely a political question, it is likewise a question of justice, of humanity,
5 of morality, of religion; a question which, while it involves considerations
of immense importance to the welfare and prosperity of our country, enters
deeply into the home-concerns, the every-day feelings of millions of our
fellow beings. Whether the laborer shall receive the reward of his labor, or
be driven daily to unrequited toil - whether he shall walk erect in the
10 dignity of conscious manhood, or be reckoned among the beasts which
perish - whether his bones and sinews shall be his own, or another's -
whether his child shall receive the protection of its natural guardian, or be
ranked among the live-stock of the estate, to be disposed of as the caprice
or interest of the master may dictate - ... these considerations are all
15 involved in the question of liberty or slavery.

And is a subject comprehending interests of such magnitude, merely a


“political question,” and one in which woman “can take no part without
losing something of the modesty and gentleness which are her most
FANPAGE TÀI LIỆU TIẾNG ANH NÂNG CAO

appropriate ornaments”? May not the “ornament of a meek and quiet


20 spirit” exist with an upright mind and enlightened intellect, and must
woman necessarily be less gentle because her heart is open to the claims of
humanity, or less modest because she feels for the degradation of her
enslaved sisters, and would stretch forth her hand for their rescue?

By the Constitution of the United States, the whole physical power of the
25 North is pledged for the suppression of domestic insurrections, and should
the slaves, maddened by oppression, endeavor to shake off the yoke of the
taskmaster, the men of the North are bound to make common cause with the
tyrant, and put down, at the point of the bayonet, every effort on the part of
the slave, for the attainment of his freedom. And when the father, husband,
30 son, and brother shall have left their homes to mingle in the unholy warfare,
“to become the executioners of their brethren, or to fall themselves by their
hands,” will the mother, wife, daughter, and sister feel that they have no
interest in this subject? Will it be easy to convince them that it is no concern
of theirs, that their homes are rendered desolate, andtheir habitation the
35 abodes of wretchedness? Surely this consideration iss of itself sufficient to
arouse the slumbering energies of woman, for the overthrow of a system
which thus threatens to lay in ruins the fabric of her domestic happiness; and
she will not be deterred from the performance of her duty herself, her
family, and her country, by the cry of political question.

40 But admitting it to be a political question, have we no interest in the welfare


of our country? May we not permit a thought to stray beyond the narrow
limits of our own family circle, and of the present hour? May we breathe a
sigh over the miseries of our countrymen, nor utter a word of
remonstrance against the unjust laws that are crushing them to the Earth?
45 Must we witness “the headlong rage heedless folly, with which our nation is
rushing to destruction, and not seek to arrest its downward course? Shall we
silently behold the land which we all love with all the heart-warm affection
of children, rendered a hissing and a reproach throughout the world, by this
system which is already tolling the death-bell of her decease among
50 the nations? No: the events of the last two years have cast their dark
shadows before, overclouding the bright prospects of the future, and
shrouding the destinies of our country in more than midnight gloom, and
we cannot remain inactive. Our country is as dear to us as the proudest
statesman. and the more closely our hearts cling to “our altars and our
55 homes,” the more fervent are our aspirations that every inhabitants of our
land may be protected in his fireside enjoyments by just and equal laws; that
FANPAGE TÀI LIỆU TIẾNG ANH NÂNG CAO

the foot of the tyrant may longer invade the domestic sanctuary nor his hand
tear asunder those whom God himself had united by the most holy ties. Let
our course, then, still be onward!

14. The author’s main purpose in the passage is to

A. accuse fellow abolitionists of overlooking the contribution that women have


made to the movement.

B. argue that the causes of abolition and women’s rights are continuations of the
spirit of the American revolution.

C. make the case that women’s tights are meaningless while slavery exists.

D. encourage women to see their participation in the abolitionist cause as just and
important.

15. Which statement provides the best description of a technique that Smith uses
throughout the passage to advance her main point?

A. She presents claims in the form of rhetorical questions that mostly have implicit
negative answers.

B. She criticizes her opponents by quoting self-contradictory remarks they have


made.

C. She illustrates each of her central ideas with an emotionally powerful anecdote.

D. She emphasizes the reasonableness of her views by presenting them as though


they are universally held.

16. How does the author develop her argument about slavery as a “political
question” (line 2) over the course of the passage?

A. She claims the designation of an outdated one and then offers alternative
definitions.

B. She dismisses the resignation as too narrow but then demonstrates its relevance
to her audience.
FANPAGE TÀI LIỆU TIẾNG ANH NÂNG CAO

C. She contends that the designation has become trite and then invites her audience
to revitalize it.

D. She describes the meaning of the designation has for men and then challenges
women to embrace it.

17. Which choice best summarizes the first paragraph?

A. The author explains a conventional viewpoint and presents evidence supporting


it.

B. The author rejects a claim and elaborates on her reasons for doing so.

C. The author introduces her subject and provides historical background for
understanding it.

D. The author identifies a problem and proposes steps to remedy it.

18. In the passage, the authors argues that it is possible for women to engage in
which activity?

A. Acting according to humanitarian principles while preserving their femininity

B. Adhering to personal morality while being politically neutral

C. Contributing to their family’s financial security while meeting social


expectation

D. Resisting calls for war while still opposing slavery

19. Which choice provides the best evidence for the answer to the previous
question?

A. Lines 19 – 23 (“May … rescue”)

B. Lines 29 – 33 (“And when … subject”)

C. Lines 35 – 37 (“Surely … happiness”)

D. Lines 53 – 56 (“Our … laws”)


FANPAGE TÀI LIỆU TIẾNG ANH NÂNG CAO

20. According to the author, the U.S. Constitution requires which action on the part
of the Northern free states if slaves were to revolt?

A. The Northern states would have to sever ties with the slave states.

B. The Northern states would have to give shelter to refugees from the slave states.

C. The Northern states would have to help the slave states fight the slaves’
rebellion.

D. The Northern states would have to provide financial assistance to the rebelling
slaves.

21. In context, what is the main effect of the author’s use of the word “tyrant” in
lines and ?

A. It identifies a specific individual as oppressive.

B. It highlights the threat of aggression from abroad.

C. It critiques the limited roles of women in antislavery movements.

D. It emphasizes the unjustness of slavery.

22. As used in line, “slumbering” most nearly means

A. lethargic

B. drowsy

C. dormant

D. unconscious

23. In the passage, the author most strongly suggests that slavery affects the United
States by

A. lowering the country’s reputation in the international community

B. leading many women to disavow their allegiance to the country

C. causing violent conflicts in many areas of the country


FANPAGE TÀI LIỆU TIẾNG ANH NÂNG CAO

D. weakening the authority of the country’s government


Your answers
14. 15. 16. 17. 18.

19. 20. 21. 22. 23.

Part 3. In the passage below, seven paragraphs have been removed. For questions
24-30, read the passage and choose from the paragraphs A-H the one which fits each
gap. There is ONE extra paragraph which you do not need to use. Write your
answers in the corresponding numbered box provided.

If these bones could talk ...

To a palaeoanthropologist, the past is an open book, but one that fails to tell the whole
story. The covers are missing. The first chapters may never be found. There are hardly
any pages, and most are so smeared and crumpled, so foxed and faded, that the text could
mean almost anything. The cast of characters is confusing and narrative thread anybody's
guess. Is it a detective story, a cliffhanger, or a romance? Can there be a happy ending?

24.

Homo floresiensis was the mysterious survivor unearthed from a cave on the island of
Flores in Indonesia: a pygmy descendant, perhaps, of Homo erectus, perhaps even
connected to an earlier human species, but with this special feature: the bones were only
18,000 years old. So Homo sapiens, Homo erectus, Homo neanderthalis and Homo
floresiensis must have all shared the planet at the same time, tantalisingly recently: within
the last 100,000 years perhaps. Now only Homo sapiens survives.

25.

Stringer, 57, is head of human origins at the Natural History Museum in London. One of
palaeoanthropology's big players, he has spent his career in pursuit of Homo
neanderthalis and is also one of the great proselytisers of the Out-of-Africa theory, the
one that says the human story begins on just one continent. Homo floresiensis, however,
astonished him.

26.
FANPAGE TÀI LIỆU TIẾNG ANH NÂNG CAO

'Nature is constantly experimenting. I think a lot of people thought that humans were
somehow different; that we had this all embracing culture and this unifying adaptation,
which meant that human evolution progressed in a somewhat different way, because of
our technology and the way we probably vainly think we are partly controlling the world
now. So people project backwards and think that humans are somehow special. The
evidence shows us that our evolution was as complex and as undirected, I suppose, as
that of any other species we have studied.'

27.

Modern humans probably popped up within the last 200,000 years, but the things that
make modern humans so distinctive in the fossil record - symbolic art, pottery and
jewellery - bloomed only about 50,000 years ago. Nobody in the world of
palaeoanthropology considers modern humanity to be the flower of creation, either. A
temporary bloom, maybe.

28.

Genetic evidence suggests humans may have come close to extinction a number of times
in the past. Modern humans shared the Middle East with Homo neanderthalis 120,000
years ago, and as Cro-Magnons became the sole tenants of Europe 30,000 years ago, a
terrain held successfully by the Neanderthals for more than 100,000 years. Did they
compete? Did they co-exist? Did they trade, or cohabit?

29.

'I still tend to the view that the primary message would have been: different. They would
have had a different body language, a completely different way of communication; they
would have had different behaviours.'

30.

He and his co-author Peter Andrews - a former head of human origins at the Natural
History Museum, and an expert on the early part of the human story - tried to tell the
story of human evolution not just through time, but through its context, Stringer says:
how you set about excavating a site, what a piece of tooth or jaw can tell you about
ancient human behaviour. In that, the title of the book means what it says: complete.
FANPAGE TÀI LIỆU TIẾNG ANH NÂNG CAO

The missing paragraphs:


A It's humbling, Stringer says. ‘We shouldn't see ourselves as the summit of the
perfection of whatever evolution is trying to achieve. We seem to be very successful at
the moment in terms of our numbers but, looking at it on a geological timescale, how
successful will we look in 50,000 years, which is a very short time, geologically
speaking?'

B 'Neanderthals were certainly human and evolved as us in their own way, but they were
different. They had several hundred thousand years of evolving their own anatomy and
behaviour. But when these people met in Europe would they have seen each other as
people? Or as someone different?' he says.

C What stories could these bones tell? And who could have dreamed, before their
discovery that some tree-climbing, pygmy-elephant-hunting human candidate could have
survived on a tropical island while Homo sapiens moved into the Fertile Crescent,
preparing to invent agriculture, civilisation and global terrorism?

D He thinks the Neanderthals perished at a moment of maximum stress in the stop-go,


hot-cold pattern of climate during the last ice age. Though they left their mark in the
Pyrenees, they never got to Britain at all. But then the human occupation of Britain itself
is a bit of a riddle. There is evidence of it, most of it indirect, of little pulses of human
occupation, and then a gap of 100,000 years when no humans appeared to have visited
Britain at all. Modern humans finally moved in and stayed only 12,000 years ago.

E These people were capable of making tools and butchering large beasts like rhinos.
They may not have killed these beasts themselves - they were, after all, dangerous
animals - but even if they were just scavenging, it must have taken some degree of
cooperation and organisation to have driven off the lions or wolves, and secured the
carcass for themselves.

F There is a story-so-far, but that potted version of events is forever being revised, and
nobody knows that better than Chris Stringer, one of the authors of a book published
today called The Complete World of Human Evolution. Complete? Stringer spent eight
years on the text. Then, late last year, he had to sit down in one night and compose an
entirely new chapter to incorporate the discovery of Homo floresiensis, also known as the
Hobbit.

G Here is the orthodm,'Y, pieced together over a century or more by Darwin's disciples:
primate creatures with a capacity for walking upright emerged perhaps twenty million
years ago. From these emerged the ancestors of all gorillas, all chimpanzees and all
humans. There is no line of evolution: think, instead, of foliage, and the surviving
humans and two species of chimpanzees are just nearby buds at the ends of twigs dose
together on the tree oflife.
FANPAGE TÀI LIỆU TIẾNG ANH NÂNG CAO

H 'Until that turned up, we had no idea that ancient humans had ever reached as far as
Flores. We certainly had no idea that there was a completely new kind of human - or is it
even human? That is still being argued about – living there, and the fact that it was still
around there when modern people passed through the region. Each of those is astonishing
and that shows how little we knew about human evolution in that part of the world. We
are building up the pieces of a huge, complex jigsaw, and we still have a lot of spaces to
fill in,' he says.

Your answers
24. 25. 26. 27. 28. 29. 30.

Part 4. The passage below consists of five sections marked A-E. For questions 31-40,
read the passage and do the task that follows. Write your answers (A-E) in the
corresponding numbered boxes provided.

The Unusual Behaviour of the Bowerbird


A
Attracting a mate is one of the fundamental undertakings of life in the animal kingdom,
and many creatures go to extreme lengths or exhibit unusual techniques for this very
purpose. Take, for example, the common mouse that attracts females by its unique high-
pitched songs, or the female flamingo that adds colour to its feathers in order to appeal to
the male of the species. Indeed, there is certainly no shortage of weird and wonderful
courting rituals in the animal world, but very few of these are more unusual and
impressive than those of some species of the bowerbird, who can master DaVinci-like
feats of design and knowledge in order to win over its female equivalent. Commonly
found in Papua New Guinea and Australia, there are around ninety different species of
this bird, and their range is impressive in both size and colour. As such they exhibit a
range of efforts and behaviours in order to succeed in finding a mate.
B
Naturally, many types of bowerbird behave in ways common to other species of birds by
using physical signs and movements in order to attract others. One such example of this
is how, when a female arrives, the male’s pupils enlarge and he emits a distinctive call
from his throat as a way of indicating his interest in the female. The male, if lucky
enough that the female hasn’t already departed unimpressed by his intentions, then begins
a series of unusual jerking movements with its wings to keep her attention, a display that
has been likened to kinds of traditional human dancing such as the Paso Doble. This
display can, with some bowerbirds, culminate in perhaps one of the stranger mating
techniques in which the male sometimes begins to headbutt the female’s chest, certainly
not the kind of behaviour you’d expect from an everyday courtship! This is not just an
FANPAGE TÀI LIỆU TIẾNG ANH NÂNG CAO

intricate show, but also a well-rehearsed one in which the male frequently changes and
adjusts their signs and movements depending on his success rate in attracting a female.
C
As if this wasn’t enough, some types of bowerbirds really go the extra mile to find their
other halves, engaging in elaborate construction work that takes a considerable amount of
forward planning and hard work. Many male bowerbirds erect intricately decorated nests,
known as bowers, in a variety of elaborate ways, even stealing from other males’ bowers
in order to have the most impressive home and be chosen as a mate. These often
extremely complex bowers can be built in a tent shape, with the males placing sticks
around a small tree, or what could best be described as an igloo shape, with a passageway
entrance into a central space full of ornaments. Whatever the type of bower, they all
comprise a form of visual enhancement little seen in the animal world and more akin to
our own forms of home decoration, albeit in a simpler form.
D
Think, if you will, of a market stall trader who has all his wares on display in an enticing
fashion, showing off individual items to potential buyers in the hope of a purchase. The
bowerbird’s behaviour is reminiscent of this, with their bowers including hundreds of
tiny, often colourful objects both natural and manmade, such as flowers, berries, coins
and glass. Each of these small pieces is exactingly arranged so as to appeal to females.
While the bower’s inside is intricately decorated, the male also shows larger objects to
the female to catch their attention. And this might occur more than once, as the females
go back and forth watching the males’ displays and visiting different bowers until they
choose the bower that has caught their eye sufficiently to select the male owner as their
mate. Females commonly stop at a variety of bowers in order to select their preferred
candidate, and some males may be chosen by multiple mates, while others are passed by
altogether.
E
Recent investigations into bowerbirds and their bowers have identified that the birds
create a pattern of decoration so detailed and clever that they make their bowers appear
much bigger than what they actually are when viewed by the female. In fact, the male
bowerbird tends to go back and forth into their bowers so they can ensure they’ve
achieved the desired effect, and which they are meticulous about. Recent research shows
that if a male’s bower is altered in any way, they will painstakingly restore it to their
original design. In addition to this, incredibly, their chances of mating are found to be
directly related to the regularity of the patterns they create within the bowers. The
complexity of this mating behaviour, from both the male and female perspectives,
indicates that the bowerbird is a behaviourally complex family of birds, possibly more so
than any other bird alive today, and almost certainly the next best home architects after
humans.
In which section are the following mentioned?
31. The types of structural layout that bowerbirds use for their homes.
32. How optical illusions play a part in helping the bowerbird attract a mate.
33. The way in which female bowerbirds are impatient when it comes to mate selection.
FANPAGE TÀI LIỆU TIẾNG ANH NÂNG CAO

34. The way in which bowerbirds can be underhanded in obtaining their materials.
35. A physically aggressive mating technique used by the bowerbird.
36. The trial and error involved in making a bower visually attractive.
37. The procedure undertaken by females in mate selection.
38. The fundamental quality that makes a bower a success.
39. How the male bowerbird is almost unequalled in its mating efforts.
40. The transactional nature of the mating behaviour of the male bowerbird.

Your answers
31. 32. 33. 34. 35.

36. 37. 38. 39. 40.

Key and explanations:

Part 1.
1. i
E-learning is the unifying term to describe the fields of online learning, web-based
training, and technology-delivered instruction, which can be a great benefit to corporate
e-learning.
2. ix
In addition to generally positive economic benefits, other advantages such as
convenience, standardized delivery, self-paced learning, and variety of available content,
have made e-learning a high priority for many corporations.
3. iv
E-training promises more effective teaching techniques by integrating audio, video,
animation, text and interactive materials
Nelson reported a significant difference between the mean grades of 406 university
students earned in traditional and distance education classes, where the distance learners
outperformed the traditional learners.
4. vii
On the other hand, nobody said E-training technology would be cheap.
5. v
FANPAGE TÀI LIỆU TIẾNG ANH NÂNG CAO

E-learning isn’t expected to replace the classroom entirely. bandwidth limitations are
still an issue in presenting multimedia over the Internet. Furthermore, e-training isn’t
suited to every mode of instruction or topic. For instance, it’s rather ineffective
imparting cultural values or building teams.
6. iii
a go-between style of the Blended learning, which refers to a mixing of different
learning environments, is gaining popularity.
7. A
IBM, for instance, claims that the institution of its e-training program, Basic Blue, whose
purpose is to train new managers, saved the company in the range of $200 million in
1999.
8. B
other advantages such as convenience, standardized delivery, self-paced learning, and
variety of available content, have made e-learning a high priority for many
corporations.
9. F
a go-between style of the Blended learning, which refers to a mixing of different learning
environments, is gaining popularity.
10. D
Today, over half of the 400-plus courses that Rockwell Collins offers are delivered
instantly to its clients in an e-leaming format, a change that has reduced its annual
training costs by 40%.
11. B
For one thing, bandwidth limitations are still an issue in presenting multimedia over the
Internet.
12. C
If your company has a unique corporate culture it would be difficult to convey that to
first-time employees through a computer monitor.
13. E
In addition, there is a perceived loss of research time because of the work involved in
developing and teaching online classes.
Part 2.
FANPAGE TÀI LIỆU TIẾNG ANH NÂNG CAO

14. D
In the passage, Sara T. Smith addresses the Second Anti-Slavery Convention of
American Women. In the second sentence of the first paragraph, Smith states that
confronting slavery is “a question of justice” and that it involves “considerations of
immense importance to the welfare and prosperity of our country.” In the third paragraph,
Smith argues that women shouldn’t be deterred from participating in the abolitionist
cause. In the last paragraph, she argues that women “cannot remain inactive” in
confronting slavery as “our country is as dear to us as to the proudest statesman. . . . Let
our course, then, still be onward!” Therefore, Smith’s main purpose in the passage is to
encourage women to see their participation in the abolitionist cause as just and important.
15. A
Throughout the passage, Smith poses questions that aren’t answered explicitly until the
last paragraph, but the leading tone of the speech makes it clear that the implied answer
to these questions is “no.” In the second paragraph, Smith questions her critics’ claim that
upholding humanitarian values undermines conventional feminine virtues. In the third
paragraph, she wonders how women can “have no interest” in the subject of slavery when
it could lead to the destruction of their families through war. In the last paragraph, she
asks women numerous questions and then answers them with a “no.” Thus, a technique
that Smith uses throughout the passage to advance her main point is to present her claims
in the form of rhetorical questions that mostly have implicit negative answers.
16. B
In the first sentence of the passage, Smith introduces the argument that slavery is a
“political question” and therefore not “within the ‘province of woman.’” In the second
sentence, Smith voices her opposition to this argument: “It is not true that [slavery] is
merely a political question, it is likewise a question of justice, of humanity, of morality,
of religion.” In other words, slavery is too broad a problem to be classified solely as
“political,” in Smith’s view. However, in the fourth paragraph, Smith returns to the
political aspect of the argument at hand by addressing how women must engage in the
subject of slavery on a political scale. She argues that “admitting [slavery] to be a
political question” doesn’t mean that women have “no interest in the welfare of our
country,” as women must criticize slavery and its “unjust laws” and seek to stop the
nation’s “downward course” by choosing to not “remain inactive.” Therefore, Smith
develops her argument about slavery as a “political question” over the course of the
passage by dismissing the designation as too narrow but then demonstrates its relevance
to her audience.
17. B
FANPAGE TÀI LIỆU TIẾNG ANH NÂNG CAO

In the first sentence of the passage, Smith relays a claim: “We are told that it is not within
the ‘province of woman,’ to discuss the subject of slavery; that it is a ‘political question,’
and we are ‘stepping out of our sphere,’ when we take part in its discussion.” In the next
sentence, Smith rejects this claim: “It is not true that it is merely a political question, it is
likewise a question of justice, of humanity, of morality, of religion.” She then argues that
the subject of slavery “involves considerations of immense importance to the welfare and
prosperity of our country, enters deeply into the home-concerns, the every-day feelings of
millions of our fellow beings” and expands upon this point by providing an example of
the difference, under slavery, between laborers who are enslaved and those who are
within the “dignity of conscious manhood.” Therefore, the best summary of the first
paragraph is that Smith rejects a claim and elaborates on her reasons for doing so.
18. A
In the first sentence of the second paragraph, Smith relays the sentiment, presumably
voiced by those opposed to women abolitionists, that “woman’’ can take no part [in the
debate over slavery] without losing something of the modesty and gentleness which are
her most appropriate ornaments.’” Smith opposes this view in the following sentence:
“must woman necessarily be less gentle because her heart is open to the claims of
humanity, or less modest because she feels for the degradation of her enslaved sisters,
and would stretch forth her hand for their rescue?” The leading tone of this rhetorical
question makes it clear that Smith would answer it with a “no.” Thus, Smith argues that it
is possible for women to act according to humanitarian principles while preserving their
femininity.
19. A
The previous question asks which activity Smith argues it is possible for women to
engage in. The answer, that she argues that women can act according to humanitarian
principles while preserving their femininity, is best supported in the last sentence of the
second paragraph: “must woman necessarily be less gentle because her heart is open to
the claims of humanity, or less modest because she feels for the degradation of her
enslaved sisters, and would stretch forth her hand for their rescue?” The leading tone of
this rhetorical question makes it clear that Smith would answer it with a “no.” In other
words, Smith believes that women can uphold humanitarian principles while maintaining
conventional feminine virtues.
20. C
Choice C is the best answer. In the first sentence of the third paragraph, Smith states “by
the Constitution of the United States, the whole physical power of the North is pledged
for the suppression of domestic insurrections, and should the slaves, maddened by
oppression, endeavor to shake off the yoke of the taskmaster, the men of the North are
FANPAGE TÀI LIỆU TIẾNG ANH NÂNG CAO

bound to make common cause with the tyrant, and put down, at the point of the bayonet,
every effort on the part of the slave, for the attainment of his freedom.” In other words,
according to Smith, if slaves were to revolt, the US Constitution would require that
Northern states help the slave states fight the slaves’ rebellion.
21. D
The word “tyrant” describes a cruel and unfair ruler. It is first used in the first sentence of
the third paragraph, when Smith argues that in the event of a slave rebellion in the slave
states, “the men of the North are bound to make common cause with the tyrant, and put
down, at the point of the bayonet, every effort on the part of the slave, for the attainment
of his freedom.” The word occurs again in the seventh sentence of the last paragraph,
when Smith asserts the strength of women’s “aspirations that every inhabitant of our land
may be protected . . . by just and equal laws” so that “the foot of the tyrant may no longer
invade the domestic sanctuary.” In both instances, the word “tyrant” is used to represent
slaveholders and their allies. Thus, Smith’s use of “tyrant” emphasizes the unjustness of
slavery.
22. C
In the last sentence of the third paragraph, Smith argues that the threat of a war
precipitated by slavery “is of itself sufficient to arouse the slumbering energies of
woman” to speak out against slavery’s injustice. In other words, women have the
potential to protest slavery, but they have been relatively inactive, or dormant, up until
now. Therefore, the word “slumbering,” as used in this sentence, most nearly means
dormant.
23. A
The fifth sentence of the last paragraph poses the following question: “Shall we silently
behold the land which we love with all the heart-warm affection of children, rendered a
hissing and a reproach throughout the world, by this system which is already tolling the
death-bell of her decease among the nations?” In other words, the continuation of slavery
in the United States is being criticized “throughout the world,” such that the existence of
slavery affects the United States by lowering the country’s reputation in the international
community.
Part 3.
24 F: The first paragraph uses a book metaphor and talks about 'the story of the past'.
Paragraph F introduces the book that Chris Stringer and Peter Andrews have written.
25 C: The paragraph before the gap mentions the newly discovered human species for the
first time and compares it to other known early human species. This new evidence raises
questions, which are then posed in paragraph C.
FANPAGE TÀI LIỆU TIẾNG ANH NÂNG CAO

26 H: The paragraph before the gap refers to Stringer's background and ends with the
statement' Homo floresiensis, however, astonished him', which suggests that some
comments by Stringer may follow. The paragraph after the gap consists of a quote, with
no mention of the speaker's name, which implies that the speaker is continuing a
comment which began in the gapped paragraph. Paragraph H is a quotation, and explains
what the speaker finds 'astonishing' about the new discovery.
27 G: The paragraph before the gap explains the nature and complexity of human
evolution. Paragraph G describes the accepted view of human evolution using the
metaphor of a tree. In the paragraph after the gap, this metaphor is continued: 'Modern
humans probably popped up within the last 200,000 years, but the things that make
modern humans so distinctive in the fossil record - symbolic art pottery and jewellery -
bloomed only about 50,000 years ago.'
28 A: The last sentence of the paragraph before the gap suggests that our own species is
transient. Paragraph A continues this idea: 'We should not see ourselves as the summit of
the perfection ... how successful will we look in 50,000 years ... ?' The paragraph after the
gap says that humans nearly died out several times in the course of their evolution.
29 B: The paragraph before the gap mentions the Neanderthals, and raises questions
about their relations with other human species. Paragraph B considers the Neanderthals
and raises a further question: did other human species regard them as different from
themselves? Stringer's answer to this question is in the paragraph after the gap.
30 D: This paragraph presents Stringer's view of what became of the Neanderthals,
concluding what has been said in the three previous paragraphs, and leads into the final
paragraph, regarding the authors' intentions in writing their book.
Part 4.
31. C
These often extremely complex bowers can be built in a tent shape, / or what could best
be described as an igloo shape, with a passageway entrance…
32. E
create a pattern of decoration so detailed and clever, that they make their bowers appear
much bigger than what they actually are when viewed by the female. / tends to go back
and forth into their bowers so they can ensure they’ve achieved the desired effect…
33. B
The male, if lucky enough that the female hasn’t already departed unimpressed by his
intentions…
FANPAGE TÀI LIỆU TIẾNG ANH NÂNG CAO

34. C
even stealing from other males’ bowers, in order to have the most impressive home and
be chosen as a mate.
35. B
in which the male sometimes begins to headbutt the female’s chest …
36. E
In fact, the male bowerbird tends to go back and forth into their bowers so they can
ensure they’ve achieved the desired effect…
37. D
And this might occur more than once, as the females go back and forth watching the
males’ displays and visiting different bowers until they choose the bower that has caught
their eye… / commonly stop at a variety of bowers in order to select their preferred
candidate…
38. E
…incredibly, their chances of mating are found to be directly related to the regularity of
the patterns…
39. A
no shortage of weird and wonderful courting rituals in the animal world, but very few of
these are more unusual and impressive than those of some species of the bowerbird…
40. D
Think, if you will, of a market stall trader, who has all his wares on display in an enticing
fashion, showing off individual items to potential buyers in the hope of a purchase. The
bowerbird’s behaviour is reminiscent of this, with their bowers including hundreds of
tiny, often colourful, objects both natural and manmade, such as flowers, berries, coins
and glass. / While the bower’s inside is intricately decorated, the male also shows larger
objects to the female to catch their attention.
FANPAGE TÀI LIỆU TIẾNG ANH NÂNG CAO

You might also like